Young Christian: Volume 4, 1914

Table of Contents

1. Inspiration of the Scriptures: The Bible - its Unity, Part 2
2. Oh, How I Want to See the Man That Saved Me!
3. Truths for Young Christians: Backsliding
4. Scripture Study: Matthew 18
5. Nehemiah: The Building of the Wall, Part 1
6. What God Hath Said on the Second Coming of Christ and the End of the Present Age: Part 1
7. Today
8. Correspondence: Meaning of Ecc. 11:2
9. Inspiration of the Scriptures: The Bible - its Unity, Part 3
10. The Happy Soldier
11. Truths for Young Christians: Restoration
12. Absolvo Te
13. Scripture Study: Matthew 19
14. Nehemiah: The Building of the Wall, Part 2
15. What God Hath Said on the Second Coming of Christ and the End of the Present Age: Part 2
16. Correspondence: Rewards; Day of the Lord; Cain's Wife
17. Inspiration of the Scriptures: The Bible - its Unity, Part 4
18. I Shall Be Satisfied
19. He Shall Be Satisfied
20. In Season and Out of Season
21. Truths for Young Christians: Story of a Second Conversion
22. Scripture Study: Matthew 20
23. Nehemiah: The Building of the Wall, Part 3
24. What God Hath Said on the Second Coming of Christ and the End of the Present Age: Part 3
25. Correspondence: Bible Inspired and Apocrypha Not
26. Inspiration of the Scriptures: The Bible - its Unity, Part 5
27. Practical Conversations With Our Young People
28. Questions and Answers for March
29. Questions for April
30. A Word Fitly Spoken
31. Truths for Young Christians: Young Men Wanted, Part 1
32. Scripture Study: Matthew 21
33. Nehemiah: The Building of the Wall, Part 4
34. What God Hath Said on the Second Coming of Christ and the End of the Present Age: Part 4
35. Correspondence: John 8:56; Isa. 28:17-18; Baptism
36. Inspiration of the Scriptures: The Bible - its Unity, Part 6
37. Practical Conversations With Our Young People: Unequally Yoked
38. Questions for May
39. Results Are in God's Hand
40. Truths for Young Christians: Young Men Wanted, Part 2
41. Human Happiness and Heavenly Joy
42. Scripture Study: Matthew 22
43. Nehemiah: The Building of the Wall, Part 5
44. What God Hath Said on the Second Coming of Christ and the End of the Present Age: Part 5
45. Forward
46. Correspondence: 1 Peter 4:18; Acts 2:5
47. Inspiration of the Scriptures: The Bible - its Perfection, Part 1
48. The Unfailing Word of Prophecy
49. Practical Conversations With Our Young People
50. Questions for June
51. Truths for Young Christians: Undoubting Faith in God
52. Scripture Study: Matthew 23
53. Nehemiah: The Building of the Wall, Part 6
54. What God Hath Said on the Second Coming of Christ and the End of the Present Age: Part 6
55. Correspondence: Acts 8; Matt. 28:9 and John 20:17
56. Inspiration of the Scriptures: The Bible - its Perfection, Part 2
57. The Christian Traveler
58. Practical Conversations With Our Young People: Obedience
59. Questions for July
60. Answers to Questions for May
61. Truths for Young Christians: Service, Part 1
62. Scripture Study: Matthew 24:1-20
63. Nehemiah: The Building of the Wall, Part 7
64. What God Hath Said on the Second Coming of Christ and the End of the Present Age: Part 7
65. Correspondence: Rom. 8:16 and 1 John 5:10, The Witness of the Spirit
66. Inspiration of the Scriptures: The Bible - its Perfection, Part 3
67. The Christian Traveler
68. Practical Conversations With Our Young People: Questions for August
69. Answers to Questions for June
70. Truths for Young Christians: Service, Part 2
71. Steadfast, Unmovable, Always Abounding
72. Scripture Study: Matthew 24:32-51
73. What Is the Camp? Hebrews 13:12-13, Part 1
74. What God Hath Said on the Second Coming of Christ and the End of the Present Age: Part 8
75. Correspondence: Daniel 7:9, 13; 1 Cor. 4:4
76. Inspiration of the Scriptures: The Bible - its Perfection, Part 4
77. The Mission of Tracts
78. Practical Conversations With Our Young People: The Need of Dependence
79. Questions for September
80. Answers to Questions for July
81. Truths for Young Christians: Three Sore Evils
82. Matthew 24: Scripture Study
83. What Is the Camp? Hebrews 13:12-13, Part 2
84. What God Hath Said - on the Second Coming of Christ and the End of the Present Age: Part 9
85. Extracts: Selfishness
86. Correspondence: The Lord's Will; the Epistle of James; Rom. 5:3
87. Inspiration of the Scriptures: The Bible - its Perfection, Part 5
88. Their Sins and Iniquities Will I Remember No More: Hebrews 10:17
89. Practical Conversations With Our Young People: Assembling Ourselves Together
90. Questions for October
91. Answers to Questions for August
92. My Desire
93. Truths for Young Christians: Christian Devotedness, Part 1
94. Scripture Study: Matthew 25
95. What Is the Camp? Hebrews 13:12-13, Part 3
96. What God Hath Said on the Second Coming of Christ and the End of the Present Age: Part 10
97. Correspondence: Sabbath; Giving Thanks Always; John 21:11
98. Inspiration of the Scriptures: The Bible - its Blessedness, Part 1
99. An Hour of Prayer
100. Practical Converstions With Our Young People: A Precious Service in Matthew 27-28
101. Questions for November
102. Answers to Questions for August
103. Truths for Young Christians: Christian Devotedness, Part 2
104. Scripture Study: Matthew 26:1-30
105. Worship
106. They That Are Christ's at His Coming: Part 1
107. Correspondence: Life at the Judgement; Great White Throne
108. Inspiration of the Scriptures: The Bible - its Blessedness, Part 2
109. The Arithmetic of Heaven
110. Practical Conversations With Our Young People: God's Will and Our Work
111. Questions for December
112. Truths for Young Christians: Christian Devotedness, Part 3
113. Scripture Study: Matthew 26:31-75
114. Worship
115. They That Are Christ's at His Coming: Part 2
116. Correspondence: New Birth and Possession of the Holy Spirit

Inspiration of the Scriptures: The Bible - its Unity, Part 2

If a friend handed to us a ponderous volume, consisting of sixty-six books, written by between thirty and forty persons, and at different times, extending over fifteen hundred years, and said, Notwithstanding all their differences, there is a remarkable unity throughout, should we not be astonished? As a matter of fact, there is no such book as the Bible in this respect; nor could there be, unless all the writings it contained had been under the guidance of One mind, and its communications throughout given by the One Spirit.
One thing which would be likely to strike some persons in considering the principle of unity in a book, would be to compare the end with the beginning, and see if there be any connection as to similarity or contrast. In the Bible it is written, “Known unto God are all His works, from the beginning of the world” (Acts 15:18). Let us turn and examine a few scriptures as to this.
The first words we find in the Bible are, “In the beginning God created the heaven and the earth” (Gen. 1:1), and, as a matter of fact, much of the Bible is about God’s heavenly and earthly people; also as to things in connection with the present heaven and earth; and in the end of the book we read of “a new heaven and a new earth” (Rev. 21:1).
In the beginning of the book it is said, “Let there be light, and there was light”; afterward we are told that Christ is “the light of the world”; and in the end of the book we read, that “the Lamb is the light thereof.”
In the beginning, we read of a tree of life in the garden of Eden, from which man was afterward excluded through his sin; in the end we find “the tree of life” with its many fruits, and are taught that the faithful will eat of “the tree of life, which is in the midst of the paradise of God” (Gen. 2:9; Rev. 2:7; 22:2).
A river, too, was in Eden, and at the end of Revelation we read of “a pure river of water of life, clear as crystal, proceeding out of the throne of God and of the Lamb” (Gen. 2:10; Rev. 22:1).
In Genesis 2 we see the first man (figure of Him that was to come) and his help-meet, of whom he could say, “This is now bone of my bones and flesh of my flesh”; and in Revelation 21 we have presented to us “the bride, the Lamb’s wife....having the glory of God”; of whom it had been said, that He “nourisheth and cherisheth it,” and that “we are members of His body, of His flesh, and of His bones” (Rev. 21:9,11; Eph. 5:29-30).
In the earthly paradise man was in dominion over the fish of the sea and over the fowl of the air, and over every creeping thing, and the name he gave to every living creature that was the name thereof; but toward the end of the book, the Lord Jesus, the last Adam, will bring this groaning creation into the liberty of the glory of the children of God, and have His rightful place as “Lord of all,” having subdued all things unto Himself (Gen. 1:28; 2:19; Psa. 8; Phil. 2:10-11; Phil. 3:21). In the beginning we have Satan tempting; then sin, and the curse; and in the end, we see Satan in the lake of fire, sin taken away, righteousness dwelling, and no more curse. In the beginning sorrow and death; in the end, “no more death, neither sorrow nor crying, neither shall there be any more pain; for the former things are passed away” (Gen. 3; Rev. 20:10; 2 Peter 3:13; 1 John 3:5; Rev. 21:1-4).
Surely, then, we find a remarkable unity of thought in the beginning and ending of the Bible, though the contrasts are most striking; because the Son of God had come meanwhile to accomplish redemption, destroy the works of the devil, take away sins, make good the promises, vindicate God in all His ways, honor Him in perfect obedience as Man, and glorify Him in clearing us from all iniquity, and bringing us to God, to share the inheritance with Him who is Heir of all things.
Another mark of unity is found in the truth it sets forth throughout. If early in the Old Testament it is said of man, that “the imagination of the thoughts of his heart was only evil continually,” it is said in the New Testament that “the carnal mind is enmity against God; for it is not subject to the law of God, neither indeed can be” (Gen. 6:5; Rom. 8:7). If a prophet in olden time said, “all flesh is grass, and all the goodliness thereof is as the flower of the field....the grass withereth, the flower fadeth; but the Word of our God shall stand forever”; an apostle, seven hundred years after, writes the same, only adding to “the Word of the Lord endureth forever,” “And this is the Word which by the gospel is preached unto you” (Isa. 40:6-8; 1 Peter 1:24-25). If the Psalmist exclaimed, “Forever, O Lord, Thy word is settled in heaven,” our Lord said, “Heaven and earth shall pass away, but My words shall not pass away” (Psa. 119:89; Matt. 24:35). If the testimony of a prophet was, “Not by might, nor by power, but by My spirit, saith the Lord of hosts,” an apostle informs us, that “the things of God knoweth no man, but the Spirit of God” (Zech. 4:6; 1 Cor. 2:11). If Moses was inspired to write, “it is the blood that maketh an atonement for the soul,” we read in Hebrews, that “without shedding of blood is no remission” (Lev. 17:11; Heb. 9:22). If an Old Testament writer warned the people not to “add unto the word” which he commanded them, “neither shall ye diminish ought from it,” the ancient writings are not closed without enforcing the exhortation by saying, “Add thou not unto His words, lest He reprove thee, and thou be found a liar”; nor can the canon of Scripture be concluded without the last of Revelation giving us the most solemn warning concerning it. (Deut. 4:2; Prov. 30:6; Rev. 22:18-19).
Take another subject. All through the entire volume, from Genesis to Revelation, we find since man became a sinner, that he has been accounted righteous before God on the principle of faith, and never on the principle of works; a fundamental truth of vital importance. We read, that God clothed Adam and Eve with coats of skins; that is, their nakedness could only be truly covered up from the eye of God through the benefit derived from the death of a sacrifice. Abel’s offering shows out the same. Also in Genesis 15; we read, “Abram believed in Jehovah and He counted it to him for righteousness”; and David, who lived nine hundred years after, describeth the blessedness of the man to whom the Lord imputeth righteousness without works, saying, “Blessed are they whose iniquities are forgiven, and whose sins are covered. Blessed is the man to whom the Lord will not impute sin” (Rom. 4:6-8). These scriptures are quoted by the Apostle Paul to make clear to us, that the principle on which all are justified from all things, is that of faith, without the deeds of the law. Hence, “the righteousness of God which is by faith of Jesus Christ, unto all and upon all them that believe” (Rom. 3:22,28).
The typical instruction in the Old Testament having its accomplishment in the New Testament, gives a remarkable complexion of unity to the whole Bible. Take, for instance, Abraham offering up his loved and only son, Isaac; what an accurate fulfillment of the type we have in God’s delivering up His only-begotten Son as a sacrifice for us! In this one instance we have shadowed forth divine love and grace in laying our iniquity on Him; divine righteousness in judging unsparingly our sins on Him instead of on us; and divine power to usward in raising Him up from the dead, and giving us risen life in association with Him.
(To be Continued).

Oh, How I Want to See the Man That Saved Me!

It is a great thing when the soul gets beyond the fact of its deliverance, wonderful and blessed as that is, and lays hold by faith upon the person of the Deliverer; for it is being occupied with Him, in the having to do with Himself personally, and addressing Him in happy, assured confidence of heart as one now known and delighted in, that positive and increased blessing of an inexhaustible character consciously comes to the believer.
The more I value the immensity of the blessing I now possess, the more surely should I desire to make the acquaintance of the one who has conferred so wonderful a boon upon me at the cost of the sacrifice of Himself.
How much, dear reader, do you and I know of personal intimacy with Him of Jacob’s well at Sychar, and of Martha’s cottage at Bethany, now the Man enthroned in glory? How much of that individual intercourse with Himself, without which each recurring day would be to us a cold and cheerless blank?
Alas, how many habitually grieve Him by accepting the benefit bestowed, while exhibiting indifference as to the Benefactor, depriving themselves thus of that peculiar joy which fills the heart for the first time in that thrilling moment when we are conscious of what has never dawned upon us before – that we are personally known to Christ and He to us! Nor can we doubt that it is equally a time of joy to Him when a soul in the beauty of its new-born spiritual life is brought thus fully into conscious acquaintance with Himself, to enter upon an intercourse as lasting as God’s eternity.
It is the occasion on which the believer can say, “Well, now I know Him, my Savior! Not merely do I know what He did for me when He was here; but I know the one who has left the scene, the Man now in glory, and have been so brought into the secret of His own presence to have immediate contact with Himself, that, indeed, I know Him more intimately than I know any earthly relative, and am known of Him infinitely better than by any such!”
We are fully persuaded that hundreds of believers, who are well assured of the blessing they have received, go on in coldness and leanness, withered and stunted in soul, because of the absence of this. What they need is to have their hearts stimulated to seek this direct knowledge, in cultivated and constant intercourse, of the person of Christ. The Spirit of God loves to conduct the soul of the believer to Christ now, as also He will his body by-and-by, when morning breaks and glory dawns. For us nothing could be more profitable or more blessed; for Him no tribute so acceptable!
It is as somewhat illustrating this point that we put the following little narrative before the reader.
A few years ago a poor woman, one of a number who earn a scanty living by washing at the riverside near Glasgow, and whose only possession was the tub in which her daily task was performed, had the misfortune to fall into the Clyde, and as the river was deep and the current strong, her case was imminent, no help being apparently at hand. Suddenly a man who was a renowned swimmer and had saved many lives, plunged into the stream; but only by extreme exertion, and well-nigh at the cost of his own life, did he succeed in rescuing the object of his solicitude. The old woman herself had been so long submerged that animation was suspended, and no little effort was requisite before consciousness returned.
And now, dear reader, what do you think were the first words which, issuing from her lips, manifested to those around that she had really come back, as it were, from death to life?
Some expression of anxiety as to her home, her family, her friends? Some disclosure of her feelings while in the jaws of death, or on her discovery that she had been rescued?
No, nothing, nothing of this! But words that should be a touching lesson for us, who have been further gone than she towards a far more terrible fatality, and who have been rescued, not at the almost, but at the actual cost of another’s life. Her words were those which head this paper, “O, how I want to see the man that saved me!” Beautiful exclamation in the mouth of one who had nearly perished, but whose unselfish gratitude led her to concern herself about him whose self-sacrificing work had brought her back from death.
The man came at her word. Again she spoke, “O, sir,” she said, “you’ve saved me, and I’ve naught in the world save yon tub; but, O! if you’ll take it you’re welcome, with all my heart!” The man, no less astonished than gratified, made no reply, but doffing his hat, went round collecting from the assembled crowd, and speedily coming back poured all he had received into her lap, enriching her as she had never in her life either experienced or expected.
Is it not thus, though in an infinitely higher and more blessed way, that God, having given us eternal life in Christ, with Him also freely gives us all things? Have we, like the poor woman, experienced deep longings of heart to see the one who has saved us, and when we have made His acquaintance laid all we possess with all our heart at His feet? If so, surely we shall have found that, inasmuch as it is more blessed to give than to receive, He will be no man’s debtor; but taking to Himself the higher blessedness which is His due, He will pour into our lap all that He has received, to share with us the spoils of His own victory, the guerdon of His own work! And thus to us shall belong the double and lasting indebtedness which our narrative illustrates.
May we who have been so wondrously blessed, and who sometimes sing of Him, “And gave us all that love could give,” be led of the Holy Spirit into personal acquaintance with the Man in glory whom grace has made our satisfying portion forever. And may the taste we thus acquire for what we more and more find only in Himself intensify, as it surely will, the longing desire of our hearts to see Him face to face, when the day dawns and the shadows flee away!
“In this was manifested the love of God towards us, because that God sent His only begotten Son into the world that we might live through Him. Herein is love, no that we loved God, but that He loved us, and sent His Son to be the propitiation for our sins” (1 John 4:9-10).

Truths for Young Christians: Backsliding

We have three great enemies ever seeking to overcome us: the world, the flesh, and the devil; and in proportion as we give place to any of these, do we depart from God. These three we find in Peter’s case in Luke 22. In Luke 22:45-50 he is led astray by the flesh, in sleeping when he should have watched, in striking when he should not have resisted. In Luke 22:54-55 he is led astray by the fear of the world: first, in straying far from Christ’s side; secondly, in fellowship with His enemies. And, lastly, in Luke 22:58,60, he is thrice led astray by the devil: to deny Christ, to swear, and to deny Him again.
One might, indeed, say such a course is foreshadowed in the first Psalm. The counsel of the ungodly, the dictates of fleshy reason, led to the smiting with the sword; standing in the way of sinners is illustrated by standing and warming himself; while sitting in the seat of the scornful is found in Luke 22:55.
And now, dear reader, what about yourself? Listen to the following words: There is no heart, in the wide world, so unhappy as his, who has been drawn aside from the holiness and joy of obedience, to paths of self-seeking and of sin.
The Path of the Backslider
“What peaceful hours I once enjoy’d
How sweet their memory still!
But they have left an aching void
The world can never fill.”
And such is the language, in poetry or in prose, of the soul, whose “earliest love” has been left who has, alas! in some way or other, forsaken the Lord, for the enjoyment of the favors of the world.
“My people have committed two evils; they have forsaken Me, the fountain of living waters, and hewed them out cisterns, broken cisterns, that can hold no water” (Jer. 2:13). Such was God’s lamentation of old. How rightly He styled Himself “THE FOUNTAIN OF LIVING WATERS” – the source and spring of blessing; and how solemnly descriptive is the expression, “broken cisterns, that can hold no water,” of the experience acquired by departing from Him.
He knows where the blessing is found. We, alas! often through seas of sorrow, have to learn that the cisterns to which we have recourse are, in truth, broken, and that they hold no water, and that there remains, as the only result of our declension, “an aching void,” a distracted and discontented heart; a state of soul, indeed, which had no parallel in the most wretched hours of our unconverted days.
Ah! beneath many a smiling face, behind many a ringing laugh, underlying much forced activity and unnatural effort, there is to be found a heart of misery, that seeks by these means to conceal the fact of its departure from God.
And yet how vain that effort – how hollow that laugh! The stag may continue to bound gaily over crag and moor, and the bird may soar awhile swiftly on high, but the gunshot wound is doing its work, and, sooner or later, the gay bounding will cease and the strong wing will droop. So, too, the Word of God will prove effectual, though long slighted; and the wayward soul, though brought by paths of deep and searching trial, will find that the love, wherewith it was loved, was an “everlasting love”; such a love as could turn its eye, full and forgiving, on a poor failing Peter, and effect by its silent, yet wounded look, his entire restoration.
Thou Hast Left Thy First Love
Do You not own and feel the truth of these words? Can you not recall, with an aching heart, the bright and holy memories of the past, the once loved Bible, the place where “prayer was wont to be made,” the happy work for your Lord? It may be some poor, cold, formal task, professedly for Him, still occupies you, but all the time you hear its voice ever saying, “Thou hast left thy first love.” You have gradually not only left the things you once loved, but returned to those you once bated for Christ. The ensnaring novel, eating away your brain and time, the worldly song, the amusements of this world, are all binding their chains around you, and you are not happy. You try to be, but you cannot succeed. You envy the happy carelessness of the dead souls around you. They feel no remorse, the pleasures of the world contain no hidden sting for them. They have never known and loved the Savior you have forsaken. The voice of conscience is not ceaselessly saying to them, as to you, “You are doing wrong. You are sinning against the light.” Consider now where was your first step of departure? Was it not so small as to be almost imperceptible? You did not begin by throwing away your Bible for a romance, you did not at once exchange the meeting far the concert hall. No! the first thing was a gradual neglect of private reading and prayer. As your heart got cold, and you lost your interest in it, the devil whispered, “Give it up, it is no use going on with a form; wait till your heart gets warm again”; well knowing that in saying this, he was cutting you off from the warmth and light. And you obeyed him. You did not read, or pray, this morning when you arose, nor yesterday, nor the day before. O! beloved reader, truly yours is a sad case; but yet, there is abundant grace to meet it.

Scripture Study: Matthew 18

The teaching of this chapter is important for the present time, when Christ is rejected and absent, and before the kingdom in glory, as in the 17th chapter, has come. It is connected with the 16th chapter, and unfolds the principles that belong to the kingdom and the church, as seen at the present time. It reveals the ways of God in regard to the assembly, and the character suited for this testimony.
Matthew 18:1-17. The disciples ask. “Who is the greatest in the kingdom of heaven?” And Jesus called a little child unto Him, and set him in the midst of them, and said, Verily, I say unto you, expect ye be converted, and become as little children, ye shall not enter into the kingdom of heaven.” In the little child we get the spirit of dependence, meekness and humility, and they must become as little children. This is the proper spirit for one connected with a rejected Lord. “Whosoever therefore shall humble himself as this little child, the same is greatest in the kingdom of heaven. And whoso shall receive one such little child in My name, receiveth Me.”
On the other hand, he who caused one of the Lord’s believing little ones to be stumbled, should be visited with terrible judgment. Woe unto the world, because of such offenses. And it matters not, if what hinders is as valuable to the disciples as a hand or foot or eye, it is better to be losers now, than be lost for eternity: better to enter into life maimed or with one eye, rather than to be cast into eternal fire.
They must exercise the greatest care in grace to help those believing little ones, and use the most unrelenting severity towards themselves in whatever might ensnare them. That which is only for time, is nothing compared to eternity. Severity toward self, and caring in grace for others, is the rule of the kingdom.
They were not to despise the little ones, such as the Lord set before them, for if unable to look after themselves, they were the objects of the Father’s care, and they had one to represent them in His presence. They were not without sin in them, but sin not active, for they were but infants. The Father did not despise them, and the Son of Man came to save that which was lost. Here the child’s sinful state is seen, and the sacrifice of Christ is also seen meeting it. (Note: The word “seek” is not in this chapter, indicating that He is speaking of one who has not yet reached the years of responsibility). In Luke 19:10, He speaks of those grown up.
The disciples were to have the humble and confiding spirit of a little child, and they were also to have the spirit of the Father, in their care over the weak and the lowly. This is the character suited to the kingdom (Matt. 5:43-48).
In regard to offenses against one’s self, the spirit of meekness and regard for the offender’s good was to be exercised; he was to gain his brother. He was to go and tell him his fault alone; if he hearkened to him, his brother’s soul would be restored, and the matter would end there; if he would not hearken, he was to take one or two more, that in the mouth of two or three witnesses every word might be established; if these means of restoration failed, it was to be made known to the assembly; and if this did not break him down, he was to leave him alone, as one of the nations or tax-gatherers – one not suitable company for a believer.
This is not discipline in the assembly; nor is it telling the assembly how to act. (The assembly must find out how to exercise discipline from the Lord in their midst). This is instruction to a Christian, and shows the spirit he should walk in towards one who had offended him; and desire to gain his brother, is to be his object.
Matthew 18:18-20 gives the assembly with Christ in their midst. In Matthew 16:18, He spoke of building it. Here it is a gathered company, and what was exercised by Peter as an apostle in Matthew 16:19, is here conferred on the assembly with the Lord’s presence in their midst.
The without and the within applies now to the assembly (1 Cor. 5:12).
If we look at Paul’s writings we will find much instruction about the Holy Spirit’s presence and power in the assembly, and gathering Christians as members of one body to Christ the Head; but in Matthew the assembly is recognized with Christ’s presence in the midst of those gathered to His name.
Matthew 18:18-20. Heaven binds what is bound by the assembly – the two or three.
Matthew 18:19. The Father grants what they ask in prayer. “For where two or three are gathered together to My name, there am I in the midst of them.”
The unity of the spirit (Eph. 4:3) would teach us that what is done in one place in Christ’s name, is done for the whole church of God everywhere.
Deeply important are these verses. Wondrous favor and dignity put upon the two or three when truly gathered to His name. But sad, indeed, when claimed by some who only pretend to it, in whose lives or associations there lacks separation from evil, or the unity of the spirit – qualities which are invoked in that worthy name – His name.
Matthew 18:21 leads us to another characteristic of Christ in the believer as suited to His kingdom, namely, pardoning grace. The children of the kingdom are imitators of God (Eph. 5:1-2), who is always ready to forgive. The assembly, having the Lord in the midst, needs to judge evil through discipline in order to clear His name from that which is not in keeping with His character. His name must not be associated with evil; and for the good of the one who sinned, that his conscience may be aroused to see God’s hatred to evil. But individually, we need to be ready to pardon without end, when the wrong is confessed.
Matthew 18:23. In this parable we have a picture of the Jews. They had not only broken the law, but crucified the Son of God. On the cross the Lord interceded for them: “Father, forgive them, for they know not what they do!” This prayer is answered in Acts 3 by Peter giving them, as a nation, another offer of pardon. This also was refused. From Stephen’s murder, they are given up as a nation, to punishment. Their wickedness is still seen in 1 Thessalonians 2:14-16, and there they will remain until the Lord can say, “Comfort ye, comfort ye, My people, saith your God .... she hath received of the Lord’s hand double for all her sins” (Isa. 40:1-2).
Salvation is individual at this present time, for Jew or Gentile, but “not My people” is written now upon the Jews till the day of their restoration comes (Hos. 1:9-10).
Matthew 18:35 gives this a practical application for us all. If the spirit of grace and forgiving love is not filling our hearts, we are not in happy communion with our Father.
Thus we see that the spirit of the kingdom now is not outward power, but lowliness wrought in us by the Father’s love and grace; the enjoyed favor of God keeps us from seeking earthly greatness. Filled with the spirit of grace, we will cherish the lowly, and pardon those who have wronged us. The character of God is seen in us as we walk in communion with Him.
The assembly is made up of individuals, so that each of us should have this character. The assembly represents Christ on earth, for He is in their midst, and what is of God should be seen in it. May He give us grace to enter into this blessed place and portion, as two or three gathered to Christ’s name.

Nehemiah: The Building of the Wall, Part 1

“Ought ye not to walk in the fear of our God?” “The fear of the Lord is the beginning of wisdom.” In that fear, desiring to know and do the will of God, let us look at the lessons in Nehemiah, written for our instruction. If we study this book in the presence of the Lord, we shall hear Him speaking to us in it, as to present events.
In Nehemiah 1, we see a man before God. He learns the state of the remnant of the Jews, and that the wall of Jerusalem is broken down. He bows in confession and prayer. Deeply earnest is this man of God, as he pleads with Jehovah for the state of the fallen, yet the redeemed by power. Thus he pleads: “O Lord, I beseech Thee, let now Thine ear be attentive to the prayer of Thy servant, and to the prayer of Thy servants, who desire to fear Thy name” (Neh. 1:11).
Thus we see him before the Lord, feeling acutely the state of Israel and the city of the Great King. He owns fully their deep sin in departing from the Lord: “We have dealt very corruptly against Thee, and have not kept the commandments, nor the statutes, nor the judgments which Thou commandest Thy servant Moses.”
Now, as these things were written as types for us, may I ask, Have we been thus before the Lord, in deep confession, as to the present state of the church of God? Have we thus wept, and mourned, and prayed, for the blood-bought people of the Lord in this day?
Let us seek no mere controversy, but sit down before the Lord, and compare the present captivity of the church in the world with what it was in the beginning. Has not the wall been broken down? When God by the Holy Spirit first built the church, there was the wall of separation. All believers were together, and formed one body, as all the houses in the ancient city formed the one Jerusalem, with its wall strong and high. Even so we read of the one church of God, “and of the rest durst no man join himself to them” (Acts 5:13). Have you sat down before the Lord? Look, then, back along the dark ages, the centuries of captivity, wherein this wall of separation has been broken down.
As God prepared Nehemiah, by this deep exercise of heart in His own presence, for his future work, so has God been pleased in like manner to raise up servants, prepared by Himself, for special work. But there must be this process of heart-preparation. I would not write another word for controversy, but there are many souls bowed down at the thought of what calls itself the church; God will use these thoughts for their help, and, I trust, for His own glory.
After deep prostration and exercise before God, in Nehemiah 1, we find as the result, divine yearnings and activities of love for the welfare of the people of God in Nehemiah 2.
All this brings before us for the first time, Sanballat the Horonite, and Tobiah, the servant, the Ammonite. Now, as these and their companions are brought before us throughout this book as the enemies and opposers of Nehemiah, and his work for God in building the wall, it is important to know who they were, and whom they represent. They were, then, Horonites, Ammonites, and Arabians. But they were dwelling in the land of Israel in Nehemiah 4:7; Sanballat spake before his brethren and the army of Samaria; and Samaria in the beginning formed part of the land of Israel – they were active, boastful, subtle, men of authority in the land, but not of it. Do they not, then, represent the active, boastful, subtle men of authority who are in the professing church, but who are really strangers to God, and not of the church at all, but are the enemies and opposers of those desirous of carrying on the work of God, in caring for the saints, and in building the wall of separation to God?
If we now turn to the history of these men, we shall find seven forms or aspects of enmity to God’s work. “When Sanballat, the Horonite, and Tobiah, the servant, the Ammonite, heard, it grieved them exceedingly that there was come a man to seek the welfare of the children of Israel” (Neh. 2:10). And when has God raised up a man in like manner, to seek the real welfare of the church of God, but those have been found – and not a few of them – who have been grieved exceedingly? How great was the grief of the clergy when God raised up a Wiclif, a Huss, or a Luther! But especially do we find these seven marks of opposition to the work of God during this last half century. What a grief it has been to many, that God should have raised up men to seek the real welfare of the church of God, apart from all sectarianism. Some years ago men were brought, like Nehemiah, on their faces before the Lord. Amazed at the departure of the church from the commandments of her Lord, they were bowed in confession and prayer. And the Holy Spirit put earnest yearnings in their hearts for the one church of God.
Philadelphia (Rev. 3) answers to Nehemiah, as antitype answers to type. One must be alone a good deal with God to understand this. There were but few men with Nehemiah when he arose in the night and no man knew what God had put into his heart. Just take a ride with him around Jerusalem. Dragon wall and dung port wall broken down. Such are the things you will find in and around the church in ruins. That is the church as seen in the hands of men.
Very clearly have the Scriptures foretold all this. The present state of Christendom is most accurately described in the Word. (See 2 Tim. 3; 2 Peter 2:1-9; culminating in Rev. 17-18). Its progress is marked in detail in its seven stages in Revelation 2-3. Neither is there one intimation that it would be restored to its primitive glory as the bright witness of a rejected Christ. A feeble remnant is found in Philadelphia, clinging to the person and Word of Christ, and keeping His patience.
As Nehemiah, then, rode round Jerusalem (Neh. 2:11-16), so ride around Christendom. O, I ask you to reflect, what are God’s thoughts about Romanism and Protestantism? View the whole scene in the presence of God and in His fear. Did Nehemiah hang down his hands in despair? No! he said, “Ye see the distress that we are in, how Jerusalem lieth waste, and the gates thereof are burned with fire: come and let us build the wall of Jerusalem.” Thus he encourages them, and the hand of God was good upon him. They reply, “Let us rise up and build.”
This brings us to the second form of opposition. “But when Sanballat the Horonite, and Tobiah, the servant, the Ammonite, and Geshem the Arabian, heard, they laughed us to scorn, and despised us, and said, What is this thing that ye do? Will ye rebel against the king?” The first from of opposition was grief; the second is laughter. Compared with the whole nation, they were indeed a feeble remnant.
They longed to see the sacred city enclosed within the wall of separation. And shall that city of the king be more dear to them than the sacred enclosure of the saints of God, around the person of Christ, be to us? As Nehemiah stirred up the remnant to build the wall, so has the Holy Spirit stirred up a few, each in his place, to build this wall, so long cast down. O, how the modern Sanballats have laughed and despised! What is this thing that you feeble, silly, Christians will do? Yes, there has been a time for grief, and a time of laughter.
(To be Continued).

What God Hath Said on the Second Coming of Christ and the End of the Present Age: Part 1

There is a vast difference between taking up the Word of God, to hear what HE hath said, and taking it up to search out passages that seem to uphold any theory that one may hold. Now, all true Christians must feel that the second coming of our Lord Jesus Christ is one of the most interesting subjects that can occupy our thoughts.
It is proposed then in this article to look carefully at what God has said.
First, at what God hath spoken by His Son, in the four gospels; and secondly at what God hath spoken by His spirit, in the epistles of the apostles.
Before turning to the words of the Son of God, let us read, as introduction, the words of Gabriel, sent of God: “Fear not, Mary: for thou hast found favor with God. And, behold, thou shalt conceive in thy womb, and bring forth a son, and shalt call His name JESUS. He shall be great, and shall be called the Son of the Highest: and the Lord God shall give unto Him the throne of His father David: and He shall reign over the house of Jacob forever; and of His kingdom there shall be no end.”(Luke 1:30-33) Surely every “shall” in this wondrous passage, must be as really and as literally fulfilled as was the birth of Jesus. God said He should be born: it came to pass. God says He shall reign over the house of Jacob: it will surely come to pass.
Let us now turn and hear what God hath spoken to us by His Son, in the four gospels. I would notice the seven parables in Matthew 13. The present period was then unknown, and Jesus only spake of it in parables. But we who now have the teaching of the spirit of God, and know the mysteries of the kingdom of God, can read them more like histories than parables.
The first is the parable of the sower. Instead of God setting up that blessed reign of Christ, foretold in all the prophets, there is this time of sowing or preaching the Word. And how searching the words of Jesus; only one part out of four, even of those who professed to receive the Word, are saved and bring forth fruit. “Some fell upon stony places,” “some fell by the wayside,” “and some fell among thorns.” O! reader, beware lest thou art one of these classes, and thou perish forever. O! beware of the care of this world, and the deceitfulness of riches.
But some will ask, does the Lord Jesus teach that this state of things will continue; or does He teach that, by and by, all will receive His Word and be saved? Let the second parable answer that question. A man sowed good seed in his field, an enemy sowed tares. He explains it Himself: “He that soweth good seed is the Son of Man; the field is the world; the good seed are the children of the kingdom; but the tares are the children of the wicked one; the enemy that soweth them is the devil; the harvest is the end of the world (or age); and the reapers are the angels.”(Matt. 13:37-39) So that you see, my reader, plainly, whoever may preach the conversion of the whole world, Jesus taught the very opposite, and that in the world, the wicked and the righteous would grow together, until the very end of this age. The wicked should then be “cast into a furnace of fire; there shall be wailing and gnashing of teeth. Then shall the righteous shine forth as the sun in the kingdom of their Father.”
The third parable, the mustard-tree, teaches, that when the professing body should have greatly increased, the wicked spirits who tried at first to pick up the seed would lodge in its branches. Judas was one of the first of these birds; but now their name is legion.
The fourth parable, the hid leaven. “The kingdom of heaven is like unto leaven, which a woman took and hid in three measures of meal, till the whole was leavened.” (Matt. 13:33) This, perhaps, gives the saddest picture of all; but not more sad than true. So far from the church converting the world, the whole of professing Christendom has become leavened with the working in secret of this woman’s leaven of iniquity. Leaven in Scripture always denotes evil: the leaven of the Pharisees – leaven of Herod-leaven of malice and wickedness.
The fifth, sixth, and seventh parables teach the same truth. It is not the whole field, but the treasure in it: not the whole world, but the one pearl – the one church of God, that is being taken out of the world. All are not converted, but in the great net of the present dispensation of time, there are good and bad. “So shall it be at the end of the age.” The wicked shall then be severed from among the just.
(To be Continued).

Today

Lord, for tomorrow and its needs
I do not pray,
Keep me, my God, from stain of sin,
Just for today.
Let me both diligently work
And duly pray,
Let me be kind in word and deed,
Just for today.
Let me be slow to do my will,
Prompt to obey;
Help me to sacrifice myself,
Just for today.
Let me no wrong or idle word
Unthinking say;
Set Thou a seal upon my lips,
Just for today.
Let me in season, Lord, be grave,
In season gay;
Let me be faithful to Thy grace,
Just for today.
Lord, for tomorrow and its needs
I do not pray;
But keep me, guide me, love me, Lord,
Just for today.

Correspondence: Meaning of Ecc. 11:2

Question 89: Please give the meaning of Ecclesiastes 11:2. E. M. S.
Answer: Ecclesiastes is wisdom under the sun. Ecclesiastes 11:1-6 evidently looks at and exhorts us to use opportunities without regard to opposing elements. “Casting the bread on the waters,” “Giving the portion” and “Sowing the seed” as opportunity affords, leaving results with God.
This suggests for us service to the Lord, sharing with others what we enjoy; giving a portion to all we can reach; sowing the seed evening and morning, as we find opportunity. May our walk and ways, as well as our words, be “holding forth the word of life” (2 Thess. 2:17).

Inspiration of the Scriptures: The Bible - its Unity, Part 3

The unity of the Bible is further shown by the one leading subject throughout, being Christ Jesus, the Son of God, full of brightness and blessing; while the history of man, side by side, tells in all ages the sad tale of his antagonism to God, and unbelief in His goodness and mercy.
Let us look at the dark background of this divinely drawn picture of man’s ways, though created in the image of God, whose delights were with the sons of men.
1. Man created upright, and in innocence, listened to the lie of Satan, instead of standing firm by the Word of the Lord God; he therefore sinned, and thus death came into the world, and death passed upon all men, for that all have sinned (Gen. 3).
2. From fallen Adam to Moses, man though now having a conscience, and knowing good and evil, showed increasingly his departure from God till he actually became a god-maker. (Rom. 1:20-23).
3. From Moses to Christ, Israel, though in foolish self-reliance and ignorance, promised to keep the law, yet had they the advantage of God’s immediate dealings and care, a religious ritual, priesthood, and prophets; but they became such abominable idolaters, and worse than the heathen, that God had to give them unto captivity; and those who returned from the Babylonish captivity, Judah and Benjamin, when their Messiah came received Him not, and openly preferred a known murderer, Barabbas, to Christ.
4. From Christ’s death, resurrection, and the preaching of the gospel in the power of the Holy Spirit come down from heaven, sinners are called and saved for eternal glory by Christ Jesus. The effect still is that “few” comparatively believe, and the “many” are going on the broad road to destruction. When the Lord comes to receive His saints, all who have believed through grace, will be caught up to meet Him in the air, and be taken to the Father’s house. But when He comes out of heaven with His saints, He tells us that the world will be as it was in the days of Noah, ripe for judgment, and meriting everlasting destruction (1 Cor. 15:23; Matt. 24:37-39; 2 Thess. 1:7-9).
5. Christ reigns, and His saints reign with Him. He will “reign in righteousness,” therefore He must judge first the living then put down all rule, all authority, and power, and finally at the close of the thousand years, judge the dead, small and great. The effect of our Lord’s personal reign as King of Israel, and King over all the earth, will be that the knowledge of the Lord will cover the earth “as the waters cover the sea”; but it will be knowledge instead of the new birth in many instances, and restraint from Christ’s personal reign, so that they will yield “feigned obedience” (Psa. 66:3, margin); the consequence will be, that when Satan, who has been bound during the millennium, is let loose again, myriads will fall away, and fire come down from heaven in judgment upon them (Rev. 20:8-9).
6. This is followed by “a new heaven and a new earth,” in which righteousness dwells. The works of the devil having been destroyed, sin and iniquity completely taken away, and all things made new, righteousness now abides. Before law men were “filled with all unrighteousness”; under law righteousness was demanded in the way of works; by the gospel righteousness is reckoned without works to everyone that believes on Jesus; during Christ’s reign He reigns in righteousness; in the eternal state righteousness dwells in heaven and in earth.
But before leaving this dark side of the picture, let us never forget than man has utterly failed in every trial to which he has been subjected, and will do so, more or less, till the new creation order of things is fully established by Him, who said, “Behold I make all things new.” Man in innocence, surrounded with every possible privilege and blessing, sinned. The Flood having long after this taken all away in judgment except eight souls, because all flesh had corrupted God’s way on the earth, the chief of the spared eight becomes drunk, and through it lasting shame was brought on some of his descendants. Abram called out to trust in God, at the first trial of faith so fails, that he goes down into Egypt for help. The children of Israel, so confident of their own ability, no sooner promised to keep the law, saying, “All the words which Jehovah hath said will we do,” than they made an idol of gold, and danced around it, saying, “These be thy gods, O Israel, which brought thee up out of the land of Egypt.” When priesthood after this is set up, perhaps the first thing they did was to offer strange fire, which God commanded them not, and there they died before Jehovah. Moses, the man who was specially noted for meekness, could not enter the land because of, his rashness in smiting the rock, and calling God’s people rebels. The sons of Israel, long after this, wished to have a king, but he soon lost his place because he acted as he thought best, that is, as a rationalist, instead of in obedience to God’s Word (1 Sam. 15:1-23). David, a man after God’s own heart, fell into grievous sin immediately all his enemies had been subdued. After all this, God sent prophets to His people, and the people persecuted, stoned, or slew them. Jehovah sent John the Baptist to the Jews, and first imprisonment, and then death was his portion. Our Lord came with grace and truth, going about doing good, delivering all that were oppressed, and saving all that came to Him; but they said, “This is the heir; come, let us kill Him, that the inheritance may be ours.” At last they cry, “Away with Him, crucify Him.” The Holy Spirit came down after Christ’s ascension, by whose power the apostles and others preached the gospel of the grace of God, and from that time to this many resist and few believe. Such is man, and, unless born of God, such he will be, for “they that are in the flesh cannot please God.” Hence Stephen, in his famous speech before the Sanhedrim, said they were “stiff-necked and uncircumcised in heart and ears,” and declared them guilty of not having kept the law which they had received, of having persecuted and slain the prophets, of having been the betrayers and murderers of the Just One, and of always resisting the Holy Spirit. And in the coming age, as we have seen, with Satan bound, creation delivered, Israel blessed and enjoying their own land beyond all description, the church in manifested glory over it, Christ Himself ruling and reigning, all persons and things in subjection to Him, even then, when Satan is let loose for a little season, myriads will fall away, and openly dishonor God. Well has it been said, “What is man that Thou art mindful of him?
(Continued and to be continued).

The Happy Soldier

Some years ago it was my privilege to work as district visitor in London. In a cellar in one of the courts assigned me lived a pious old soldier, who had lost one of his legs in battle. This, however, did not afflict him, no, nor his poverty; but his wife was ungodly, and this lay as a heavy burden on his heart. He had a trifling pension, which, with the scanty product of a mangle, was all they possessed. The simple faith and piety of the old soldier at once won my heart. I often visited him, and was refreshed and edified by his remarks while reading the Word of God to him.
One morning the mail brought me a letter from a friend to whom I had written about this aged couple. She had been interested with their history, and sent me five shillings in stamps, to be laid out for them as I might judge best. I set out at once to carry them the good news. In vain, however, did I stand at the top of the dark stairs this morning, and call aloud to Mrs. G. to open the door, that I might find my way down. It was of no use, she was scolding aloud, and was deaf to every other sound. I groped my way, and, making for the door, gave a loud rap, which soon brought Mrs. G.’s voice to a momentary hush, and an expression of regret that she had not heard me. I replied, that I was greatly surprised and troubled to find her scolding so loudly.
“It is enough to provoke a saint,” she said, “to see him go on as he does.”
“O! don’t trouble the lady with these things,” said her husband; “let’s have some of the words of God, for truly we need them this morning.”
Mrs. G., however, was not to be so silenced; she would give vent to the anger that swelled her breast. I will relate her grievance in her own words.
“Now here’s a man for you, ma’am! without a bit of care for his wife! The other day we had only one penny in the house, and I sent him to get in a bit of bread; but, instead of that, he goes and gives it away to a tramp he knows nothing of!”
The old soldier looked deeply grieved. “My dear lady,” he said, “there are two ways of telling every story!” and then, with much emotion, he gave me his own version. It was very true! The penny was all they had, and he was proceeding to the baker’s, when a traveling man with his wife and three children sitting on a door step arrested his attention. He found that, like himself, they were natives of Scotland, sick and hungry; he spoke to them words of consolation from the Bible, and found to his joy that they were fellow-believers in the Lord Jesus. On parting he slipped his penny with a thankful heart into the hand of his afflicted brother. It was not until he had done so that he remembered, with dark forebodings, “What will wife say?”
Here Mrs. G. interrupted him with an exclamation, that “he must be a pretty husband who would rob his wife to give to a stranger.”
“Let me finish,” said he, “and you shall see, ma’am, how the Lord returned that little offering more than ten-fold.” He then went on to relate that, not daring to go back empty-handed, he walked up and down, asking the Lord to supply his need, not for himself, for he was now no more hungry, but for his wife’s sake. While walking to and fro, a gentleman inquired of him the way to the post office; the soldier offered to show him the way, and while walking together the gentleman entered into conversation with him, and asked if he was not old G-, whom he had known years ago? G-replied that he was, upon which the gentleman put a shilling into his hand, and bade him God speed.
“Now,” added the old Christian, “is not our Master true to His Word, and does He not bless a hundredfold all we do for His sake?”
I was deeply touched by this narrative, and felt solemnly impressed with the fact of God’s individual providence, and with the wondrous links in that great chain of life, which reveal to those who look for them the unceasing care and love of Jesus for His people. I recalled to mind, also, the letter I had received this morning; so I inquired what was their present trouble.
Here Mrs. G. once more broke forth in complaints. The landlady had demanded their rent by twelve o’clock that day, as she had a payment to make up. They had but a few halfpence in the house, and the old woman was for hastening off her husband with some things from the mangle, which would bring them sixpence more. “But I could not get him to go!” exclaimed she; “he said he must first ask the Lord; so instead of doing as I bid him, there he has been sitting over the Bible, and as if he had not lost time enough already, he must needs go down on his knees, and all my shaking and scolding him could not get him up till just before you came! and now it’s within half-an-hour of twelve.”
Old G., I should have observed, was standing with his stick and hat in hand, and a bundle under his arm when I came in, as if ready to go out.
“How much do you owe?” I inquired.
“Just five shillings,” replied she. “It’s fifteen pence a-week, as you know, ma’am, and it is just four weeks last Saturday.”
I said nothing, but opened the letter. I read to her that portion which related to her husband, then gave him the five shillings’ worth of stamps.
It was a moment never to be forgotten. The old man stood speechless with joy, with his beaming eyes lifted up in sweet thankfulness to his heavenly Father, while Mrs. G. sank down upon a chair, and, covering her face with her hands, wept tears of shame and sorrow.
“May God forgive me!” said she, “I am a wicked woman. Yes, I see it all now. I didn’t believe it, but it’s just as G. read it out of that very Bible not half-an-hour back, ‘Before they call I will answer.’ O! I didn’t believe it – I didn’t believe it! May God forgive me!”
God’s love had at last melted her stubborn heart, and the overpowering sense of the fact, “Thou, God seest me,” made her tremble with fear for her unbelief.
From this time a brighter day began to dawn upon old G.’s night of sorrow. His wife, so long the hinderer of his peace, and the object of his agonizing intercession, would now often sit by his side when he read the Bible, which had become more needful to him than his daily bread.

Truths for Young Christians: Restoration

In Christian life we must distinguish between two things that differ: daily defilement and positive backsliding; from the one we need cleansing, for the other we need restoration.
Touching a dead body unawares brought defilement to the Israelite under the law, and so any contact of spirit with this world and evil is defiling. There is no excuse, however, for it, for we are called to walk in spirit above it all, and if we get defiled by inadventure or carelessness, we have only ourselves to blame. Still these constant defilements, practically almost inseparable from our walk in this world of sin, are quite distinct from a gradual departure, first in our heart, but after in our walk, from the living God. We have already dwelt on the steps of failure. It is our happier task now to describe the return of a soul to full joy and communion.
In the first place, we must observe that daily defilements contracted, and not cleansed, are a bar to fellowship with Christ, and thus tend to lead us astray altogether. No soul that is in communion with Christ strays away. Communion (maintained by the Word and prayer) must cease ere backsliding begins. And it is because many of us are content to go on for so long, without enjoying real communion in our souls with Christ, that we are in such danger of backsliding. Restoration, therefore, means restoring communion.
The daily defilement, in fact any contact with evil, is met by the constant washing of the Word carried on by Christ, who as a servant forever (Ex. 21:6), loving his wife (believers collectively, Eph. 5), and his children (believers individually, Heb. 2), cleanses us by the washing of water by the Word, as shown in the lovely scene in John 13.
A special provision, however, exists for the restoration of a believer, after having fallen into sin, which is set forth by the type of the red heifer in Numbers 19. The essence of this type is that it presents the application of the cleansing power of the Word, in special connection with the death of Christ (typified by the ashes of the heifer in the running, or living, water); in whose death, on reference to Numbers 19:6, we also find that all that is of this world, from the highest to the lowest (cedar and hyssop, see 1 Kings 4:33), as well as all its glory (the scarlet) has been consumed. Thus our heart is reminded not only of the wondrous love of Christ in dying, but of the separating power of this death, which we in our sins had forgotten. (The water is called the “water of separation”). We find that these ashes were sprinkled twice, on the third day and on the seventh day; the first doubtless showing the convicting power of the Word on the conscience, leading to true confession, and the other bringing the full sense to the heart, of the love that has put all our sin away.
In the history of the Apostle Peter, we get the first of these sprinklings, when the Lord turned and looked on his erring disciple. That look broke Peter’s heart; it brought all the enormity and heartlessness of his sin to mind! and he went out and wept bitterly. The Lord, however did not cease His work of restoration at this point. When He rose from the dead on the third day, His first thought was for poor, erring, broken-hearted Peter. “He appeared first of all to Simon Peter.” Then, in that secret interview of which we have no details, the Lord completed the work of grace He had begun. One thing which we may learn from the secrecy of this interview, is that restoration is a secret work between the soul and Christ. Another point of great practical interest is to remember that although there may have been years of backsliding, there is no need for years of restoration. There is no such a thing as gradual restoration to Christ. When once His love has melted and conquered the straying heart, all the coldness of years is gone in a moment. And what a moment for the soul! When once more the long silent voice of our Beloved is heard speaking to our hearts again, the same yesterday, today and forever, and then at last we know the deep meaning of those words, “He restoreth my soul.”
The active work of the Lord in our soul, however, is not the first work of His grace towards us.
We must remember that from Revelation 12:10, and from other scriptures, it appears that in some way Satan has access at any rate to the outer courts of God’s presence, and his hateful business is to accuse us to God day and night. But we have one there ready to answer every charge on our behalf on the ground of His accomplished redemption. Hence it is written, “If any man sin we have an Advocate with the Father, Jesus Christ, the righteous.” This advocacy of Christ may long precede His work of restoration which leads us to confession according to 1 John 1:9. The one is what He does for us in heaven, the other what He works in us down here, leading us to true self-judgment. A soul truly restored has a deep sense of the love of Christ, just as we get a far greater view of the death of Christ in the red heifer than we have in the Paschal Lamb. Two things always accompany restoration, a deeper horror of sin and all that hinders communion with Christ, and a deeper sense of His changeless love.
It may be that now, towards the close of another year of grace, the eye of someone is reading these lines who has strayed from Christ. The heart has become cold, hard, and apparently dead; and yet it is not really dead, for some feelings have passed through it even while reading these few lines. And still, as the gradual steps of backsliding are thought over, the distance that separates us from Christ seem so vast that return seems impossible. Do not, beloved reader, rise with this hopeless thought. You may be fully restored at this moment. All that is needed is that you should turn from your sin to Christ, and fully own and confess to Him all that you have done. He will not keep you waiting years, or months, or weeks, or even days. Let there be true, full confession, and He is faithful and just now to forgive you, and to cleanse you from all unrighteousness.

Absolvo Te

One Priest alone can pardon me.
Or hid me “Go in peace,”
Can breathe that word “Absolvo te,”
And make these heart-throbs cease;
My soul has heard His priestly voice
It said, “I bore thy sins, Rejoice!”
He showed the spear-mark in His side,
The nail print on His palm:
Said, “Look on Me, the Crucified;
Why tremble thus? Be calm!
All power is Mine – I set thee free –
Be not afraid – ‘Absolvo te.’”
By Him my soul is purified,
Once leprous and defiled;
Cleansed by the water and the blood,
God sees me as a child:
No priest can heal or cleanse but He;
No other say, “Absolvo te.”
He robed me in a priestly dress,
That I might incense bring
Of prayer, and praise, and righteousness,
To heaven’s Eternal King;
And when He gave this robe to me,
He smiled and said, “Absolvo te.”
In heaven He stands before the throne,
The Great High Priest above,
“Melchisedec” – that one alone
Can sin’s dark stain remove;
To Him I look on bended knee,
And hear that sweet “Absolvo te.”
“A little while!” and He shall come
Forth from the inner shrine,
To call His pardoned brethren home;
O bliss supreme, divine!
When every blood-bought child shall see
The Priest who said. “Absolvo te.”

Scripture Study: Matthew 19

Matthew 19.
Matthew 19:1-2. The Lord is in Judea again, continuing His gracious work upon the multitude that followed Him.
Matthew 19:3-9. He teaches here that what is instituted for man, Judaism cannot put aside, and Christianity must uphold it. The Pharisees tempting Him, say, “Is it lawful for a man to put away his wife for every cause?” The Lord leads them back to God’s original institution, That abides, one man and one woman were to unite in marriage; they are one in the sight of God. “What God hath joined together, let not man put asunder.” “Moses, because of the hardness of your hearts, suffered you to put away your wives; but from the beginning it was not so. And I say unto you, whosoever shall put away his wife, except it be for fornication, and shall marry another, committeth adultery; and whoso marrieth her that is put away, committeth adultery.” It is man’s lust that sets aside God’s institution, “Marriage is honorable in all, and the bed undefiled; but whoremongers and adulterers God will judge” (Heb. 13:4). The Lord’s authority “I SAY unto you,” establishes this. The only case where separation is allowed, is where the bond has been already broken by sin.
The converted Gentiles had to be taught to respect this tie as a necessary thing (Acts 15:20, 29; 1 Thess. 4:3-7). Christianity, by Christ’s authority, puts marriage in its place as a divine institution for man.
Matthew 19:10-12. Here another principle is brought in. Some were not able to receive what He said, only those to whom it was given. Some were unfitted for marriage naturally, some were unfitted by men, but there were others, such as the Apostle Paul, who, by the power of the Holy Spirit, and the gift of God, gave himself entirely up to the Lord’s service (1 Cor. 7:7,37). If a man in the power of the Holy Spirit has the gift from the Lord to do this, and be entirely heavenly, so much the better, yet it would be evil to condemn or speak against the relationship which God originally instituted (1 Cor. 7:7).
Matthew 19:13-15. The Lord shows His affection for the children. It is human nature again in its simple, lonely, confiding character before being outwardly defiled by practice of sin. In Matthew 18, the child illustrated the spirit suited to the kingdom. How encouraging to parents to see the Lord receiving the children; laying His hands on them, and rebuking His disciples who rebuked those who brought them, by saying. “Suffer little children, and forbid them not, to come unto Me: for of such is the kingdom of heaven.” He knew the value of a child. He laid His hands on them. What encouragement to parents. What will we ask of Him for our children?
“I ask no worldly portion for my child;
I seek not of Thee gold, or rank, or fame;
Save it from sin; from human passions wild;
An interest in Thy love is all I claim.
Lord, make it Thine – Thine only – is my plea,
To love, and serve, and live alone for Thee.”
J. G. Deck
Matthew 19:16-22. In the young man we see one in nature outwardly good and amiable, but the evil in him leads him to turn his back on the Lord and go away from Him. He shows desire to learn the right way, and he admires the Lord’s goodness, and he wanted to have eternal life. He had not learned himself. He says, “Good Master, what good thing shall I do, that I may have eternal life?” He thought he was good, and could do good. The Lord answered, “Why callest thou Me good? there is none good but one, that is, God; but if thou wilt enter into life, keep the commandments.” Notice the Lord leaves “eternal” out, for the law was for life on earth, for living by, not to gain heaven by. But the young man thinks he can and had kept the commandments from his youth up, and asks further, “What lack I yet?” Jesus said unto him, “If thou wilt be perfect, go and sell that thou hast, and give to the poor, and thou shalt have treasure in heaven, and come and follow Me.” Will he do it? Alas! possessions are his god, the true God is shut out. The Lord exposed his covetous heart, and he went away sorrowful, for he had great possessions. “Thou shalt have no other gods before Me” (Ex. 20:3). “The rich He sends empty away.” Irreproachable in his behavior, the Lord tries his heart; he loves earthly things, and God is left out, he clings to his possessions. What he desired to possess is not esteemed. Man at his best estate is altogether vanity, his heart is away from God. Possessions, which in the Jew’s mind, were a mark of divine favor, are seen here holding the heart away from God, and spiritual blessing is not esteemed.
Matthew 19:23-24. “Then said Jesus unto His disciples, verily I say unto you, that a rich man shall hardly enter into the kingdom of heaven. And again I say unto you, it is easier for a camel to go through the eye of a needle, than for a rich man to enter into the kingdom of God.” What a complete change to the disciples’ minds this was. “Who, then can be saved?” Impossible with men, but with God all things are possible: The rich cannot save himself, but God can reduce him to nothing but a sinner in his own eyes, and then will save him (1 Sam. 2:6-8; Luke 1:53). The Lord knows how to humble the rich spirit, and to make it poor before Him. He knows how to make a Saul of Tarsus, to feel that his righteousness is filthy rags, and that he is the very chief of sinners. The disciples had to learn that all of self and earth was worthless for salvation in the sight of God, but that God could find the way of salvation for the humbled soul.
Matthew 19:27-29. “Then answered Peter, and said unto Him, Behold, we have forsaken all, and followed Thee! what shall we have therefore?” and Jesus said unto them, Verily I say unto you, that ye which have followed Me, in the regeneration when the Son of Man shall sit in the throne of His glory, ye also shall sit upon twelve thrones, judging the twelve tribes of Israel. And every one that hath forsaken houses, or brethren, or sisters, or father, or mother, or wife, or children, or lands, for My name’s sake, shall receive an hundred fold, and shall inherit everlasting life.” The regeneration means, the renewed world, what we speak of as the millennium or reign of Christ. The disciples are to have the center places in the kingdom to reign over and judge the twelve tribes of Israel. And everyone who had left all that by nature they loved, for Christ’s name, would have an hundred fold (heavenly comforts for the soul, but with persecutions, see Mark) in this life. What we lose now for Christ, is heavenly gain now in realizing our possessions in Christ. We realize now what eternal life is, and shall enter into its fullness at the end.
Matthew 19:30. “But many that are first shall be last; and the last shall be first.” This is the Lord’s sovereignty in calling and rewarding His servants. Matthew 20:1-16 continues it.

Nehemiah: The Building of the Wall, Part 2

In Nehemiah 3, the wall is being built. Each little company is in its place building the wall. Is not this a striking picture of what has taken place in these last days? Wherever the truth of the one body of Christ – the one church of God – has been accepted in the fear of the Lord, each little company has acted upon it, in building the wall of separation; and the divine Architect has made each piece fit, like the well-worked courses of masonry. The work is of God; His good hand is with the feeble remnant.
It may be called “exclusive” – it must be so. You cannot build a wall, but it must be an exclusive wall. Why set up its doors and bars, if not to preserve and exclude? We cannot sincerely receive the blessed truth of the one body, but this must exclude all sectarianism. Can we accept the truth of one God, and then tolerate the other gods of the heathen? No more can we accept the truth of the one body of Christ, and accept the many bodies of Christians!
This brings us to the third form of opposition, “But it came to pass, that when Sanballat heard that we builded the wall, he was wroth, and took great indignation, and mocked the Jews.” (Neh. 4:1) Thus we have had grief, then laughter – now the building of the wall is a fact, there is wrath. Is not this picture also sadly fulfilled before our very eyes? Sanballat’s wrath against the builders of this wall was not more bitter, than the bitter hatred against the sacred enclosure of souls being really gathered to Christ, the true center. What! say they, not tolerate our denominations? What! exclude all that does not seek uncompromising conformity to Christ?
Sanballat spake to his brethren and the army of Samaria, “What do these feeble Jews?” And indeed what were they, compared with the army of Samaria? “What do these feeble Christians?” Ah, indeed, what are they, compared to the armies of Christendom around? Are they going to remove the heaps of rubbish? Are they going to level sectarianism in a day?
This wrath is succeeded by the fourth form of opposition. Sanballat mocked the Jews. “Now Tobiah the Ammonite was by him, and he said, Even that which they build, if a fox go up, he shall even break down their stone wall” (Neh. 4:3). Thus the enemy, whilst hating with bitter hatred the work of God, outwardly appears to make light of and mock at it. Is not this exactly so in our day? Well, there may be grief, laughter, wrath, and mocking; but the work goes on. The wall is growing fast – piece is joined to piece. The work of God spreads. Souls are hearing the voice of the Shepherd, and leaving every fold of man – are being gathered within the sacred enclosure, around the precious person of the Great Shepherd, Christ Himself. Christ is exalted, and all that does not exalt Him is excluded. Man is nothing.
Now, what will Sanballat and his company do? This brings us to the fifth form of opposition. “But it came to pass, when Sanballat, and Tobiah, and the Arabians, and the Ammonites, and the Ashdodites, heard that the walls of Jerusalem were made up, and that the breaches began to be stopped, then they were very wroth, and conspired all of them together to come and to fight against Jerusalem, and to hinder it” (Neh. 4:7-8). We have had grief, laughter, wrath, mocking; now there is to be fighting – determined, uncompromising opposition to the work of God.
Is it not even so? Has not every sect in Christendom agreed in this, to fight against, to oppose, the building any further of the wall of separation to Christ? And as these companies consulted to come upon the Jews unawares, so often, when God has been blessing His Word in a given place, has the enemy come unawares, sowing evil reports, and sought to stop the work. Behind the scene are wicked spirits in the heavenlies. Surely, we need the whole armor of God, and “our God shall fight for us.” The work at Jerusalem still went on; so it is now; the more opposition, the more it drives to God, and the more the work goes on. The trumpet of truth is heard to give a certain sound, and the saints resort thither unto it.
(To be continued).

What God Hath Said on the Second Coming of Christ and the End of the Present Age: Part 2

Matthew 24. In this chapter we find the plain teaching of Jesus to His little flock of Jewish disciples. It is the same subject as the seven parables, only in plain words, not in parables. In the first few verses, Jesus foretells the destruction of the Jewish temple – which, we all know, came literally to pass. He was seated on the Mount of Olives – the very spot where His feet shall stand when He comes to reign. The disciples came and inquired privately, “Tell us, when shall these things be, and WHAT SHALL BE THE SIGN OF THY COMING, and of the end of the world (or age)?” Now read carefully Matthew 24:4-51, and mark, there is not one thought of the world’s conversion. He foretells there will be false Christs – deceivers – wars, and rumors of wars – famines, pestilences, earthquakes – persecutions, sorrows, death – iniquity abounding, and the love of many waxing cold. And instead of the world receiving the gospel, it is preached for a witness; and then the end comes. Much has been fulfilled; and much in this chapter has yet to be fulfilled. Bear in mind, that all these words of Jesus were spoken to His Jewish disciples, and have special reference to what shall befall that nation. In Matthew 24:15 He says to them, “When ye, therefore, shall see the abomination of desolation spoken of by Daniel the prophet, stand in the holy place (whoso readeth, let him understand) then let them which be in Judea flee to the mountains.” This evidently proves that the temple of Jerusalem has to be rebuilt; for the abomination of desolation is to stand in the holy place. And, if you read Daniel 12:11-13, and compare it with Daniel 9:27, you see the fearful act of the head of the Roman power, who causes the sacrifice and oblation to cease in the midst of the week – then mark when the abomination stands in the holy place.
Then the words of Jesus will be most valuable to the godly Jews, who believe His testimony in that day. The moment they see this take place, they take it as the signal to escape. They have not even time to go into the house to fetch their clothes. Then takes place “the great tribulation, such as was not since the beginning of the world to this time – no, nor ever shall be.” The angel, speaking to Daniel of these days, says, “And there shall be a time of trouble, such as never was since there was a nation, even to that same time: and at that time thy people (that is, the Jews) shall be delivered, every one that shall be found written in the book. And many of them that sleep in the dust of the earth shall awake, some to everlasting life, and some to shame and everlasting contempt” (Dan. 12:1-2). Clearly, then, the setting up of the abomination of desolation, and the time of tribulation, are both future. And in proof of this, our blessed Lord says, what will take place immediately after, the tribulation: “And then shall appear the sign of the Son of Man in heaven: and then shall all the tribes of the earth mourn, and they shall see the Son of Man coming in the clouds of heaven with power and great glory” (Matt. 24:29-30).
Most certainly, then, all tribes of the earth are not converted; or why do they mourn when Jesus comes? Jesus says, “Verily I say unto you, This generation shall not pass till all these things be fulfilled.” How remarkably this is so before our very eyes! Though scattered among all nations, the generation or race of the Jews still exists, and waits the fulfillment of all these things. The Lord then goes on to state, “As the days of Noah were, so shall the coming of the Son of Man be.” O, what a solemn thought, that this world is to go on eating and drinking – rejecting Christ, just as the world despised, the preaching of Noah, “and knew not until the flood came and took them all away; so shall also the coming of the Son of Man be!” The most solemn warnings to watch and be ready are then given “for in such an hour as ye think not the Son of man cometh.” Terrible will be the doom of that servant who SHALL SAY IN HIS HEART, MY LORD DELAYETH HIS COMING. Yes, he calls Him, “My Lord”; but his portion will be with the hypocrites, “where there is weeping and wailing, and gnashing of teeth.” Such are the solemn words of the Son of God. O! that my reader may be awakened to the solemn inquiry, Am I ready, and waiting for the Lord?
Matthew 25. The whole of this chapter also is upon the same subject. The illustration of the ten virgins teach most plainly, that instead of all being converted when He comes, some of those who profess to be His are shut out. All slumbered and slept. O, professor! if you should hear those words, “I know you not”; “Watch, therefore, for ye know not the day nor the hour.”
Another illustration is then given of this period, during which Jesus is away in heaven: “As a man traveling into a far country.” And again, the whole of His servants do not improve their talents. Then “When the Son of Man shall come in His glory, and all the holy angels with Him, then shall He sit on the throne of His glory.” Then to the end of the chapter the Lord most plainly describes the judgment of the living nations at His coming. You will notice, if you read carefully, that there is nothing said in this place about the judgment of the dead: that is quite a distinct event, as we shall find as we go on in the Word.
Even when standing before the high priest on the solemn night of His betrayal, Jesus said, “Hereafter shall ye see the Son of Man, sitting on the right hand of power, and coming in the clouds of heaven.” Alas, man always refused this testimony. The high priest declared it blasphemy and pronounced Him worthy of death. (Matt. 26:64-65).
I would now turn to Mark 13. The solemn warnings of Matthew 24 are repeated: “And then shall they SEE the Son of Man coming in the clouds, with great power and glory.” Not only the porter of the house is to watch, but to mark the uncertainty of the hour when Jesus shall come, ALL are to watch. Some will say, “Ah, you do not need to think about the coming of the Lord; it will not take place in your day.” Jesus says, “WATCH ye, therefore, for ye know not when the master of the house cometh;” at even, or at midnight, or at cockcrowing, or in the morning, lest coming suddenly, He find you sleeping. And what I say unto you, I say unto all, WATCH.”
O, how near, then, the Lord’s coming must be. The present night of His absence is thus divided into four parts: – “even,” “midnight,” “cockcrowing,” and “morning.” Where are we? The evening of the dispensation is already past; yea, the midnight of the dark ages is past, or middle ages, as they are called. The awakening of the Reformation is past. Ah, the morning breaks. Watch! watch! the day will surely break. O, blessed are they who shall be found ready!
(Continued)

Correspondence: Rewards; Day of the Lord; Cain's Wife

Question 90: Are rewards given at the judgment seat of Christ, or at the beginning of His reign? J. W. B.
Answer: Rewards are given to the heavenly saints at the judgment seat of Christ, where they are manifested. Approval or disapproval is there expressed, so that when they come forth with the Lord to claim the kingdom they are clothed with the righteousness of the saints, and their places in the kingdom are taken by them (Rev. 19:8; Matt. 19:28). We also find that rewards are given to others when He comes with His saints (Dan. 12:3; Rev. 22:12; see also Rev. 11:15-18).
Question 91: Does the “Day of the Lord” take in also His coming for the church? J. W. B.
Answer: “The Day of Christ” (Phil. 1:6,10; 2:16) is when Christ sees His work perfected in His church. It is when He has His joy in them, and Paul has his joy in his labors among them also (1 Thess. 2:19; 3:13).
“The Day of the Lord” (1 Thess. 5:2) is when the Lord comes to the world as a thief in the night to claim His rights and to put down all evil (Isa. 2:12; Joel 2:31; 3:16; Obad. 1:15).
“The Day of God” (2 Peter 3:12, 13) is when
“All taint of sin shall be removed,
“All evil done away;
“And we shall dwell with God’s beloved,
“Through God’s eternal day.”
Question 92: Where did Cain get his wife?
Were Adam and Eve the only man and woman created? A. B.
Answer: The theory that others besides Adam and Eve were created is against Scripture. Adam was the head of the race; his sin brought ruin on all his progeny – that is, everybody. Christ is the second man out of heaven, head of the redeemed race (Rom. 5:19; 1 Cor. 15:45-47).
The genealogies of Scripture give only the names of those needed to fill up the Word of God. It is not consecutive history. Cain’s generations (Gen. 4:16-22) were contemporaneous with Seth’s line (Gen. 5), the line of grace.
Adam was 130 years old before Seth was born and he lived 800 years more. “He begat sons and daughters;” we know nothing about their number.
There was nothing wrong in brothers and sisters marrying at this time. There was no law against it, and no other way. God arranged it for the settlement of man upon the earth.

Inspiration of the Scriptures: The Bible - its Unity, Part 4

The Bible – Its Unity
It is important though to observe the unity of thought pervading all Scripture as to man’s utter ruin, an incurably bad condition, calling for nothing less than being born anew. If, early in Scripture, we are told that “every imagination of the thoughts of his heart was only evil continually,” many hundreds of years after, another prophet declared, that “the heart is deceitful above all things, and desperately wicked”; while hundreds of years later, our Lord said, “from within, out of the heart of men, proceed evil thoughts,” and He did not name anything good in it. Later on an apostle declared, that “the carnal mind is enmity against God; for it is not subject to the law of God, neither indeed can be” (Rom. 8:7); so that as to man’s moral condition in God’s sight, ever since the fall, the testimony throughout has been, that “there is none righteous, no not one.” The giving of the law, instead of helping or improving those who accepted the yoke, only caused the offense to abound, and gave “the knowledge of sin.” Happy are they who so know the Lord Jesus Christ as their life and peace, as to be not under law, and vainly hoping to reach God by doings and efforts, but as brought to God in Christ and through His precious blood can bow in adoring praise and thanksgiving to Him for the accomplished work of eternal redemption. Such is the uniform testimony of God concerning man throughout the entire volume of inspiration.
The bright line which runs from Genesis to Revelation, and gives the whole book a unity which nothing else could, is its testimony to the infinite glory of the person of the Son, the eternal efficacy of His one sacrifice for sin, His moral worth and excellency beyond all thought, the glorious offices on our account He now sustains, as well as His glories yet to be revealed when He cometh with clouds, and His saints accompany Him, to put down all that is contrary to God, and fulfill all the promises to Israel, and much more, which assure us of the verity of our Savior’s words when speaking of the “Scriptures,” “they are they which testify of Me” (John 5:39).
It has been well said, that redemption was no after thought with God – that God’s thoughts and purposes of love were toward us before the foundation of the world. And, in sweet accordance with this, we find the first man, who “was a figure of Him that was to come,” with his loved and loving helpmeet by his side, on awaking from his deep sleep; saying, “This is now bone of my bones.”
But by one man sin entered into the world, and “by man came death”; but no sooner had these enemies been brought into God’s creation than we hear of a Redeemer, a suffering Redeemer, too, who should effectually render null all the power of that old serpent, the devil. This Abel believed, and therefore offered a firstling of the flock and of the fat thereof; and “Jehovah had respect unto Abel and to his offering.” Noah’s burnt-offerings of every clean beast and of every clean fowl, again in type set forth the sweet savor of the offering of Christ, and the blessings to man and the earth which flow from it to this day. Abram offering up Isaac, as we have seen, prefigures, as no other type does, that “the Father sent the Son, the Savior of the world”; that He would provide a lamb for a burnt offering. Thus we have in Isaac, bound on the altar and afterward loosed from it, a striking type of the death and resurrection of God’s only-begotten Son. And as the Holy Spirit makes no mention of Isaac after this, till he comes forth to meet, and embrace his beloved bride, so in the intermediate chapters we have the line of pilgrimage and present circumstances of the man of faith, as well as the father of the typical dead and risen one, calling out a bride for his loved and only son, the heir of all his possessions. God’s way of blessing having been set forth by the death and resurrection of Isaac. Sarah dies, type of the Jewish system being broken up, the man of faith is a pilgrim and stranger, can find nothing here to possess but a grave; and though in the world, not of it, not beholden to it or indebted to it for anything, he obtains a grave for which he pays full price. On the other hand the true sent one is calling out a bride for the dead, risen, and now hidden son by whose testimony she is separated in heart unto him, whom she has not seen but loves; and the first glance she has of him detaches her from everything here, and she hides herself under her veil. Absorbed with the object of her heart, self was lost sight of in the consummation of her longing desire to “see his face.”
And so we might trace in Joseph again the dead and risen Christ in relation to Israel. While in Exodus, Leviticus, and Numbers, as well as Deuteronomy, types and shadows abound in the various sacrifices offered of the infinite value and various aspects of that one offering of the body of Christ, offered once for all, never to be repeated; “for by one offering He hath perfected forever them that are sanctified” (Heb. 10:14). In so many ways, and by such various types, the sufferings and death of our adorable Savior are set before us in many parts of the sacred writings, that we can now go from the New Testament statements of facts to learn details in the records of the Old Testament types and prophecies.
Not only did God declare that our Redeemer should be the seed of the woman, the seed of Abraham, and the fruit of David’s loins, as concerning the flesh, but that He should be the virgin’s Child, and yet His name be Immanuel, God with us. It may be frequently noticed in Scripture, that when the Savior’s perfect Humanity is brought before us, His deity is also mentioned not far off. Again, we are told that Bethlehem would be the place of the Savior’s birth; and there it is added, “whose goings forth have been of old from everlasting,” words which can only apply to deity. Prophets had long before declared that He would grow up before Jehovah as a tender plant, be a Man of sorrows and acquainted with grief, be despised and rejected of men who were so blind that they saw no beauty in Him that they might desire Him, so that in astonishment the prophet could add, “Who hath believed our report, and to whom is the arm of the Lord revealed?” Not only His spotless and unblemished life, in suffering, temptation, and sorrow occupied the prophetic page, but the cross in all its unutterable woe was again and again set forth. If one spake of Him as wounded for our transgressions and bruised for our iniquities, and that it pleased Jehovah to bruise Him, and to put Him to grief as having our iniquities laid upon Him, another was able to foretell the details of those sorrows some hundreds of years before they occurred, and that His cry at that moment under desertion would be, “My God, My God, why has Thou forsaken Me?”
The actual death of the cross had been predicted as having His hands and His feet “pierced.” The derision and mockery and cruel scourging of men, and His heel under the bruising of Satan, were not forgotten. His betrayal for thirty pieces of silver, the scattering of the sheep when the Shepherd was smitten, the rejection of the “Stone” which was to be the Head of the corner were not omitted by the prophetic pen. That the soldiers should part His garments among them, and for His coat cast lots, that He should occupy in death a rich man’s grave, that He would be numbered with the transgressors, bear the sins of many, pray for the wicked murderers, and His soul be made an offering for sin, that a bone of Him should not be broken, but that they should look upon Him whom they “pierced,” we have only to look into the New Testament to find every jot and tittle of it literally fulfilled. That He died for our sins according to the Scriptures; was buried, and that He rose again the third day according to the Scriptures, was what Paul declared in the gospel that he preached (1 Cor. 15:3-4); and Peter also owning Him as “Lord of all,” and coming to judge the living and the dead, delighted to add, “to Him give all the prophets witness that through His name, whosoever believeth in Him shall receive remission of sins” (Acts 10:43). If in almost the beginning of the sacred Volume the blood of the firstling of the flock was shed, as the only way of sinful man approaching God, at nearly the end we find it is blessedly recorded, “Unto Him that loveth us, and hath washed us from our sins in His own blood, and hath made us kings and Priests unto God and His Father, to Him be glory and dominion forever and ever, Amen.” Is it possible, we may well ask, that anything can more clearly demonstrate the unity of the Bible than the discovery that the one grand absorbing and paramount subject throughout, is Jesus the Son of God who is Lord of all, and the Savior of sinners who believe?
(Continued and to be continued).

I Shall Be Satisfied

I shall be satisfied,
But not while here below,
Where every earthly cup of bliss
Is wisely mixed with woe.
When this frail form shall be
Forever laid aside,
When I shall in His likeness wake,
I shall be satisfied.

He Shall Be Satisfied

He shall be satisfied,
When all He died to win,
By loving kindness gently drawn,
Be safely gathered in.
When in the glory bright
He views His glorious bride,
Sees of the travail of His soul,
He shall be satisfied.

In Season and Out of Season

“To everything there is a season”; and the season for the gospel worker is this present moment; “the night cometh when no man can work.” Never again will he have a better opportunity for working for the Lord than now.
“Now is the accepted time, now is the day of salvation.”
Now is the period which may be brought to a close at any moment, for when the Master of the house is risen up, and hath shut the door, this “now,” this day of salvation will be over. Now the golden present, the opportunity which will never return. Therefore, let us heed the Scripture exhortation, “Be instant in season, and out of season,” remembering that in and out of season means always, everywhere. Nor is it only because the day is far spent and the night is at hand, that we need exhort one another to the work, for a deep desire after reality in divine things abounds in many hearts, and calls for the loving energy of every true servant of God.
The only ease infidelity offers is “a leap in the dark”; and the only balm formalism presents is an undefined shadow of a substance which may or may not be found when life is passed! The no-belief of the skeptic covers unrest of soul; the ceaseless effort of the formalist to work for rest, shows that the desired end is not attained. But the true Christian has rest, for he has Christ, and Christ dwelling in the heart by faith, fills the heart with life, light and liberty, and is the unanswerable witness to theories of darkness, and doctrines of unrest.
Young Christians, you have “the treasure” (2 Cor. 4:7); in you is the fountain of living waters (John 7:38); and you are set here upon the earth to bestow of the treasure, and to communicate of the living waters to others. Men, women, and children around you thirst; they are poor, they need Christ, and God has given you His salvation, not merely for your own blessing, but that you may be a blessing to others – even as He said to Abraham, “I will bless thee.... and thou shalt be a blessing.” And certain it is, the nearer a Christian dwells to the source of all blessing – God Himself – the fresher, the sweeter, the richer are the streams of blessing that, through the Spirit, flow out of his heart to the refreshment and blessing of others.
Let us then arise, and shake off the dust of the earth from our souls, and go forth as from Christ to the Christ-less world, with hearts, and hands laden with divine bounties to the unsaved and the unsatisfied. Be instant in season and out of season, for men always need Christ, and peace, and rest, and joy.

Truths for Young Christians: Story of a Second Conversion

I had been a Christian for about ten years when the second conversion of which this is the story. occurred, for I can as clearly speak of two conversions in my history, as I doubt not the Apostles Andrew and James could speak of two in theirs – one when they believed in Christ as Messiah, and the other when He became their sole object on earth, detaching them, moreover, by His express call, even from their daily occupation. Peter, indeed, could speak of a third conversion, expressly mentioned by the Lord as such: “When thou art converted, strengthen thy brethren” (Luke 22:32).
But while these instances show that such cases are by no means rare, it is well to remember that when God does save a man, expressly as a pattern to them “which should hereafter believe in Him to life everlasting,” he is so completely turned to God from everything at the outset, that we do not read of anything that could he regarded as a second conversion in the history of the Apostle Paul. It should, however, be clearly understood what is meant by conversion. The Greek word is generally translated “to turn,” or “turn about.” In Matthew 9:22, we read: “But Jesus turned Him about, and when He saw her.” In Matthew 12:44, we find: “I will return into my house.” In both cases the word is the same as that translated “conversion.”
Any definite turning to God is a conversion to God. People may turn to other things; we often hear of a conversion to politics, or to some school of medicine; with these, however, we need not concern ourselves now, as the only conversion I have to speak of is “to God.”
When first we believe, we are converted, as we read of the Thessalonians, “they turned to God from idols, to serve the living and true God.” But this may only be a turning from some one thing, as in their case, “from idols,” which might still leave room for a turning from the world, or a turning from self.
It must be plainly understood that conversion in no way always implies salvation, which can only be once, and is forever, whereas, a conversion (as we have seen) may occur several times in a Christian’s life.
This, indeed, I clearly apprehended in my own case, having been saved as I have said, ten years before the conversion of which I now write, and during the whole of that time being perfectly clear as to eternal life.
I had always enjoyed sound gospel teaching, and was also rightly taught what a Christian should be, although I had never grasped the power. I used to attend the meetings regularly on the Lord’s day and I helped also a little in working among the young.
I also went to one meeting in the week, feeling it was not right to do less. Under the plea, however, of necessary bodily exercise, I mixed largely with the world in sports and amusements. In traveling I desired to see as much of the world as possible – in short, although I recognized the claims of God over part of my time, I lived the rest to please myself.
Not that I ran to any excess; on the contrary, in every respect my life was very regular and steady. It is of principles I now speak, and my principle was to please myself as far as possible, without violating what my conscience told me God required.
Of course, I was not very happy. In this I am sure you, my reader, will bear me out if you are trying to trim with the world and draw the line, so as to include as much of it as possible.
The very fact of marking out a boundary between myself and the world, short of that formed by the cross and grave of Christ shows that we are not of one mind with God in the matter and are therefore out of communion with Him on this subject, and no soul can be happy where it is out of communion.
In this way, therefore, I went on drawing my own lines of demarcation, which included those pleasures that I most loved, while rigorously excluding those I did not so much care for, or had a bad conscience about, which lines were necessarily always shifting and uncertain, not being established by God’s Word; and as I had a good many Christian companions, each with lines of their own drawing, and which often did not coincide with mine, the question continually arose, “Is this right?” or “Is that wrong?”
But the Lord had a better course for me than always steering as near as possible to the rocks and shoals which I loved, although I knew the danger I ran if I struck upon them.
It was in the autumn I left home for a month’s pleasure tour. The program was a delightful one to me, and just suited my tastes, being mainly by water. One of my companions was a child of God, the other not.
At first everything went well, our plans prospered, the weather was fine, the scenery magnificent.
But after a few days, things suddenly took a turn. We suffered shipwreck, and after being nearly drowned, were compelled to give up the water and travel by land.
All this spoke to my conscience, which was by no means completely at its ease, for none of my boundary lines between what was wrong had really satisfied it; but of one thing I was glad; by going on foot instead of by water, I found we should be able to include in our route a town I had long wished to visit, in order to see a Christian who lived there, and of whose consistent and happy life I had very often heard. Fortunately, the town was in the midst of beautiful scenery, so I was enabled to change our route without disclosing what was in my heart.
After some unpleasant adventures, we reached the town about seven o’clock one evening. On leaving our hotel after dinner, I went to the meeting room and found that a lecture was being given by the very man I had come to see.
I cannot remember a word of that address. I had attended meetings and lectures from my infancy. God was now going to work in my soul by other means.
I was not unknown by name to the lecturer, and was, with my friends, invited to supper.
Somehow, I listened that night with wonderful interest to what I heard of the Lord’s work, in which he was so happily engaged; and as I looked at his face, I felt that he, at least, had found a source of pleasure in serving his Master to which I was still a stranger. What struck me, too, was not so much the work he spoke of as the manner in which it seemed to flow from a real love to Christ, so that even I felt it must be a very happy life to lead.
At a late hour we left, promising to breakfast with our new friend in the morning. We had to tell him something of our plans, although I must say I felt somewhat ashamed of laying my tour of pleasure by the side of his work for the Lord. I thought a good deal that night about the different paths he and I were pursuing, both children of God.
In the morning, after breakfast, he brought out a map and showed us our way to a neighboring lake of great beauty, which we had arranged to reach by boat, and he bid us good-bye; but with a sad look at me, that went to my heart, for I saw he pitied me.
I let my companions row while I steered. All at once, when about half way to the lake, the truth flashed upon me, and I saw I was the slave of self instead of being Christ’s free man, and His servant. I saw I was being dragged about, for its own pleasure, by the wretched self that God had condemned, and I felt it was not I. I had different tastes. I longed to serve Christ, and as the sense of His love to me, and His forbearance all the long years I had known Him, filled my heart, I felt I was in an intolerable bondage I would endure no longer.
I felt I had a right to be free. Christ had died to set me free, and yet here I was working like a galley slave to please myself. What made me see it so clearly was that I had just left a free man. He, at any rate, was not toiling at the old oar. He was under a new master, and was free from the tyranny of the old.
A slave will endure a great deal of bondage if he is not brought face to face with freedom; but if he is in the company of a free man, his soul must indeed be dead if he does not long to lose his fetters. My mind at any rate was made up. I would not endure it another day.
The time past of my life was indeed more than sufficient to have lived in the flesh, to have wrought my own will, and Christ having suffered for me in the flesh, I armed myself with the same mind, no longer to live in the flesh, after the desire of men, but according to the will of God. It was from myself I now turned (that I had served so faithfully), to Christ; from doing my own will to a desire to do His.
I sat in the boat with all this passing in my mind, and said nothing; but I prayed to the Lord to make this conversion a very real one, and to enable me from that day to do His will, and not my own.
At last I began to think how to get out of my tour, as I longed to spend some days where I was to see more of my friend, who, not by his words, but simply by the force of living for Him who died for him, had been the means of this, my second conversion, and the Lord opened the way in a remarkable manner. My unconverted companion began talking about the tour, and how tired he was of walking. I proposed to stay a few days where we were, while he paid a visit to some friends he had near, to which he assented.
By this time we reached the lake, but I must confess its beauties are almost forgotten in the remembrance of the beauty I saw in the path of Christ.
On our return, my companion went on his way, while we who were Christians went up to see my friend again. He was surprised at the sudden change in our plans, but on hearing we were staying in the town, insisted on our making his house our home.
This we did, and what I saw in his life fully confirmed me in my discovery, that to please one’s self is slavery, and that the only liberty and happiness for a Christian is to do the will of God.
This, then, is the simple story of my second conversion from the principle of serving self to serving Christ; for although in many respects I did the same things, by God’s grace it was in measure through the influence of a new principle, and it is this that is of all importance in God’s sight. What we do is of course a serious question, but why we do it is a far deeper one, both to God and ourselves.
I have hesitated for many years to record these experiences, feeling how feebly they presented the great truth of deliverance from self, and knowing how still more feebly I have carried it out. But seeing in Scripture how often a personal testimony is given, I look to the Lord, that He may use this narrative to the full deliverance of any of my readers who may still be seeking to serve two masters.
“For the love of Christ constraineth us; because we thus judge that if one died for all, then were all dead. And that He died for all, that they which live should not henceforth live unto themselves, but unto Him which died for them and rose again (2 Cor. 5:14-15).

Scripture Study: Matthew 20

Matthew 20:1-16. In this parable is shown the sovereignty of the Lord in calling and rewarding the laborers in His vineyard. It is not a question of obtaining salvation, for we do not labor for salvation. It is to teach us that He is our Master. Peter said, “What shall we have therefore?” Here is the answer even to those who begin late in the day, ‘‘Go ye also into the vineyard, and whatsoever is right I will give.” Laboring for Christ because we love Him, and trust Him, is better than laboring for reward. It is grace, not law. We love to do it because of His grace to us: He made us His.
Reward is encouragement to those who are serving Him with His approval. It is not the motive to lead us to serve. Those who bargained for the penny, received it; those who trusted and served Him, received what His heart delighted to give them. “What is right I will give you;” they trusted Him. The assurance of reward is given when the Lord is the motive for the labor. If we get the reward as the motive for laboring, it falsifies the character of true service for Him. Then Peter and the disciples must learn that it is His to do with as He pleases, and, therefore, many that are last shall be first, and the first last: for many be called, but few chosen. It is the sovereign grace of God that is the source of true blessing. Peter was called, and a place given him by the Lord. Paul came in later on, as last, but he was a chosen vessel to unfold the heavenly mystery of Christ and the church as a stronger testimony to grace, so the last was first, and the first last. Each has his place to fill as given by the Lord (Eph. 4:8). Again, notice, this is not salvation, but service after we are saved.
Matthew 20:17-23. On the way up to Jerusalem the Lord takes His disciples apart to tell them privately what is to befall Him in the very place where He should have been crowned king. It is the Son of Man, despised and rejected.
“Behold, we go up to Jerusalem, and the Son of Man shall be betrayed unto the chief priests and unto the scribes, and they shall condemn Him to death, and shall. deliver Him to the Gentiles to mock, and to scourge, and to crucify; and the third day He shall rise again.” Sad tale of man’s sin, and of the shameful treatment the Lord received at his hands. Were there none to sympathize? Alas! even His disciples are thinking of their own glory. At the moment the Lord announces His going down to the lowest place in suffering service, the sons of Zebedee and their mother come to seek the highest place. They seek their own glory; He sought to do the Father’s will. He is the dependent man; He must suffer; He can share that with them, and in answer to the mother’s request for them, that they may sit, the one on His right hand, and the other on His left in His Kingdom. He replies, “Ye know not what ye ask. Are ye able to drink of the cup that I shall drink of, and to be baptized with the baptism that I am baptized with? They say unto Him, We are able. And He saith unto them, Ye shall drink indeed of My cup, and be baptized with the baptism that I am baptized with; but to sit on My right hand and on My left, is not Mine to give, but for whom it is prepared of My Father.” They can share His sufferings, the inward and the outward sufferings (except atonement; in this He is alone). The rewards He leaves to the Father to give as He had prepared in His sovereignty and purpose. He renounces all claim except to do the Father’s will, and those who serve Him must trust the Father for the recompense.
Matthew 20:24-28. When the ten heard it they were full of indignation. Were they better than the two? The Lord’s explanations show their need of rebuke also. “Ye know that the princes of the Gentiles exercise dominion over them, and they that are great exercise authority upon them. But it shall not be so among you; but whosoever will be great among you, let him be your minister (or servant); and whosoever will be chief among you, let him be your servant (or slave). Even as the Son of Man came not to be ministered unto, but to minister, and to give His life, a ransom for many.” In the world, authority was sought after; but the spirit of Christ sought to serve others. It led to the humblest place, and to seek the good of others. It was the grace that ever shone in bright perfection in the blessed Lord displayed before them – an example for them to depend on the grace of Him whom they served, to take the lowest place in service for others, after the pattern He had put before them.
“These are the principles of the heavenly kingdom: perfect self-renunciation, to be contented in thorough devotedness; this is the fruit of love that seeketh not her own – the yieldingness that flows from the absence of self-seeking; submission when despised; meekness and lowliness of heart. The spirit of service to others is that which love produces at the same time as the humility which is satisfied with this place. The Lord fulfilled this, even unto death, giving His life as a ransom for many.”
Matthew 20:29 begins the closing events of the Lord’s life on earth. It is His last presentation to Israel as the Son of David, the Lord, the true King of Israel, the Messiah. It began at Jericho, where Israel entered the land – the place where the curse so long rested. The two blind men who own Him as Lord, Son of David, have their eyes opened; these are His believing people, and they follow Him.
These picture the believing remnant of Israel, His people, who, in the future day, will wait for His appearing.

Nehemiah: The Building of the Wall, Part 3

Nehemiah 5 is very solemn. There was failure amongst the remnant. As Peter said, and well he knew it, “We are men of like passions with yourselves” – and surely we also know it. Are we better than others in ourselves? Far be the thought. But, O, the grace that has gathered to that blessed one, to whom no man can come except the Father draw him. As our Lord said: “It is written in the prophets, and they shall be all taught of God. Every man therefore that hath heard, and hath learned of the Father, cometh unto Me” (John 6:44). The Father is not gathering souls to poor failing man, but to His own Son.
Men have formed themselves into innumerable bodies; but God by His Spirit has restored the long-lost truth of the one body of Christ – Christ the only true center. It is now an accomplished fact, that the wall of separation from every human society is being built. Souls are gathered on the same basis as at Pentecost, though in themselves but a feeble remnant out of the camp of Christendom. There is the camp of a leavened Christendom, and there is the sacred enclosure outside that camp, gathered to Christ, and bearing His reproach.
This just brings us to the sixth form of opposition – what Sanballat and his companions did when they heard that Nehemiah had built the wall. “Then Sanballat and Geshem sent unto me, saying, Come, let us meet together in some one of the villages in the plain of One. But they thought to do me mischief.”
Then Nehemiah “sent messengers unto them, saying, I am doing a great work, so that I cannot come down: why should the work cease, whilst I leave it, and come down unto you?” (Nehemiah 6:3).
We have had five forms of opposition – grief, laughter, wrath, mocking, and fighting; now we have subtlety without. It is as if they said, Do not be so narrow and exclusive. Do come down from your sacred enclosure to “one of the villages in the plain of One.” “Let us meet together.” Do come down, and sanction us in the plain of One. Do you ask, what was this plain of One? Turn to Nehemiah 11: 35, “Zod and One, the valley of craftsmen.” Do leave the only center of worship within those walls of Jerusalem, and come down to any one of the villages of “the craftsmen.” Well did they know that if the true worship of God was set up within that divine enclosure, they would feel like the Ephesians in after times, that their craft was in danger. “Sirs,” said the men of Ephesus, “ye know by this craft we have our wealth” (Acts 19:23-41).
Thus we have the camp of Samaria, with its villages of craftsmen, on the one side – open, compromising, liberal – willing to meet all, and take counsel with all together; ion the other side a few feeble Jews, gathered in separation on God’s ground, within the hated exclusive walls. And through the help of God they stand firm, and act as those who know they are just where God would have them to be, and doing that which is pleasing in His sight.
It was not one effort, or two, but four times did Sanballat send messengers after this sort, to induce, if possible, the servants of God to give up their exclusiveness, and come down from their excellency to the low level of the plain of One, the villages of the craftsmen. Still God preserved him — “I answered them after the same manner” (vs. 4). To Nehemiah it was a great work to be uncompromisingly for God.
Sanballat, judging after his own heart, now sends the fifth time his servant, with an open letter in his hand: “therein was written, it is reported among the heathen, and Gashmu (or Geshem) saith it, that thou arid the Jews think to rebel, for which cause thou thinkest to build the wall, that thou thinkest to be their king, according to these words.... Come now therefore, and let us take counsel together.” Very firm was the reply, so like a man that walks in peace with God: “There are no such things done as thou sayest, but thou feignest them out of thine own heart.” If Nehemiah had been acting in the pride of a self-seeking heart, then nothing could be more narrow, close, yea, contemptible; but he was acting in the fear of Jehovah, and nothing could be more beautiful and faithful.
Is not all this a picture of the movements around us in this very day? Nothing could be more strikingly so. There is the sacred enclosure of a few feeble saints, gathered to Christ; and there is the great camp of the Greek, Roman, and Protestant churches. And as there were many Jews still in captivity, so are there many Christians in this great camp of Babylon. But is it not written, “There were false prophets also among the people, even as there shall be false teachers among you, who privily shall bring in damnable heresies, even denying the Lord that bought them, and bring upon themselves swift destruction. And many shall follow their pernicious ways; by reason of whom the way of the truth shall be evil spoken of, and through covetousness shall they with feigned words make merchandise of you”? (2 Peter 2:1).
(Continued from page 51).

What God Hath Said on the Second Coming of Christ and the End of the Present Age: Part 3

But perhaps my reader will ask, if Jesus does not teach that the world would be converted by the preaching of the gospel? Does He plainly say the contrary? Let us turn now and carefully examine the Gospel of Luke, and there we shall get a decided answer to the above question. (Luke 12:35-48). In these verses there are two classes of servants. I would observe, a man may be a servant and not a son, as Balaam, and many others. Those servants are greatly blessed who are found watching When the Son of Man cometh. But the evil servant who said in his heart, MY LORD delayeth His coming – and especially that servant who knew his Lord’s will, and prepared not himself, neither did according to His will, shall be beaten with many stripes.” O! is not this a highly favored land? Does any nation know the will of God better than we do? Surely, then, as it was with Judea, so will it be with this land. The heaviest judgments of God’s wrath will be poured out on this now highly favored land. Her doom will be infinitely worse than the dark lands of paganism, where the will of the Lord has not been known. Surely, then, this warning is not a light matter; and who knows how near?
But if you now turn to Luke 17:24-37, the Lord says here most decidedly, that He must be rejected. And this rejection goes right on to the coming of the Son of Man. A rejection which He likens to the days of Noah and of Lot, “Even thus SHALL it be in the day when the Son of Man is revealed.” Yea, so far from the world being converted, He says, “Nevertheless, when the Son of Man cometh, shall He find faith on the earth?” (Luke 18:8).
Luke 19:11-27. In these verses the parable of the nobleman going into a far country is repeated, with the plain prophecy that the great mass of the citizens hate Him, and say right out, that they will not have Him to reign over them. And instead of these being converted, at the return of Christ, they are slain before Him.
Luke 21. This chapter is in many respects parallel with Matthew 24: and Mark 13. We must bear in mind that the listeners to this discourse expected that the long-expected kingdom of God on earth should immediately appear. Instead of which, the Lord makes known a period of great suffering and persecution. What a contrast to all their thoughts. Instead of reigning over the nations, they should be hated of all men for His name’s sake. Then, in the 20th verse, a subject is named that is omitted in both Matthew and Mark: “And when ye shall see Jerusalem compassed with armies, then know that the desolation thereof is nigh.” “Then let them which are in Judea flee to the mountains.” Strange as this might appear; so utterly opposed to the hopes of the nation, yet we know it actually came to pass. The Roman armies did compass Jerusalem, and the Jewish disciples did flee to the mountains. A Jew might have said, O, it is impossible; God has promised that this city shall be the metropolis of the whole earth (Isa. 2:1-4). Now what was the cause of the Jews’ darkness and mistake? They knew not the doctrine of the cross, man’s need of the death of Christ and a new resurrection – existence in Him. And I often think it is the same now. Men do not understand the cross. They think, just as the Jew thought, Judaism would become the kingdom. So men now utterly mistake the gospel by thinking Christianity is destined to improve humanity. If this were the case, there would be no need for the coming of Christ. But the cross is death to humanity. The cross says humanity cannot be improved. Death to it. The resurrection unfolds God’s only principle of blessing, the new creation. Truly every promise of God shall be fulfilled, when the time of the kingdom comes on earth. In like manner some will say, It is impossible that these great destructions should take place, because God hath said, “The earth shall be full of the knowledge of the Lord as the waters cover the sea” (Isa. 11:9). This shall certainly be the case in the days of the kingdom. But before those days come, let us closely observe these words of Jesus. The days of vengeance came on Judea; there was great distress in the land, and wrath upon that people. This prophecy, from verse 20 to 24, may be said to be condensed history – prophecy fulfilled before our very eyes. They (the Jews) fell by the sword; they were led away captive into all nations. Jerusalem is trodden down of the Gentiles. For 1800 years this prophecy has been, and still is fulfilled. Though at the time Jesus uttered these words, His own disciples neither understood nor believed what He said, for we find them afterward asking Him, “Lord, wilt thou at this time restore again the kingdom to Israel?” But does the Lord say how long Jerusalem is thus to be trodden down? Yes, distinctly: He says, “Until the times of the Gentiles are fulfilled.” And what then? Will the world be converted then? The Lord says no such thing. But then takes place, as in Matthew and Mark, the great tribulation, “Distress of nations with perplexity.” “Men’s hearts failing them for fear.” “And then shall they see the Son of man coming in a cloud with power and great glory.”
(Continued from page 55).
(To be continued).

Correspondence: Bible Inspired and Apocrypha Not

Question 93: What proof have we that the Bible is the inspired Word of God, and by what authority do we know that the Apocrypha is not inspired? M. G.
Answer: The Bible claims in both Old and New Testament to be the Word of God (Isa. 1:2, 10, etc.; 2 Tim. 3:15-17; 2 Peter 1:21). To unbelief there is no proof, but to faith it proves itself to be God’s Word by what it works in the soul. Applied by the, Spirit it makes us feel our need, and presents Christ to us, meeting the need and satisfying our affections. Everything in it circles around Him. The Scriptures have one mind, that is, to honor Him. It is this that proves it to faith to be the Word of God. It declares Christ to be the Son of God. It is woven together as one mind He gives it authority in the soul. John 5:39,46-47.
Those who have examined the Apocrypha say that it lacks the above qualities and is thus proven to be an imitation, the writings of uninspired men.

Inspiration of the Scriptures: The Bible - its Unity, Part 5

We must not, however, forget that, with all the similarities, there are very striking contrasts. No two systems could be more distinct and separate than Judaism and Christianity, or law and grace. As to the former, the law said, Do and live; the gospel says, Believe and live; the law said, Thou shalt love God and your neighbor, or be cursed; the gospel says, “God so loved the world that He gave His only begotten Son, that whosoever believeth in Him should not perish, but have eternal life.” With regard to Judaism, the three things which characterized it were, (1), a place of worship on earth; (2), an earthly order of priesthood between God and the people; (3), all the people at a distance from God – outside the veil; but Christianity, in virtue of the accomplished work of Jesus, is characterized in Scripture, (1), by access to God with confidence inside the rent veil; (2), worshiping the Father in Spirit and in truth; and (3), a heavenly order of priesthood – Christ the High-Priest and all believers priests. Any other order of priesthood is subversive of Christianity, and why? Because the believer’s standing is always in heavenly places in Christ Jesus, and the Holy Spirit has come down to indwell and abide with us forever.
Another thing that marks the unity of the Bible is the remarkable way in which its various parts are adapted and interwoven with each other, so that not one of the smallest books of it could be taken from us without serious loss. Perhaps someone would be ready to say, “It would be no loss to miss the book of Esther, for neither God nor Lord is once named in it.” Such, however, have little considered that the omissions of Scripture often show their beauty and perfection. It is so in this and other cases. When many of God’s people returned from the Babylonish captivity, there were some who, though they had the thoughts and feelings of His people, yet did not act in faith in returning to the land God had given them; such God never forsakes, though He does not openly show Himself to them. He always cares for His own, however weak and fallen they may be. This the book of Esther remarkably illustrates, and it is a point of all importance in the ways of God.
Others may say, Could we not part with the book of Proverbs or of Ecclesiastes without loss? Certainly not. In Proverbs 8. we have one of the finest descriptions given of Christ as “wisdom” found in Scripture; and in other parts, many of His actings and wise counsels, besides His kingly power and Sonship. And by losing Ecclesiastes we should be deprived not only of the experience of one who had had it in his power to try everything of earthly blessing to secure happiness, yet found it all to be vanity and vexation of spirit, and everything stamped with death; but we should lose the comfort given us of two things “under the sun” which are not vanity; one seemed “great” to him; this was the deliverance and salvation wrought out on Calvary, alluded to at the end of chapter 9; the other is the service of Christ – “Cast thy bread upon the waters; for thou shalt find it after many days” (Ecc. 9:13-15; 11:1). These things also show that the sacred writings are so adapted to each other, so fitted together and interwoven, that the more they are pondered, the more the unity of the Bible becomes apparent. To lose any part of what we now possess would be like missing some links of a golden chain, and could not fail to produce a gap or defect.
Before closing this part of our subject, there is another character of unity seen amidst all the infinite variety of the Holy Scriptures. We sometimes find the prophets, though unknown to each other, and without any intercourse, going on from the points where others ended, though almost all the prophets went on to the reign of Messiah. Look, for instance, at Isaiah, though he saw terrible failure in Judah and Jerusalem, and even touched on the Jews’ rejection of Christ coming in humiliation, yet he does not speak of their captivity; whereas, Jeremiah, about a hundred years after, not only enters much into their being given into captivity, but declares it will be for seventy years. Daniel, after this, finds out from Jeremiah’s books, that the captivity will be for seventy years, and goes on not only to give a prophetic sketch of the times of the Gentiles, but he also speaks of his own people in the famous prophecy of the seventy weeks. Hosea tells us of their present state and its continuance until the children of Israel return to seek Jehovah (Hosea 3:4-5). The post-captivity prophets give us an appalling description of their moral condition, without hope, except in a few who fear Jehovah, till the Sun of Righteousness shall arise with healing in His wings. The prophecy in Malachi of John preceding the Lord coming in grace, and of Elijah preceding the Lord coming in power and glory, were both referred to by our Lord, thus connecting the Old and New Testaments, between which there is a period of more than four hundred years. Nearly a thousand references are made in the New Testament to the Old; and a multitude of prophetic scriptures in the Old have the records of their fulfillment in the New. It need scarcely be added, that the so-called Apocrypha is nowhere quoted in the New Testament, nor does it commend itself to a spiritual mind as inspired. It is doubtless correctly refused as such.
The same unity of purpose and links of connection are seen in the New Testament. If Matthew closes with showing Christ risen and on the earth, Mark goes on to tell us of His ascension, and sitting on the right hand of God.
Luke tells us not only that they saw Him go up into heaven out of their sight, but the disciples were told to wait for power from on high – the coming of the Holy Spirit; while John’s Gospel goes on still further, for there Christ not only speaks of new relationships. His ascension, His breathing on His disciples the Holy Spirit – risen life – but hints at His coming again, saying, “If I will that he tarry till I come.” In speaking to Thomas He refers to the Jewish remnant being blessed on seeing Him after we are gone; and the last chapter gives us a millennial scene.
So in the epistles. Peter looks at us as “pilgrims and strangers,” going on to the inheritance “reserved in heaven” for us; but under divine keeping all along the path, with trials and difficulties by the way. Paul, however, usually begins with us as in heavenly places in Christ Jesus, and bids us walk here on earth as those who are heavenly, with all our resources in the glorified Son of God, and keeps His blessed coming before us as our hope. John goes on from the rapture at our Lord’s coming, sees us in heaven during the apocalyptic judgments of seals, trumpets, and vials; then, after the judgment of the great whore, and after the marriage supper, we come out of heaven with Christ in manifested glory to judge the quick and the dead, to establish His rightful place and kingdom on earth, and subdue all for God’s glory. It is impossible, then, not to be struck with the element of unity amidst all the almost endless variety of the contents of the Bible, and to see that one mind, and that divine, even the Holy Spirit, must have indited it.
(Continued from page 62).

Practical Conversations With Our Young People

The Lord willing, we purpose taking up a new line in the form of practical conversations on Bible subjects, in which we hope our young readers will take deep interest, as we believe they will be a means of meeting difficulties on the minds of many, and producing more exercise of heart about the varied truths of the Word of God.
How necessary in a day like this when declension is coming in on every hand, that the young in the faith should be well fortified and stand firm for the precious truth of God, that He may be glorified, and that we may be kept from the many little things which we often allow in our daily life that rob us of communion with Christ.
The object of the questions will be to stimulate an interest to study and put in practice the Word of God.
Questions will be given each month and some of the answers to the same will be printed the following month. No initials will be given, so that all may be free to express their thoughts and thus be a help to each other.
We give a few questions that were sent out privately in March, and their answers, so that all may see the character of the work.
We desire to have all answers in by the 15th of each month, addressed to W. E. Pietsch  ... . who will give personal attention to each letter, and will send answer in writing whenever it is necessary.

Questions and Answers for March

1. Give Five Scriptures, proving you are a Christian?
2. What is Repentance?
3. What is Confessing Christ?
4. What is Separation from the World?
ANSWERS TO QUESTION 1.
(a) “If thou shalt confess with thy mouth the Lord Jesus, and shalt believe in thine heart that God hath raised Him from the dead, thou shalt be saved.” Romans 10:9.
(b) “Whosoever believeth that Jesus is the Christ is born of God: and every one that loveth Him that begat, loveth him also that is begotten of Him.” 1 John 5:1.
(c) “We know we have passed from death unto life because we love the brethren. He that loveth not his brother abideth in death.” 1 John 3:14.
(d) “Having predestinated us unto the adoption of children by Jesus Christ to Himself, according to the good pleasure of His will, to the praise of the glory of His grace, wherein He hath made us accepted in the beloved.” Ephesians 1:5-6.
(e) “Ye are all the children (sons, N. Trans). of God by faith in Christ Jesus.” Galatians 3:26.
ANSWERS TO QUESTION 2.
(a) “I abhor myself and repent in dust and ashes.” Job 42:6.
(b) “Father, I have sinned against heaven and in Thy sight, and am no more worthy to be called Thy son.” Luke 15:21.
(c) “Repentance is the full and terrible realization of your sins, or the seeing yourself as God sees you, a lost and guilty sinner, unfit to be in the presence of a holy God.”
ANSWERS TO QUESTION 3.
(a) “Signifying to the world that as a guilty sinner you have accepted God’s pardon.” “Come, see a man, that told me all things that ever I did: is not this the Christ?” John 4:29.
(b) “He that hath My word, let him speak My word faithfully.” Jeremiah 23:28.
(c) “Let your light so shine before men, that they may see your good works.” Matthew 5:16.
ANSWERS TO QUESTION 4.
(a) “The ceasing of doing those things that the people of the world do, or to cease following all the pleasures of this world.”
(b) “Living in communion with the Lord.”
(c) “Be not conformed to this world: but be ye transformed by the renewing of your mind, that ye may prove what is that good, and acceptable, and perfect, will of God.” Romans 12:2.

Questions for April

1. What essential things are necessary to Christianity?
2. Should Christians be poked with unbelievers or keep company with unbelievers?
3. Should we let the things of this world occupy our time and attention at the expense of the things of Christ?
4. Are works necessary to prove our divine life to the world?

A Word Fitly Spoken

At a funeral not long since, among the strangers who gathered around the grave, there was a man, who, with tears in his eyes, watched very intently the coffin as it was being slowly let down in the grave. A person who had observed him, went and inquired, “If the deceased was a relative. or a dear friend.” “No,” he replied, “I never spoke to him in my life, and he never spoke to me but once, but what he then said resulted in my soul’s salvation.”

Truths for Young Christians: Young Men Wanted, Part 1

“Young Men Wanted – Separate from the World – Out and out for God – Devoted to Christ – Young Men, in whom the Holy Spirit of God not only dwells, but RULES.”
The foregoing paragraph in a letter written by a beloved brother in the Lord, impressed us very much. We could not refrain from expressing the fervent wish that it might be true of us. As we mused, our thoughts reverted to Daniel. We wondered why his history is set before us in such detail in the Holy Scriptures. We remembered the words, “Whatsoever things were written aforetime, were written for our learning,” (Rom. 15:4), and as, with this in view, we studied his lifer we noticed three outstanding features that characterized him, and which, we are persuaded, are essential, if we are to respond to this call of God, and if we are, in any measure, to realize the vast possibilities of life. These are:-
Purpose – Prayer – Progress
PURPOSE. When quite a youth, Daniel was carried into captivity, and that because he belonged to the people of God. We presume he had feelings like any other young man, and would not be elated, as he contemplated prospects blighted and hopes blasted. Presently, however, if we may use a worldly expression. fortune seemed to smile upon him. The king wanted young men of ability to fill certain responsible positions. Daniel seemed to be a likely candidate. Position and power were within his reach, but here came the severe test. Aspirants had to eat the king’s meat and drink the king’s wine, and “Daniel purposed in his heart that he would not defile himself” (Dan. 1:8). The Prince of the Eunuchs reasoned – Said he in effect, “You will certainly miss your opportunity; still more serious, if I hearken to you I may lose my head. Furthermore, others ‘of your sort’ (Dan. 1:10) are submitting to this, and why should you appear so odd, or set yourself up as being so much better than your brethren?” But Daniel could not be moved from his purpose; God was with him, his request was granted, and Chapter 1 tells the story of his victory. Observe! His being a man of purpose made him a separate man. Young men! if we are to answer to this call; if we are to be men of God, there must be on our part, separation from the world in every shape and form.
The king’s meat means one thing in your history, another thing in mine. It may be the cigarette or the pipe, the newspaper or the novel – anything that could not exactly be called sinful, but which gratifies the flesh, and does not glorify God. Shall we then purpose in our hearts that we will not defile ourselves with the king’s meat? The measure of our separation will be the measure of our power. The necessary result of the Holy Spirit ruling in us will be, that by engaging our heart with the Lord Jesus where He is now at the right hand of God, He will detach us from this world, where Christ is not, and thus make us a living, active, powerful testimony for Christ in this world.
PRAYER. We notice next, that Daniel was a man of prayer. He was by no means a man of leisure. Ease was a word that did not seem to find a place in his vocabulary. Immersed in the business of the State, alive to his duty as a responsible servant, and conscientious in the discharge of all that fell to his lot, he, nevertheless, found opportunity three times a day to pray. (Dan. 6:10). We feel that just here we touch a tender point, yet one of vital importance. If we are to have power with God and with men, we must make time to pray. It may necessitate our wrenching ourselves from Blanket Square half-an-hour earlier in the morning, and returning thither half-an-hour later at night. It may claim from us the spare minutes at lunch time that we, at present, devote to recreative reading. Be that as it may, we must pray! and PRAY!! and PRAY!!! if we are to be men of God in this Godless world. How many of us in our unconverted days did not mind rising early for a day’s so-called pleasure, or remaining till the small hours at the dance or the billiards, denying ourselves many of the comforts of this life, that we might minister to the flesh. Is our precious Lord not worthy? Is not our desire, to be here wholly for Him, sufficiently strong to cause us to find time to spend in His presence, that we may be equipped by Himself, and get from Him needed grace and strength to be efficient witnesses for Him during His absence? Behind Daniel’s practice was determination, behind his determination was devotedness, and behind his devotedness was God Himself. So thoroughly did the claims of God press upon that young man’s soul, that he determined to be and to do for God at all costs, and on bended knee, in His presence, he got all that he required for his brilliant testimony.
(To be continued).

Scripture Study: Matthew 21

Matthew 21:1-6. As the Lord draws near to Jerusalem, He prepares for His last presentation to Israel as their Messiah. He sent two of His disciples, saying, “Go into the village (Bethphage) over against you, and straightway ye shall find an ass tied, and a colt with her: loose them, and bring them unto Me. And if any man say aught unto you, ye shall say, The Lord hath need of them; and straightway he will send them.” This was done, that it might be fulfilled which was spoken by the prophet (Zech. 9:9), saying, “Tell ye the daughter of Sion, Behold, thy King cometh unto thee, meek and sitting upon an ass, and a colt the foal of an ass.” The Lord hath need of them, points to Jehovah’s claim on the hearts of men. So will it be in the day of His glory. (Psa. 110:3). Comparing Zechariah 9:9 with verse 5, we see that this is a testimony to His person but not yet the setting up of His kingdom in power. He is still the meek and lowly One.
Matthew 21:9-16. They brought the ass and colt, and spread their clothes on them, and set Him thereon. A very great multitude was there, moved by God to bear testimony to Him; and they spread their garments in the way, and cut down branches and strewed them in the way. “And the multitudes that went before. and that followed, cried, ‘Hosanna to the Son of David: Blessed is He that cometh in the name of the Lord, Hosanna in the highest.’” This is from Psalm 118:25, 26. Verse 24 speaks of the rest under the reign of Christ. “The day Jehovah hath made.” This testimony to His Messiahship, the power of God constrains them to give. Happy for them if it was from their hearts. When He was come into Jerusalem, the city or people were moved, saying, “Who is this?” The multitude with Him replies, “Jesus, the prophet of Nazareth of Galilee.” The despised Nazarene is for the moment seen as the King of Israel. He will also show His authority in the temple, bearing witness against the evils there, yet still He is the lowly one. He cast out all who bought and sold in the temple, and overthrew the tables of the moneychangers, and the seats of those who sold doves, and said unto them, “It is written, My house shall be called the house of prayer (Isa. 56:7), but ye have made it a den of thieves” (Jer. 7:11).
His righteousness is seen here: “Thou lovest righteousness;” but what is He to the needy ones there before Him – the lame and the blind? He healed them; He is their Savior and Healer. (Psa. 103:2-3). He is perfect in grace also.
The chief priests and scribes saw the wonderful things He did, and heard the children crying in the temple, “Hosanna to the Son of David.” It displeased them much, and they spoke to Him about it. He answered: “Yea, have ye never read, Out of the mouths of babes and sucklings Thou has perfected praise?” (Psa. 8). It is a testimony to Him as Son of Man, but He is rejected.
Matthew 21:17. He leaves them; He could not lodge there, so He went out to Bethany.
Matthew 21:18-22. In the morning, as He returned into the city, being hungry, He saw a fig tree, but on looking at it found no fruit, only leaves, and said unto it, “Let no fruit grow on thee henceforward forever.” And presently the fig tree withered away. This is a figure of Israel – man in the flesh; man did not and cannot bear fruit for God, the flesh profiteth nothing. God will act in grace under the new covenant, and Israel in that new condition will bear fruit, but not as the fig tree. It is useless to spend efforts on it to make it better. Culture will not help the flesh to produce fruit for God. It withered away. The disciples marveled at it, and the Lord says: “Verily, I say unto you, If ye have faith, and doubt not, ye shall not only do this which is done to the fig tree, but also if ye shall say unto this mountain, Be thou removed, and be thou cast into the sea; it shall be done. And all things, whatsoever ye shall ask in prayer, believing, ye shall receive.”
That mountain has been cast into the sea; Israel, as a nation, has lost her place, scattered in the sea of the nations. The believing remnant of them are brought into the church of God, and are members of the body of Christ. But we see also great encouragement to exercise faith in going to God in prayer. Lord, help us to exercise our hearts in believing prayer.
Matthew 21:23-27. The chief priests come to Him demanding: “By what authority doest Thou these things? and who gave Thee this authority?” The Lord tests their capability to judge by asking a question, which, if they will answer, He, also, will answer their demand. “The baptism of John, Whence was it? from heaven or of men? And they reasoned with themselves, saying, If we shall say, From heaven. He will say unto us, Why did ye not then believe Him? But if we shall say, Of men; we fear the people; for all hold John as a prophet,” and they answered Jesus and said, “We cannot tell. And He said unto them, Neither tell I you by what authority I do these things.”
They had neither conscience for God and the truth, nor had they any competency to judge. They were judged by Him. God’s work does not need the authority of man. If a servant has the authority of Christ as his Master, he needs no other. If Christ has not given him authority, what he can get from man counts nothing with God. It is wicked assumption.
Matthew 21:28-32. The Lord goes farther. In this parable He convicts those very men, religious leaders as they were, of being worse before God than the publicans and harlots whom they despised. They pretended to do God’s will like the one who said, “I go, sir,” and went not. Whereas, the one who said, “I will not,” afterward repented and went. They admitted that the one who repented and went, was the one who did his father’s will. Jesus saith unto them, “Verily, I say unto you, that the publicans and harlots go into the Kingdom of God before you. For John came unto you in the way of righteousness, and ye believed him not: but the publicans and the harlots believed him; and ye, when ye had seen it, repented not afterward, that ye might believe him.” Like the elder brother in Luke 15, they refused to humble themselves or recognize the work of grace in others.
Matthew 21:33-46. This parable shows Israel’s failure in their responsibility to God; everything had been done for them, but they rendered nothing in return. His servants which were sent to them, received abuse, were rejected, persecuted, killed, and lastly, He sent His Son, and they said, “This is the heir; come let us kill him, and let us seize on his inheritance.” And they caught him and killed him and cast him out of the vineyard. When the lord of the vineyard cometh, what will he do unto those husbandmen? they say unto Him, “He will miserably destroy those wicked men, and will let out his vineyard to other husbandmen, which shall render him the fruits in their seasons.” They speak their own sentence; it is just what they were doing. The Lord applies to these builders Psalm 118:22-23. They were in open rebellion against their God. Therefore, I say unto you, “The Kingdom of God shall be taken from you, and given to a nation bringing forth the fruits thereof, and whosoever shall fall on this stone shall be broken.” That is what they did (Rom. 9:32-33). “But on whomsoever it shall fall, it will grind him to powder.” This speaks of what will happen to the rebellious nation when the Lord comes again to them.
Dull as the Pharisees were, they perceived that He spake of them. They would fain have laid hands on Him, but they feared the multitude, because they regarded Him as a prophet.
His warnings were unheeded or despised, and all His testimonies to their need of a Savior only brought out the enmity that filled their hearts. Such is man, such are we, till grace has wrought in our souls.

Nehemiah: The Building of the Wall, Part 4

Is not this terrible picture fulfilled before our very eyes? and is not this merchandise wicked in God’s sight, though one of the most respectable professions of this day? So fashionable is it that many of God’s own children are entangled in it, and follow its pernicious ways. If you would read a further description of this modern camp of Samaria, read 2 Timothy 3.
God in His sovereign grace has been working in this camp, and many souls, we trust, have been saved. He can work in Greece, in Babylon, or Rome. Satan has used this circumstance, like Sanballat of old, and repeated have been the temptations to come down to some one of the villages in the plain of one. Only give up your narrow, illiberal exclusiveness, and come down to the level of the craftsmen; only acknowledge the clergy, and you may hold what you like. Do only come down from that hateful wall around the true ground of God; or, if you will not come down and acknowledge us, then you are but a sect in Jerusalem, as much as we are. You are the exclusives. Come down now; come, let us take counsel together.
Those who are separated to Christ can say, All this is feigned out of your own hearts. You know we are no sect. You know that we do not exclude any one that God has gathered to Christ, and who only seeks His honor and glory. Is it not a solemn thing to oppose the present work of God, as Sanballat did of old?
“But,” says an eminent evangelist, who remains in and approves the camp of Samaria, “will you not go with us to the preaching’s?”
“I don’t know that I will,” said a young Christian.
“What! will you not go where God is working?”
“No, I do not know that I will.” “How is that?”
“Why, God is a sovereign; but I am a subject.”
The same evangelist said to another. “I am sorry you are not with us.”
“Indeed, I am more happy to be with the Lord.”
“Why, is He not with us?”
“That may be in His grace, and I pray He may use you much; but you know you are not with Him outside the camp.”
No, we cannot be with the army of Samaria, and at the same time with the few within the sacred enclosure of the rebuilt wall. “Let us go forth, therefore UNTO HIM without the camp, bearing His reproach” (Heb. 13:13).
There is the sacred enclosure of the feeble ones in Philadelphia (Rev. 3), those who have gone forth from Sardis (Protestantism), unto Him, the holy and the true. And there is the boasting camp of Laodicea, outside of which the precious Lord knocks at the door. Are you, my reader, in the camp of Laodicea, that which is rich, increased with goods and have need of nothing? Then you have never yet gone forth unto Him, bearing His reproach. May God by His Holy Spirit make this clear to you. How could the gathered saints to Christ, outside the camp, come down and sanction the craftsmen in the valley of one? No; surely twenty thousand on the plain of One should not attract my soul from Christ.
It is a great work that God is doing by the Holy Spirit, greater far than the work He did by Nehemiah. And the enclosed remnant in Jerusalem were not more distinct from the camp of Samaria than the souls gathered to Christ are distinct from the camp of Christendom.
O, that they who have been thus gathered were more true to Christ. They have failed, but they cannot give up the only true ground of gathering around Him. They own their failure, but they cannot give up Christ.
This brings us to the seventh form of opposition to the work of God – danger within. This will illustrate the cunning subtlety of Satan. In the last case it was the temptation from without to go down to the platform of one – to compromise all that God has taught us, and sanction the craftsmen and merchandise of Christendom. Now the mischief is within. We shall do well carefully to consider it.
Sanballat does not appear on the surface. “Afterward I came unto the house of Shemaiah, the son of Delaiah, the son of Mehetabeel, who was shut up; and he said, Let us meet together in the house of God, within the temple: and let us shut the doors of the temple, for they will come to slay thee; yea, in the night will they come to slay thee.” Does not this look very plausible? Surely it is right to meet together in the house of God. But to shut the doors of the temple would be with us to put the light under a bushel. The temptation is to give up the testimony. If we will not join the religious activities of the camp, then let us seek in shut-up selfishness, and fear of man, to enjoy that sacred place of blessing and communion among ourselves, and take care of ourselves.
The opposition may indeed become more grave. But shall we give up the testimony, if it be even to save our lives? or shall we flee?
Shall we through fear shut ourselves up? Is this the mind of God? “And lo I perceived,” said Nehemiah, “that God had not sent him....Tobiah and Sanballat had hired him.” Let us also, then, having this certainty that the work is of God, not be weary, or shrink from it.
It seems to me the greatest trial and danger was from false brethren. The enemy knew that the wall was built: “They were much cast down in their own eyes: for they perceived that this work was wrought of our God” (Nehemiah 6:16). But the false brethren, even “nobles of Judah, sent many letters unto Tobiah, and the letters of Tobiah came unto them. For there were many in Judah sworn unto him.” This is, indeed, sad, and a great trial, when those who outwardly take the place of being gathered to Christ, yet like these mixed marriages of Judah, we find some dear brethren in the Lord seeking to mingle the principles of the camp with those of God. Nor should this surprise us, remembering the words of the apostle, “Also of your own selves shall men arise speaking perverse things” (Read Acts 20:29-35). No doubt these half-an-half brethren are the greatest stumbling-blocks in the way of inquiring souls. Let those gathered to Christ beware of evil associations – the greatest present danger.
(Continued from page 80).
(To be continued).

What God Hath Said on the Second Coming of Christ and the End of the Present Age: Part 4

Now, my reader, from all these words of Jesus, can we come to any other conclusion than this — that the millennium cannot possibly take place before the coming of Christ in the clouds of heaven; until then wars, tumults, on Judah days of vengeance; on all nations distress. Read, then, carefully the solemn warning, 34-36, “Take heed to yourselves.” O, do not be deceived by the cry of peace and safety. “For as a snare shall it come on all them that dwell on the face of the earth.” Ah, you see that professor going to the world’s concert, or to the world’s feast, to eat and to drink with the drunkard. There goes another with anxious brow and keen, piercing eye, grasping at the world’s deceitful wealth. Ah, these, and thousands more, are saying in their hearts, “My Lord delayeth His coming.” “Watch ye, therefore, and pray always, that ye may be accounted worthy to escape all these things that shall come to pass, and to stand before the Son of Man” (Ver. 36).
You may have observed, that all the words of Jesus so far, have reference to this earth, or His coming again to this earth. Jerusalem in Judea has been the center of His instruction. This was suited to the nation in the midst of which these prophecies were delivered.
We now turn to an entirely new subject. (John 14:2-3). I say new subject, for I am not aware of a single verse, from Genesis to this very passage, where this wondrous fact had ever been fully revealed. We forget this when reading these divine words of comfort. Every hope in the disciples around the blessed Lord, in this night of sorrow, was centered in Jerusalem, as the place of His reign. But now, His last words having been spoken to the nation in John 12, He unbosoms the secrets of His heart, for the comfort of His chosen few, during this time, or period, when He should have left them in the world alone. John 13 unfolds the tender grace of our High Priest on high. In the east it is customary for one servant to hold the basin, and for another to pour the water. But Jesus did not ask Peter to hold the basin, and John to pour the water. No, the precious Lord did it all: He took the towel, He took the basin, He poured the water, He washed their feet. O! that we better knew that tender heart. Cheer up, my drooping brother Christian; it was Jesus’ work alone to atone for sins on Calvary. It is Jesus’ work alone, as thy Great High Priest, to keep thy feet clean. Worthy alone art Thou, O Lamb of God. Thou art the author and the finisher of my salvation.
My reader, art thou a believer? Then thou art justified from all things through the precious blood of Christ, clean every whit. Then do not forget He lives to keep thee clean.
Then, in John 14, the precious Lord can hide from His loved ones no longer the amazing secret — “In My Father’s house are many mansions; if it were not so, I would have told you. I go to prepare a place for you. And if I go and prepare a place for you, I will come again and receive you unto Myself, that where I am, there ye may be also.” Of course, this was so new, they could not make out either whither He went, or the way. They had heard of the future glory of Jerusalem; but mansions in the Father’s house on high, and a place prepared for them! What, poor sinful fishermen to have a place with God the Father! O, amazing grace! Man, through sin, lost the garden of God; but Jesus was about to give His heart’s blood, that He might bring us to God Himself in heavenly glory. And mark the certainty. As surely as He has died and risen again, and gone to the Father’s house to prepare a place for us, even so sure is it, that He will come again and receive us unto Himself. Ah, what would the soldier give, in the midst of the battle’s roar, to have the certainty of reaching the home he loves – or the mariner in the midst of the raging storm? What comfort, then, these words of Jesus give! However fierce the conflict – however dark and loud the roaring tempest, the blest home of His presence is sure. O, think of this, ye tried and desolate ones – ye fellow-believers, who are widows, or orphans, in a cold world! O, cheer up, ye afflicted ones! A little while: your home is certain. Is Christ your portion now? Then your home, sweet home in His presence, is most blessedly certain. “Let not your heart be troubled, neither let it be afraid.” Think not when ye see Him, to meet an angry Judge. He who hath loved you and washed you in His own blood, comes to receive you to Himself. He says, That where I am, there ye may be also.” Perhaps you say, That may be true to them who deserve it. Did those who sat and heard these new words of wondrous grace deserve it? Ah! full well did He know. Yea, this wondrous disclosure of eternal love, was reserved to the very night on which they all forsook Him and fled. The Lord deepen in our souls the sense of this untold grace!
(To be continued). (Continued from page 84).

Correspondence: John 8:56; Isa. 28:17-18; Baptism

Question 94: “Your father Abraham rejoiced to see My day: and he saw it, and was glad.”
John 8:56. What day and when did he see it? A. A.
Answer: Abraham realized the beginning of the fulfillment of Jehovah’s promises in the birth of Isaac, and in Gen. 22, where he offers up Isaac, he receives him from the dead in a figure (Heb. 11:17-19).
The promise of future blessing is here confirmed in a type of Christ in resurrection. Verses 16-18 bid him look forward to the setting up of Christ’s kingdom, the blessed result of the work of Christ. (Psa. 22:22-31: Gal. 3:16). Faith rejoices in the abiding Word of God.
Question 95: What is the covenant in Isaiah 28:17, 18? L. L.
Answer: The Jews in the tribulation period will make a covenant with Antichrist to be allowed to go on with their temple worship undisturbed, but the Antichrist makes an image of the beast and commands them all to worship it. The godly need to flee for their lives then: they would rather die than worship a man or his image.
(See Dan. 9:27; Matt. 24:15; Mark 13:14; Rev. 13:12-17.)
Question 96: What is the use of baptism in water? What is the difference between a baptized person and one unbaptized? I. A.
Answer: John’s baptism was for Israel only. In it those baptized owned Israel’s ruined condition, and by it they committed themselves to God’s mercy as having forfeited all claims to Israel’s blessings. With Christ and John it was fulfilling all righteousness.
Christian baptism was instituted by the Lord Jesus after He rose from the dead. It was committed to His servants with the gospel. No one can baptize himself. The servants are therefore the responsible ones. By it the servants receive others into the profession of Christ’s name. Every one born into this world is either a Jew or a Gentile, but when baptized unto Christ’s name in this symbol of death, has put on Christ as a profession (Gal. 3:27). It is the means whereby the servants receive others coming into the House of God, the professing church on earth (1 Tim. 3:15). It has nothing to do with heavenly claims. It must not be confounded with being born again, that is, a divine operation of the Word and Spirit (John 1:12-13; 3:5-6; 1 Peter 1:23).
The Holy Spirit is given when a man believes the gospel of his salvation (Acts 10:44; Eph. 1:13). This is quite distinct also from baptism. Baptism is therefore what is outward, changing a person’s position on the earth.

Inspiration of the Scriptures: The Bible - its Unity, Part 6

The Bible – Its Unity
The Bible must be perfect. Being God’s words they could not be otherwise, for all His works, whether in creation or redemption, bear the stamp of being perfect. “The law (or doctrine) of Jehovah is perfect converting the soul;” and “His way is perfect” (Psa. 18:30; 19:7).
Again we read, “The words of Jehovah are pure words; as silver tried in a furnace of earth, purified seven times,” and that “Every word of God is pure” (Psa. 12:6; Prov. 30:5). The Bible must be perfect, because, as we hope we have fully proved, it emanates from God.
It is also perfect in being able not only to make wise unto salvation through faith which is in Christ Jesus, but because it is sufficient to furnish the believer completely unto every good work (2 Tim. 3:15-17).
The Holy Scriptures are infinite, too, in their quality because divinely perfect. Take up another book and you may soon master most of its contents, but the written word who can grasp? If an inspired apostle had to say, “We know in part, and we prophesy in part,” we can surely add, “The little we know we know very imperfectly.” Who can say he has fully learned the divinely-given ministry of any part of Scripture? And why? Because being God’s Word it is infinite in its height, and depth, and length, and breadth. We do well to remember this; and that because we are finite creatures, we can know only in part, and give out to others but in part.
Few things show more the divine perfection stamped upon the Bible than its infallible accuracy as to what has been already fulfilled. Let us look at a few instances. More than four thousand years ago, Jehovah said, “While the earth remaineth, seed time and harvest, and cold and heat, and summer and winter, and day and night shall not cease” (Gen. 8:22); and do they not continue to this day?
Again we read, that Abram’s seed (therefore reckoned from Isaac’s birth) should be a stranger in a land that is not theirs, etc., for “four hundred years” – which would be consequently four hundred and thirty years after the promise made to Abram; hence we read, “It came to pass at the end of the four hundred and thirty years, even the selfsame day it came to pass, that all the hosts of Jehovah went out from the land of Egypt” (Gen. 15:13; Ex. 12:41; Acts 7:6; Gal. 3:16-17).
Again, in the time of Jeroboam’s abominations. a man of God came unto Bethel by the word of Jehovah, and cried, saying, “O, altar, altar, thus saith Jehovah; Behold a child shall be born unto the house of David, Josiah by name; and upon thee shall he offer the priests of the high places that burn incense upon thee, and men’s bones shall be burnt upon thee. And he gave a sign the same day, saying, This is the sign which Jehovah hath spoken; behold the altar shall be rent, and the ashes that are upon it shall be poured out. And it came to pass when King Jeroboam heard the saying of the man of God, which had cried against the altar in Bethel, that he put forth his hand from the altar, saying, Lay hold on him. And his hand which he put forth against him dried up, so that he could not pull it in again fo him. The altar also was rent, and the ashes poured out from the altar, according to the sign which the man of God had given by the word of Jehovah. And the king answered and said unto the man of God, Entreat now the face of Jehovah thy God, and pray for me, that my hand may be restored me again. And the man of God besought Jehovah, and the king’s hand was restored him again, and became as it was before” (1 Kings 13:1-6). Now look at the accurate fulfillment of this saying of the man of God about three hundred years after.
We read of King Josiah, “Moreover, the altar that was at Bethel, and the high place which Jeroboam, the son of Nebat, who made Israel to sin, had made, both that altar and the high place he brake down, and burned the high place, and stamped it small to powder, and burned the grove. And as Josiah turned himself, he spied the sepulchers that were there in the mount, and sent, and took the bones out of the sepulchers, and burned them upon the altar, and polluted it, according to the word of Jehovah, which the man of God proclaimed, who proclaimed these words. Then he said, What title is that I see? And the men of the city told him, It is the sepulcher of the man of God which came from Judah, and proclaimed these things that thou hast done against the altar of Bethel” (2 Kings’ 23: 15-17). Were words ever more solemnly and more accurately fulfilled? How little men think they are refusing God’s Word, and dishonoring His holy name when they speak of their opinions of the Scriptures, instead of bowing implicitly to their authority and instruction!
In reference also to God’s dealings with some of the ancient cities recorded in Scripture, we have the most accurate and solemn fulfillment. Look, for instance, at one of the finest cities the world ever knew, and one which existed very early after the Deluge – Nineveh. We know from the book of Jonah that it was “a very great city.” Historians tell us that it extended in length about eighteen miles, and was surrounded with a wall more than a hundred feet high, wide enough to drive three chariots abreast, and ornamented with fifteen hundred towers. The breadth of the city was about twelve miles. Hence Jonah speaks of it as an exceeding great city of three days’ journey; and if the “six score thousand persons in it, who could not discern between their right hand and their left,” refer to young children, the population of the city must have been very large. The “much cattle” also intimates that there were fields, or parks and palaces, within the enclosure of its high and massive wall; and its ruins show there must have been extensive and magnificent buildings in it.
The modern excavations prove the magnificence of the past and fallen greatness of Nineveh. With all her worldly and royal splendor, Jehovah said, “I will make thy grave, for thou art vile  ... . Nineveh is laid waste, who will bemoan her  ... . her young children were dashed in pieces at the top of all the streets; and they cast lots for her honorable men, and all her great men were bound in chains” (Nah. 1 and 3). Another prophet said, He “will make Nineveh a desolation, and dry like a wilderness. And flocks shall lie down in the midst of her, all the beasts of the nations; both the cormorant and the bittern shall lodge in the upper lintels of it; their voice shall sing in the windows; desolation shall be in the thresholds; for he shall uncover the cedar work. This is the rejoicing city, that dwelt carelessly; that said in her heart, I am, and there is none beside me; how is she become a desolation, a place for beasts to lie down in! Every one that passeth by her shall hiss, and wag his hand” (Zeph. 2:13-15). It is said that the whole surface of the country is now covered with fragments of bricks and pottery. Not only is it “desolate,” but it has long appeared a huge misshapen mound, like a large grave, covered with rank vegetation, and a place for beasts to lie down in; so literally has the prophetic word been fulfilled. It is probable that Nineveh was built soon after the confusion of tongues.
(Continued from page 90).
(To be continued).

Practical Conversations With Our Young People: Unequally Yoked

We desire to draw attention particularly to question 2 of April, feeling the importance of the answer as given to us in 1 Corinthians 6:14-18.
There we find positive instruction to Christians, young and old, not to be unequally yoked together with unbelievers. When Christ is the object before our souls, and we are enjoying His love, thinking of all that He has done for us, we delight to speak of Him. His character as man while He was here on earth fills our souls, so that we may say, “O, Lord, make me more like Thyself!” It was His delight to do the will of Him that sent Him. “I delight to do Thy will, O My God; yea, Thy law is within My heart” (Psa. 40:8). is it the desire of our hearts to please Him who so loves us; to be like Him in all our ways? If so, it will be an easy matter to obey such a Scripture as we have before us, and the result will be praise to Him, happiness in our own souls, and we will be preserved from much sorrow that many have had to pass through who have disobeyed it. Read the article “Unscriptural Marriage” in Vol. II. We could relate many such cases, but may that suffice as a warning, to preserve all our young friends from such a step, or an unequal yoke in any connection.
The first wrong step in that line is a lack of enjoying the Lord in the soul; then comes unfaithfulness to Him, and instead of speaking about Him to the unsaved acquaintances, there is the dropping down to their level, and joining in with them in their conversation.

Questions for May

Always give Scripture for answers, as well as your own expressions.
1. Why can we not serve two masters?
2. What kind of fruit does the Lord look for from the Christian?
3. Will faithfulness to the Lord always bring success?
4. Why would the Lord rather have obedience than mere religious activity?
5. Give five scriptures, showing that all have sinned.
Send answers to address given on inside of front cover.
Answers to April Questions will be published in June.

Results Are in God's Hand

A discouraged minister had the following strange dream: He thought he was standing on the top of a great granite rock, trying to break it with a pickaxe. Hour after hour he worked on, with no apparent result. At last, he said: “It is useless; I will stop.” Suddenly a man stood by him and asked: “Were you not allotted this task, and if so, why are you going to abandon it?” “My work is vain; I can make no impression on the granite.” The stranger solemnly replied: “That is nothing to you; your duty is to pick, whether the rock yield or no. The work is yours, the results are in other hands; work on.”
In his dream he saw himself setting himself anew to his labor, and at his first blow the rock flew into hundreds of pieces. This was only a dream, but it proved a valuable and never-forgotten lesson to the minister, and a means of comfort and cheer to his soul.
Would it not be well for all of us to remember that results are ever in God’s hands, and although the great wheels of His government may seem to us slow, yet His purposes will surely come to pass. He is for His people, and will bring to pass that which is for their truest interests. He never lacks for means, for “All things serve His might.”
Like the minister on the rock, our duty is to “pick on.” When God’s time has come – and He never mistakes as to the right moment – the difficulty will vanish.

Truths for Young Christians: Young Men Wanted, Part 2

PROGRESS. Lastly, Daniel was a man of progress. He made progress, first, in the things of God, and secondly, in the things of this life. Let us consider the last, and least important, first. A brother in the Lord asked recently, “Why is it that some young men do not work so well after they profess to be saved as before?” We ventured to reply, “Because they are not properly saved.” There is a pious idea abroad that if a young man gets on in his profession or business, he is worldly minded, and self-seeking. Now, we say unhesitatingly, that he who sets himself to succeed and make his mark in this world, making all else subservient to that, may realize his desire, but the nearer he gets to the goal of his ambition the further he will be from God, the less like our adorable Lord, and the more unfit for being used by the Holy Spirit.
On the other hand, he who seeks to be here for God, will manifest it in the everyday commonplace details of life, in a way that will bring glory to God, and that God will, in His own time, duly honor. Daniel commenced at the foot of the ladder, and before long he stood on the top rung. “A dangerous position,” some one may remark. Yes; if we leave God out. As it was, Daniel was equally unaffected when the world showered upon him her honors, and when hell turned upon him its artillery.
He was as calm in the den of lions as in his official sanctum. He fully met all the requirements of his royal master, and he did it in such a way that his enemies could not find the tiniest blot on which they might fasten in order to dislodge him. All this was the result of his being a man of purpose, a man of prayer, a man of power, a man of God.
Finally, he made progress in the things of God.
God disclosed to him His secrets. “The secret of the Lord is with them that fear Him” (Psa. 25:14). He was as ready to confide in the youthful Daniel as in the veteran Abraham. Daniel did not require to read the newspaper to be informed as to what was going on, or to learn what was about to happen. He was in immediate, intimate touch with Him “who worketh all things after the counsel of His own will” (Eph. 1:11). Then he received the splendid announcement direct from the Court of Heaven that he was a “Man greatly beloved.” He learned that he was “greatly beloved” by God, and that heaven was cognizant of the fact. We confess, the height of our ambition is, not to be conspicuous in this world, not to be able to sway multitudes by impassioned oratory, not to be great amongst the people of God; but it is, to receive this magnificent degree of M. G. B., to know that we are approved and beloved by God. Let us remark in closing, we have a glorious person for whom to live, our blessed Lord Jesus Christ. In devotedness to God and love to us, He “gave Himself,” and shall we hold anything back from Him?
We have also the power to be here for Christ; that is the power of the Holy Spirit; but we shall only realize that power, as He “not only dwells, but rules in us.” Some of us are in lodgings, and as lodgers we have no say in the management and control of the house in which we dwell. May we say it reverently, we fear the Holy Spirit has been having only the place of a lodger in many of our hearts. He indwells all who believe, but He desires to fill us (Eph. 5:18), to rule in us, to dominate us, to control every detail of our lives, and to conform us morally now to that blessed one, to whom we shall be actually conformed when we see Him face to face. And now, what is to be the result of our meditation? We have penned these lines, not with the idea of lecturing young men for their shortcomings, but as the result of our own exercise before God, and as the expression of our own desire.
We would say to every dear reader, let us make an inventory of our lives; let us face boldly our liabilities and our assets; let us discover where the wastage is. Let us get God’s thoughts as to the priceless value of the present moment; let us enter at least in measure into His appreciation of His Beloved Son, for whom He desires us to live; let us get deep down in our souls the sense of the infinite power of the Holy Spirit, with which God is ready to endue us; let us consider the immense distinction of being permitted to represent our absent Lord in the scene of His rejection; let us remember that that honor and privilege can be ours only while we are in this world, and let us definitely consecrate ourselves body, soul and spirit to God, and seek by His grace, and in the power of His Holy Spirit to “live henceforth, not unto ourselves, but unto Him who died for us and rose again” (2 Cor. 5:15).

Human Happiness and Heavenly Joy

Your joys, O vain world, are all fleeting and frail.
Like riches, they make themselves wings;
Your Gaiety’s often keen Misery’s veil,
And Melody mourns while she sings.
Our Fount of delight is unfailing, divine;
In grief we can smile through our tears;
The joy of the Lord is a heavenly wine,
Which strengthens the heart that it cheers.
Then let us sing freely our heavenly joys,
Of pleasures which bloom evermore,
The peace of His presence, where nothing annoys,
The praises of Him we adore;
Declaring to all, what a portion is ours
In Jesus, the Son of God’s love,
Extolling His Name, till with perfected powers,
We worthily praise Him above.
(Continued from page 97).

Scripture Study: Matthew 22

Matthew 22:1-14. In the first part of this parable of the kingdom of heaven we have another view of Israel’s failure. Not this time their responsibility to bear fruit, but on the ground of grace – the grace of the gospel – which, if received, would produce fruit.
The kingdom of heaven is like unto a king, which made a marriage for his son. God would have His Son honored. The Jews, the invited ones, are bidden to the wedding, but they would not come. In the second vex se the message was during the Lord’s lifetime on earth. (cf. Matt. 10:5-6). But it was rejected; they would not come. In the fourth verse again the message goes forth, after the death and resurrection of Christ, when the work of atonement was finished: “I have prepared My dinner”  ... “and all things are ready; come unto the marriage.” “But they made light of it, and went their ways, one to his farm, another to his merchandise.” These are only seeking their own interests and shutting God out. The rest took His servants, and entreated them spitefully, and slew them. This is the religious opposition; the nation is hardened in its sins. Peter, in Acts 3, preached grace to the. nation, but the grace was rejected; Stephen was murdered. And because of it the King sent His armies, destroyed those murderers, and burned up their city. They, refusing grace for themselves, and also “forbidding us to speak to the Gentiles, that they might be saved, to fill up their sins alway; for the wrath is come upon them to the uttermost” (1 Thess. 2:15, 16; cf. Matt. 18:23-34), are now an outcast nation, though there is still a remnant of them saved as individuals, and brought into the church (Rom. 11:5; 1 Cor. 12:13).
Matthew 22:8-9. The servants are now sent to the highways (that is, the Gentiles) and gather together all, as many as they found, both bad and good; thus the wedding was furnished with guests. Whatever the character of the guests before, to be there they must have on the wedding garment, and at such an occasion this was furnished by the King. The standing of the guests depended on having it. High or low, rich or poor, religious or irreligious, moral or immoral, none are suited to the place without the wedding garment. All must be tested. This world is the robbing room; the righteousness of God put on by faith is the wedding garment. A sample is shown us of this testing. The King came in to see the guests and saw there a man which had not on a wedding garment; and He saith unto him, “Friend (he was really an enemy but is taken on his profession), how camest thou in hither not having a wedding garment?” He had nothing to say; no excuse, for a garment was provided; no use to say anything of his own goodness. It is. the wedding garment alone that will fit him to be there. Then said the King to the servants (not the gospel preachers this time, but the angels (Matt. 13:41-49), “Bind him hand and foot and take him away, and cast him into outer darkness; there shall be weeping and gnashing of teeth.” The end of the empty professor is eternity with the lost; his privileges have added to his torment; he is beaten with many stripes (Luke 12:47). The Jew will be judged by the law, and the Gentile shall perish without law (Rom. 2:12), but the Christ rejector has added to his many sins the worst of all in rejecting the Savior.
Matthew 22:14. “For many are called, but few are chosen.” The gospel goes out to all and may seem to bring in many. Some say, “I am a church member.” Yes, but have you taken your place as a lost sinner before God? Have you submitted yourself to God’s righteousness? (Rom. 10:3). This is what marks out those who are chosen. The good news calls them, but they do not let the need into their souls. “If our gospel be hid, it is hid to them that are lost; in whom the god of this world hath blinded the minds of them which believe not, lest the light of the glorious gospel of Christ, who is the image of God, should shine unto them” (2 Cor. 4:3, 4). The marks of those chosen of God are, they own their no goodness, and believe the gospel (2 Thess. 2:13, 14).
Matthew 22:15-22. Different classes now come forward. The Pharisees take counsel how they might entangle Him in His talk. They send their disciples with the Herodians (Herodians favored the Roman authority; Pharisees were opposed to it. Here they come together), saying, “Master, we know that Thou art true and teachest the way of God in truth; neither carest Thou for any man; for Thou regardest not the person of men. Tell us, therefore, what thinkest Thou? Is it lawful to give tribute unto Caesar, or not? But Jesus perceived their wickedness and said, Why tempt ye Me, ye hypocrites? Show Me the tribute money. And they brought unto Him a penny. And He saith unto them, Whose is this image and superscription? They say unto Him, Caesar’s. Then saith He unto them: Render unto Caesar the things that are Caesar’s; and unto God the things that are God’s. When they heard these words they marveled, and left Him and went their way.” Perfect in knowledge of their wickedness and want of conscience, His answer, wise and comprehensive, exposed their sin that had brought upon themselves this chastening of God. They wanted Him to renounce His claim as Messiah. or become an offender against the Roman government. He did neither, but showed them that they had, by their sin, come under this yoke, and now must bear it. Had they rendered to God what was due to Him they would not have been under Caesar. Let them render to God the things that are His, and to Caesar the things which are his (cf. Deut. 28:13, 44).
Matthew 22:23-33. The same day the Sadducees, who profess to believe Moses, but say that there is no resurrection of the dead, came to catch Him on that question. A woman had seven husbands and no children. In the resurrection whose wife shall she be of the seven? For they all had her. “Jesus answered them, Ye do err, not knowing the Scriptures, nor the power of God. For in the resurrection they neither marry, nor are given in marriage, but are as the angels of God in heaven. But as touching the resurrection of the dead, have ye not read that which was spoken unto you by God, saving, I am the God of Abraham, and the God of Isaac, and the God of Jacob? God is not the God of the dead, but of the living.” They are still living though dead, and will be raised again. How different from mere animals? How could the promises of God be fulfilled if the dead rise not? Their infidelity attacked the Word of God and the power of God. Jesus reminded them that what was spoken to Moses was spoken to them. What a proof is here of the abiding character of the Word of God!
Matthew 22:34-40. The multitude are astonished, the Sadducees are silenced, and now the Pharisees gather together. One of them, a lawyer, that is, a teacher of the law of Moses, tempting Him, said: “Master, which is the great commandment in the law?” Jesus said unto him, Thou shalt love the Lord thy God with all thy heart, and with all thy soul, and with all thy mind. This is the first and great commandment. And the second is like unto it, Thou shalt love thy neighbor as thyself. On these two commandments hang all the law and the prophets.” These are the gist or substance of all the law; in keeping these, all are kept. Christ kept it perfectly; yea, went beyond it, and gave Himself for us. Could any sinner do so? No; God’s Word lays bare man’s evil and helplessness. Grace comes in where failure is owned, but that is not law.
Matthew 22:41-46. The Lord now asks them a question which brings out the position He will soon take. Could they understand it? By it they are effectually silenced. “What think ye of Christ? Whose son is He?” They say unto Him, David’s. He saith unto them, How then both David in spirit call Him Lord, saying: The Lord said unto my Lord, Sit Thou on My right hand, till I make Thine enemies Thy footstool? If David then call Him Lord, how is He His son?” Here we see the rejected Christ ascended and sitting at the right hand of God, until God gives Him the kingdom and establishes His throne in Zion. He is seated there now, and now grace is gathering out His heavenly companions. The Pharisees could not understand it, nor could the prophets (1 Peter, 1: 10-12), or they would have understood all the ways of God in His sufferings and glories. And also His glorious person, as the eternal Son of God, David’s Lord, as well as David’s son. It is our blessed portion now to know these things (1 Cor. 2:9-10).
From that time they durst not ask Him any more questions.

Nehemiah: The Building of the Wall, Part 5

Thus we have very briefly examined the seven stages of opposition to God’s own work. The grief of the enemy (Neh. 2:10); the laughter of the enemy (Neh. 2:19); his wrath (Neh. 4:1); mocking (Neh. 4:3); fighting (Neh. 4:8); subtlety without (Neh. 5:1-9); subtlety and danger within (Neh. 6:10). And many a reader of this paper will say, “I have seen all seven in the opposition to God’s work in our own day.”
So the wall was finished. No amount of opposition could stay the work of God. It is so again – saints are gathered to Christ, the wall is built; the doors are set up, and God has raised up faithful men to keep the watch. The position has been assailed in sevenfold opposition; but God has preserved the sacred principle of being gathered to Christ. To Him be all praise! Surely we need to put on the whole armor of God. Our Sanballat is not dead, though his power is destroyed. These seven aspects, that is complete opposition, will continue until the coming of our Lord.
Someone now may say, If God has gathered souls to Christ as at the beginning, and if they find that the truth of the church of God being one, excludes every sect of men – yet, if this basis was large enough at the first to receive every obedient child of God – surely, then, it must be as broad, and be large enough now. Is it not a wonderful truth, that all believers form the one body of Christ – all are one? “There is one body.” And then if Christ has His place in the administration of the church, its gifts, and its worship in spirit, as at first, surely this is a large place to dwell in! Is it not large enough for every Christian on earth who desires to walk in the fear of the Lord, and according to His Word? When this truth is known, what need for all the sects that men have made? Surely no need. Then tell me, if the place is so large and so blessed, how is it that there are so few in it? Why, in some towns, there are none gathered thus to Christ, and in others, those thus gathered are in no reputation.
This was the case also at Jerusalem. “Now the city was large and great; but the people were few therein, and the houses were not builded.” Yes, this is the very question of Nehemiah 7. Compared with the largeness of the city, there were but few in it; but the number was known, and left on record, of those who had come up out of the captivity (Neh. 7:6-60). But there were a great number which went up “from Telmelah, Telharesha, Cherub, Addon, and Burner, but they could not show their father’s house, nor their seed, whether they were of Israel.” Many others also are named: “These sought their register among those that were reckoned by genealogy, but it was not found: therefore were they as polluted put from the priesthood. And the Tirshatha (or governor) said unto them, that they should not eat of the most holy things till there stood up a priest with Urim and Thummim” (Neh. 7:61-65).
All this is exceedingly instructive. If mingling with the Gentile world had caused the Israelites to lose the certainty of their nationality, is there any wonder that the effect of the church being mixed with the world should have caused so many to be uncertain, whether they are the saved children of God or not? Even with the most evangelical there is much darkness and perplexity as to this. And this is one cause, if not the chief one, why so few take the happy place of the children of God gathered to Christ. Evidently there were many Israelites who could not show their genealogy; and there are many Christians who cannot show it; they are so confused with the false position they are in, that they cannot tell whether their names are written in heaven or not. Indeed, in human churches this is not an essential point. Until lately, many denied the possibility of any knowing with certainty that they are the children of God.
Is it not also most true, that if we do not know we are saved, we cannot eat of the most holy things? We must know Jesus, the Great High Priest in the presence of God – He who once bore our sins on the cross, but who is now crowned with glory. As our righteousness, raised from the dead, we now see Him, with Urim and Thummim. In His face shine the lights (Urim) and perfections (Thummim) of God. How can you enter the holiest by the blood of Jesus if you are uncertain whether you are saved? O, search the register; never rest, my reader, until this first question is solved.
Reader, ask yourself, Is my name written in heaven? How do I know that my very sins are all forever blotted out? Is it true that God in very deed is my Justifier? What, shall nothing ever separate me from His love in Christ? If I die, am I quite sure it will be to depart and be with Christ? If I live until the Lord comes am I quite certain that He will take me to be forever with Himself? Reader, you will never answer these solemn questions by looking within, at self, at feelings, or experiences. No, it must be the look of faith at the one who has been lifted up, and is now at the right hand of God. And, mark, I rarely ever met a soul yet that enjoyed this blessed certainty – that truly had peace with God – that could comfortably remain in the camp of Christendom away from Christ in rejection. Now is it not so? Do you not feel it far more consistent, if in uncertainty, to remain in the systems of men, rather than take a place outside the camp, bearing, the present reproach of Christ? I have no doubt this will soon be the real condition of the recent converts. Left in the camp, the uncertainty of the camp will fall upon them. The remnant were few in number, and feeble indeed; and so of those gathered to Christ in this day. But the one was the work of God, and so is the other.
(Continued from page 107)
(To be continued).

What God Hath Said on the Second Coming of Christ and the End of the Present Age: Part 5

Perhaps my reader may ask, Does not the Lord mean death, when He thus speaks of His coming again? If we turn now to John 21:18-21, we here find the distinct answer to the question. The Lord plainly did not mean death; for, after speaking of the death, whereby Peter should glorify God, He speaks of another disciple and says, “If I will that he tarry till I come, what is that to thee?” “Then went this saying abroad among the brethren, that that disciple should not die; yet Jesus said not unto him, He shall not die; but, “If I will that he tarry till I come, what is that to thee?” Now, does not this passage prove that Jesus did not mean death, when He spake of His coming again? Indeed, I do not know of a single passage in the Scriptures where the coming of Christ means death. It is, indeed, very blessed that when the believer falls asleep, it is to depart and be with Christ, which is far better. Far better to depart from a body of sin and death; but this is quite a different thing from the coming of the Lord.
I would observe, the Lord did not in this precious promise, in John 14, explain how this receiving them to Himself, would take place. The explanation how, we shall find in the epistles.
Having found by the words of Jesus that the coming of the Lord does not mean death, it may be asked, Is it as clearly proved, whether the coming of the Lord will be spiritual or personal? Let us for this purpose turn now to Acts 1:9-11. “And when He had spoken these things, while they beheld, He was taken up and a cloud received Him out of their sight.
And while they looked steadfastly toward heaven, as He went up, behold, two men stood by them in white apparel, which also said, Ye men of Galilee, why stand ye gazing up into heaven? This same Jesus, which is taken up from you into heaven, shall so come in like manner as ye have seen Him go into heaven.”
How could words be more plain than these? Did Jesus go into heaven in real person, the very body that hung on the cross; or did He leave that body in the grave and go to heaven in Spirit only? If so, our preaching is vain, and ye have believed in vain (1 Cor. 15). All depends on this, if He who died on the cross as our substitute, is not raised from the dead, and ascended to heaven, a real man, as our surety man in resurrection – then if He is not thus risen, there is no gospel for us. How can I possibly know that I am justified from all things, if my surety is not raised from the death due to me, and as my surety, justified? I fear there is a sort of indistinct notion abroad that Jesus is only a spirit. This notion undermines the very foundation of the whole gospel. Hence, what pains Jesus took to convince His disciples that He was not a spirit; for, says He, “A spirit hath not flesh and bones, as ye see Me have.” Now does not the idea of a spiritual coming of Christ spring from that deadly error, that He is now only a spirit? He went to heaven and is there, a real, risen man. And IN LIKE MANNER will He come again, as real a person surely as He was on the cross. Will not the Jews say, What ARE those wounds on Thy hands? And, O, my fellow-believer, what will it be to look at those hands that were pierced for you? He is risen. He will come again in person.
But it might be asked, Can you point out a passage that distinctly proves whether the Lord will come at the beginning, or after the Millennium, or the times of blessing promised in the Old Testament? Let us turn and see as to this.
Acts 3:19-21. The Jews through blindness had crucified the Lord. Peter tells Them to change their minds, and their sins shall be blotted out; and God shall send Jesus again, “whom the heavens must receive UNTIL the times of restitution of all things, which God hath spoken, by the mouth of all His holy prophets, since the world began.” Does not the word “until” in the above passage distinctly prove that Christ will be in heaven until the beginning of the millennium, or times of restitution of all things? Then He will surely come before the commencement of the millennial kingdom on earth. This one passage removes every difficulty from the teachings of Christ in the gospel. It might have been asked, How can the wicked and the righteous live together until the harvest or coming of Christ? And how can it be, in that day, as it was in the days of Noah and Lot – the earth full of wickedness – seeing that so many scriptures of the prophets have to be fulfilled, which describe the righteousness and blessing of the earth – when all shall know the Lord, from the least to the greatest? Well, I say, this one word “until” explains it all. That time of the earth’s blessedness cannot take place before, but after, the Lord comes.
Let us now, in the second place, hear the words of the Holy Spirit in the epistles. The first passage I turn to is Romans 8:19-23. We must mark well the change; it is not now Jesus speaking to Jewish disciples, in the midst of the Jewish nation; but the Spirit of God speaking to us believers, members of the redeemed church of God, so that now every word concerns us. Hence, in this passage, creation is waiting, with earnest expectation, “for the manifestation of the sons (not the nation) of God.” “The glorious liberty of the children of God.” Creation groans and travails in pain together. “And not only they, but ourselves also, which have the first fruits of the Spirit, even we groan within ourselves, waiting for the adoption, to-wit, the redemption of our body.” We do not wait for the spiritual reign of Christ, or for Christ in spirit; we have that now. “Now, if any man have not the Spirit of Christ he is none of His” (Ver. 9). But having this, according to this passage, we wait, not for the death of the body, but for the redemption of the body – being justified – having peace with God. (Rom. 5:1). Enjoying the certainty of no condemnation (Rom. 8:1). Yes, being thus everlastingly saved, still, while in this body of sin and death, we hope and long for, and wait for, the “manifestation” – “the glorious liberty” that will take place, both for us and creation, at the redemption of the body.
(Continued from page 111).
(To be continued)

Forward

Onward and upward, and forward today!
Onward! renewing thy strength in the way;
Upward! though rugged and steep be the hill;
Forward! the word of the Lord to fulfill.
Onward, and upward, and homeward the while!
Upward! thy sunlight the Savior’s own smile;
Onward! His presence thy shade from the heat;
Homeward! for home after exile is sweet.
Onward, and upward; be strong in the Lord!
He is thy Shield, thine Eternal Reward;
He is gone forward, thy place to prepare;
Homeward thou’rt going to dwell with Him there.
Onward, and upward! the call is divine;
Rich is thy portion, for Jesus is thine.
Loved of the Lord, with ineffable love,
Bright is thy future – the glory above.

Correspondence: 1 Peter 4:18; Acts 2:5

Question 97 “If the righteous scarcely be saved,” does this mean that the righteous may be lost? 1 Peter 4:18 J. W.
Answer: We have salvation presented in three aspects in Scripture. We need always to examine the context, to tell which way the Word applies it.
1st. We have salvation as a present possession, and this can never be altered. (See Eph. 2:8 7 2 Tim. 1:9).
2nd. We are looking on to the Lord’s coming, when our bodies will be changed; then full salvation is reached. It is called the day of redemption (Rom. 8:23; Eph. 4:30; Rom. 13:11; Phil. 3:20, 21; 1 Thess. 5:9, 10; 1 Peter 1:5).
3rd. We are to work out our own salvation with fear and trembling, for it is God that worketh in us both to will and to do of His good pleasure (Phil. 2:12-13). It is to this class 1 Peter 4:18 belongs. It means saved through difficulties along the path, under the government of God. Our failures do not affect our eternal salvation, for it is secured by Christ (John 14:19). The Jews looked for temperal deliverances; we look for soul salvation in our trials. Not kept from them but kept during them. Receiving the end of our faith, soul salvation (1 Peter 1:9, Luke 21:19). We can count on the Lord to keep our souls in peace and victory for Him (Phil. 1:19-20).
Nothing, can touch the life that is hid with Christ in God (Col. 3; also Phil. 1:6); but we need daily help and guidance, and carefulness not to hinder or grieve the Holy Spirit. This is the fear and trembling lest we should hinder God’s work in us.
Question 98: In Acts 2:5, are the nations and Jews different? W. I.
Answer: Yes, Nations, Gentiles and Heathen are the same. Israel or the Jews are not reckoned with the Nations (Num. 23:9; Deut. 32:8).
But at this time (Acts 2) there were Jews scattered among the nations. Many of these pious Jews had come up to Jerusalem to worship and were present to hear the gospel preached for the first time.
We could not call the Eunuch of Ethiopia a Gentile, but a Jewish proselyte, who also came to Jerusalem to worship. (Acts 8:27). Those who received Christ, were henceforth Christians.

Inspiration of the Scriptures: The Bible - its Perfection, Part 1

Let our thoughts now turn for a moment toward another ancient city – Tyre. It was a maritime city, and its prosperity, riches, pride, and costliness have been abundantly described by the prophet Ezekiel; and Zechariah has also spoken of the awful doom that then awaited it from the hand of God. Hiram, king of Tyre, was well known in David’s and Solomon’s days, for from him they obtained much of the material for their buildings at Jerusalem. From all accounts, Tyre was a large and magnificent city, with a profusion of wealth, and all its usual accompaniments of vice and ungodliness; and it is said to have had all the chief merchandise of India and other countries. So elegant was the city in her own esteem, that the inspired penman describes’ her as saying, “I am of perfect beauty,” and adds. “All thy men of war that are in thee; and all thy company, which is in the midst of thee, shall fall into the midst of the seas in the (lay of thy ruin... what city is like Tyrus, like the destroyed in the midst of the sea?.... The merchants among the people shall hiss at thee; thou shalt be a terror, and never shalt be any more.... By thy great wisdom, and by thy traffic, hast thou increased thy riches, and thine heart is lifted up because of thy riches. Therefore thus saith the Lord God, Because thou halt set thine heart as the heart of God; behold, therefore, I will bring strangers upon thee, the terrible of the nations; and they shall draw their swords against the beauty of thy wisdom, and they shall defile thy brightness.... They shall bring thee down to the pit, and thou shalt die the deaths of them that are slain in the midst of the seas.’ (Ezek. 27:3,27,32,36; 28:5,8). Another prophet describes this city, saying, “Tyrus did build herself a stronghold, and heaped up silver as the dust, and fine gold as the mire of the streets. Behold the Lord will cast her out, and he will smite her power in the sea; and she shall be devoured with fire” (Zech. 9:3-4). Referring to her fall and terrible doom, Ezekiel further said. “Thus saith the Lord God to Tyrus the sound of thy fall .... the princes of the sea shall come down from their thrones, and lay away their robes, and put off their broidered garments; they shall clothe themselves with trembling; they shall sit upon the ground, and shall tremble at every moment, and be astonished at thee. And they shall take up a lamentation for thee, and say to thee, How art thou destroyed that wast inhabited of seafaring men, the renowned city, which was strong in the sea, she and her inhabitants, which cause their terror to be on all that haunt it! .... Thus saith the Lord God, When I shall make thee a desolate city, like the cities that are not inhabited.. I will make thee a terror, and thou shalt be no more„ though thou be sought for, yet shalt thou never be found again.” And in the beginning of the same chapter we read, “Thus saith the Lord God, Behold I am against thee, O Tyrus, and will cause many nations to come up against thee, as the sea causeth his waves to come up. And they shall destroy the walls of Tyrus, and break down her towers; I will also scrape her dust from her, and make her like the top of a rock. It shall be a place for the spreading of nets in the midst of the sea; for I have spoken it, saith the Lord God” (Ezek. 26).
From modern travelers we learn how truly literal has been the accomplishments of these most solemn prophetic utterances. They have described the town as “environed by rocks, on the ledges of which are scattered the fragments of ancient columns.” They have expressed their conviction, that the waves of the sea now roll where once stood the vast and magnificent palaces of Tyrian wealth and luxury; and that the monuments of commercial enterprise and prosperity have been overwhelmed by the storm of divine indignation, and are as if they had never been. Another traveler says, he found it “a mere Babel of broken walls, pillars, vaults, etc., there being not so much as one entire house left! Its present inhabitants are only a few poor wretches harboring themselves in the vaults, and subsisting chiefly by fishing; who seem to be preserved in the place by divine Providence, as a visible argument how God has fulfilled His Word concerning Tyre, namely, that it should be “like the top of a rock; thou shalt be a place to spread nets. upon.” To this hour all has been accomplished according to the Word of the Lord God, and as to the future, He has declared, thou shalt be built no more” (Ezek. 26:14). We cannot forbear quoting also a few sentences of a Mr. Hardy on this solemn matter. He says, speaking of the scenes of joyousness and of wealth that have been exhibited on these shores, “They have passed away, like the feverish dream of a disturbed sleep. Ships may be seen at a distance; no merchant of the earth ever enters the name of Tyre upon his books; and where thousands once assembled in pomp and pride; I could discover only a few children and a party of Turks. It was impossible,” says the writer, “not to think of another people, still more favored in their privileges, and whose commercial transactions are as extended as the world. Cities of my country! Shall it ever be said of you, that ye are no more? The patriot may sing exultingly over the achievements of his country, but the Christian will fear and tremble, and offer up prayer to God that what we deserve in justice may be withheld from us in mercy.”
Babylon was another ancient city, and unsurpassed for its beauty and magnificence. It has been described as “a square of about fifteen miles on each side.” The reader will remember that ambassadors were sent from it to Hezekiah to honor him on his recovery from sickness. The wall surrounding the city is said to have been three hundred and fifty feet high, and eighty-seven feet thick. The city was surrounded, too, with a very capacious ditch, which was kept full by the river Euphrates; so that it seemed with all this, and its many towers and gates of brass, to be quite impregnable; and so it was in man’s account; but when God speaks all things are possible to Him, and this many men do not think of. Its palaces, hanging gardens, and wealth showed a profusion of luxury and of human achievement far beyond anything that has been known since. But pride, and idolatry, and vice, after long patience, with its gross immoralities and idol worship, notwithstanding the testimony of Daniel and his associates, called for God’s judicial interference. The testimony of an inspired prophet was, “And Babylon, the glory of kingdoms, the beauty of the Chaldees’ excellency, shall be as when God overthrew Sodom and Gomorrah. It shall never be inhabited, neither shall it be dwelt in from generation to generation; neither shall the Arabian pitch tent there; neither shall the shepherds make their fold there; but wild beasts of the desert shall lie there, and their houses shall be full of doleful creatures; and owls shall dwell there, and satyrs shall dance there. And the wild beasts of the islands shall cry in their desolate houses, and dragons in their pleasant palaces.... For I will rise up against them, saith Jehovah of hosts, and cut off from Babylon the name, and remnant, and son, and nephew, saith. Jehovah. I will also make it a possession for the bittern, and pools of water; and I will sweep it with the besom of destruction, saith Jehovah of hosts. Jehovah of hosts hath sworn, saying, Surely as I have thought, so shall it come to pass; and as I have purposed, so shall it stand” (Isa. 13:19-22; 14:22,24). Where is now this magnificent city, the glory of kingdoms? Have its infidel maxims stood, or has God’s Word been fulfilled? Alas! alas! travelers tell us it is a mass of dust and barrenness, with heaps of bricks and broken pillars, instead of, as it once was, the fruitful valley of Shinar. So full is it of venomous creatures that no one is safe to approach it within a mile and a half, except for about two months in the year, when these animals never leave their holes.
What appalling facts are these we have thus far noticed in the history of some of the greatest cities that ever existed in the world, and how truly has the Word of Jehovah been fulfilled! Well is it for those who so believe God’s testimony as to tremble at His Word!
(Continued from page 117).
(To be continued).

The Unfailing Word of Prophecy

While Dr. Hamlin was in Constantinople, soon after the Crimean War, a colonel in the Turkish army called to see him and said: “I want to ask you one question. What proof can you give me that the Bible is what you claim it to be – the Word of God?”
Dr. Hamlin evaded the question and drew the officer into conversation, during which he learned that he had traveled a great deal, especially in the Fast in the region of the Euphrates.
“Were you ever in Babylon?” asked the doctor.
“Yes, and that reminds me of a curious experience I had there,” replied the visitor, who then related the following account of his visit to the ancient capital of the world: “I am very fond of sport, and having heard that the ruins of Babylon abounded in game, I determined to go there for a week’s shooting. Knowing that it was not considered safe for a man to be there except in the company of several others – and money being no object to me – I engaged a sheik with his followers to accompany me for a large sum. We reached Babylon and pitched our tents. A little before sundown I took my gun and strolled out to have a look around. The holes and caverns among the mounds that cover the ruins are infested with game, which, however, is rarely seen except at night. I caught sight of one or two animals in the distance, and then turned my steps toward our encampment, intending to begin my sport as soon as the sun had set. What was my surprise to find the men striking the tents. I went to the sheik and protested most strongly. I had engaged him for a week and was paying him most handsomely, and here he was starting off before our contract had scarcely begun.
“Nothing I could say, however, would induce him to remain. ‘It isn’t safe,’ he said, ‘no mortal flesh dare stay here after sunset. In the dark, ghosts, goblins, ghouls, and all sorts of things and out of the holes and caverns, and whoever is found here is taken off by them and becomes one of themselves.’ Finding I could not persuade him, I said, ‘Well, as it is, I’m paying you more than I ought to, but if you’ll stay I’ll double it.’ No,’ he said, ‘I couldn’t stay for all the money in the world. No mortal flesh has ever seen the sun go down on Babylon and lived to tell the tale. But I want to do what is right by you. We’ll go off to a place about an hour distant and come back at daybreak.’ And go they did, and my sport had to be given up.”
“As soon as he had finished,” said Dr. Hamiin, “I took my Bible and read from the 13th chapter of Isaiah: ‘And Babylon, the glory of kingdoms, the beauty of the Chaldees’ excellency, shall be as when God overthrew Sodom and Gomorrah. It shall never be inhabited, neither shall the Arabian pitch tent there; ‘neither shall the shepherds make their fold there; but wild beasts of the desert shall lie there; and their houses shall be full of doleful creatures; and owls shall dwell there, and satyrs shall dance there. And the wild beasts, of the islands shall cry in their, desolate houses, and dragons in their pleasant palaces; and her time is near to come, and her days shall not be prolonged.”
“That’s it exactly.” said the Turk when I had finished, “but that’s history you have been reading.”
“No, it’s prophecy. Come, you’re an educated man. You know that the Old Testament was translated into Greek about 300 years before Christ.” He acknowledged that it was.
“And the Hebrew was given at least 200 years before that?”
“Yes!”
“Well, wasn’t this written When Babylon was in its glory, and isn’t it prophecy?”
“I’m not prepared to give you an answer now,” he replied, “I must have time to think it over.”
“Very well, do so, and come back when you’re ready and give me your answer.”
“From that day to this I have never seen him,” continued the doctor, “but what an unexpected testimony to the truth of the Bible in regard to the fulfillment of prophecy did that Turkish officer give.”

Practical Conversations With Our Young People

On account of many of the young having allowed varied lines in life to be the means of turning aside from the Lord’s path, and education being one of these, we give four questions on that subject. We trust the answers from God’s Word may be used for exercising the conscience and impressing upon each one their proper use in this connection.
The object of the fifth question is to acquaint each of our young readers with scriptures on fundamental truths.

Questions for June

Always give Scripture for answers, as well as your own expressions.
1. How did Paul and Daniel use their education for God’s glory?
2. How does James tell a wise man to act?
3. What was Timothy told to avoid?
4. What is the wisdom from above?
5. Give five scriptures showing that all are under condemnation.
ANSWERS TO QUESTIONS FOR APRIL.
We are so pleased to receive such a number of answers from our young people, as it shows real searching of the Scriptures. Nearly all of the answers are very good, but on account of lack of space, we are unable to give as many as we would like to.
We trust the interest will increase in this work, as it is bound to result in blessing to each one of our souls, and to God’s glory.
ANSWERS TO QUESTION 1.
(a) The things essential to Christianity are, “Repentance toward God and faith toward our Lord Jesus Christ” (Acts 20:21).
(b) “Believe on the Lord Jesus Christ, and thou shalt be saved” (Acts 16:31).
(c) “This is His commandment, That we should believe on the name of His Son Jesus Christ, and love one another” (1 John 3:23).
(d) “Ye are all the children of God by faith in Christ Jesus” (Gal. 3:26).
ANSWERS TO QUESTION 2.
(a) “Be ye not unequally yoked together with unbelievers: for what fellowship hath righteousness with unrighteousness? and what communion hath light with darkness?” (2 Cor. 6:14).
(b) “Know ye not that the friendship of this world is enmity with God? Whosoever therefore will be a friend of the world, is the enemy of God” (James 4:4).
(c) “Have no fellowship with the unfruitful works of darkness, but rather reprove them” (Eph. 5:11).
(d) “Wherefore come out from among them, and be ye separate, saith the Lord, and touch not the unclean thing; and I will receive you, and will be a Father unto you, and ye shall be My sons and daughters, saith the Lord Almighty” (2 Cor. 6:17-18).
(e) “By faith Moses, when he was come to years, refused to be called the son of Pharaoh’s daughter; choosing rather to suffer affliction with the people of God, than to enjoy the pleasures of sin for a season; esteeming the reproach of Christ greater riches than the treasures in Egypt: for he had respect unto the recompence of reward” (Heb. 11:24-26).
ANSWERS TO QUESTION 3.
(a) “Seek ye first the kingdom of God, and His righteousness; and all these things shall be added unto you” (Matt. 5:33).
If we attend to the things of Christ, He will take care of us.
(b) “Who gave Himself for our sins, that He might deliver us from this present evil world” (Gal. 1:4).
(c) “Love not the world, neither the things that are in the world” (1 John 2:15).
(d) “Be not conformed to this world: but be ye transformed by the renewing of your mind” (Rom. 12:2).
(e) “Set your affection on things above, not on things on the earth” (Col. 3:2).
ANSWERS TO QUESTION 4
(a) “Let your light so shine before men, that they may see your good works, and glorify your Father which is in heaven” (Matt. 5:16).
(b) “By their fruits ye shall know them” (Matt. 6:20).
(c) “I will show thee my faith by my works” (James 2:18).
(d) “Having your conversation honest among the gentiles: that, whereas they speak against you as evildoers, they may by your good works, which they shall behold, glorify God in the day of visitation” (1 Peter 2:12).
(e) “Put on therefore, as the elect of God, holy and beloved, bowels of mercies, kindness, humbleness of mind, meekness, longsuffering” (Col. 3:12).
(f) “In all things showing thyself a pattern of good works... Sound speech, that cannot be condemned... For the grace of God that bringeth salvation hath appeared to all men. Teaching us that, denying ungodliness and worldly lusts, we should live soberly, righteously, and godly, in this present world” (Titus 2:7-12).

Truths for Young Christians: Undoubting Faith in God

“And Jesus answering saith unto them, Faith in God” (Mark 11:22).
“Have faith in God,” dear young Christians! Faith for the thing; of the day, faith for the intervention of God in the difficulties and trials of life, and faith in God for the things which concern His own glory. Prayer is too often formal, not from the heart; it lacks faith, and so obtains no answer.
All true Christians have faith in God, in the sense that they believe God gave His Son to die that sinners might live, and they believe Christ, their Savior. We may say that all true Christians have faith in God for their eternal good, even should their faith be at times clouded by doubts, but How few have steady faith in God for their present good!
Indeed, some are quite astonished when they hear what and how God works for those who trust Him for the day – the record reads to them like a fable, or a tale of bygone times.
Let us borrow an illustration concerning faith from our daily life. A faithful father makes a promise to his little child. That father will all anything in his power rather than allow his promise to fail, and thereby his child to doubt his word; he will inconvenience himself in any kind of way, rather than be the means of one single hesitation as to his faithfulness springing up in the mind of his little child. That father feels that he is the guardian of his child’s confidence, and he knows well that absolute trust in his word is of the utmost importance for the present moral well-being, and for the future of his child. The character of the father thus becomes the ground of the confidence of the child, who thinks thus: “My father will do what he says,” not, “Can my father do what he has promised?” nor, “Will he do it?”
Our faith in God depends greatly upon our acquaintance with Him.
If we walk with God, we shall learn daily to walk in faith. “Walking” is the general demeanor and conduct of the soul, and is governed by what governs the heart. “Have faith in God.” which is an exhortation of our dear Lord to His followers, leads us right unto God Himself in His absolute faithfulness. We cannot have faith in one of whom we know nothing, and we have small faith in those of whom we know but little but God has made Himself fully known to us in and by His Son, and Jesus bids us have faith in God, who is our Father.
If any of our readers should ask, “Where shall I draw the line, and cease bringing things to God?” and many do so inquire, thinking that only great things may be brought to Him, and that we must not, as it were, intrude our trifles upon The Almighty – we reply, “Be like a little child who has no reserve whatever in his heart, and who brings all his cares and pleasures to his father.” A little child will run to his father with a cut finger, or a broken toy, and the father is only too pleased to listen to the little one’s troubles expressed in speech almost too infantile to be clearly understood.
We want to be more child-like with our God and Father, dear Christian reader, and to bring everything to Him, and to have faith in Him for all things, all the moments of our life.
Now mark what our Lord says about our hearts as to faith – “Shall not doubt in his heart.” If we doubt in our hearts, we are mistrusting God. A little child would not doubt his faithful father, he would credit him. Those who are most simple in their faith, receive the reward of faith. Faith is a reality, and if we do not doubt in our hearts, but believe that those things which we say to God in respect of the difficulties which are the subject of our prayer shall come to pass, we shall have whatsoever we ask. “Therefore say unto you, What things soever Ye desire when ye pray, believe that ye receive them, and ye shall have them” (Mark 11:24). “Jesus answered and said unto them, Verily I say unto you, If ye have faith, and doubt not, ye shall not only do this which is done to the fig tree, but also if ye shad say unto this mountain, Be thou removed, and be thou cast into the sea; it shall be done. And all things, whatsoever ye shall ask in prayer, believing, ye shall receive” (Matt. 22:21-22).

Scripture Study: Matthew 23

Matthew 23.
The disciples are looked at here as the godly remnant of the Jews. The leaders, the scribes and Pharisees, are seen as teachers of the law. Sitting in Moses’ seat, they were to be obeyed in all they said according to that law, although they were behaving the opposite. The disciples were to recognize all that was of God, but they were also to recognize and avoid the sinful behavior of those leaders.
It is a chapter of denunciation of the wicked ways of those leaders.
Matthew 23:4. They bind heavy burdens on others, but would not move them with one of their fingers. They were but fingerposts pointing the way, but they never went themselves.
Matthew 23:5. Their works and religious dress and outward sanctity were but to be seen of men, and (verses 6, 7) to be honored of men.
Matthew 23:8. But the disciples were not to be called Rabbi, for Christ was their Master; they were brethren.
Matthew 23:9. They were to call no roan father upon earth for One is their Father, who is in heaven. This forbids the religious titles of today. that men take assuming authority above others. Not even Master (teacher), for Christ is your instructor (verse 10).
Matthew 23:11-12. The service of Christ is lowly service. God will exalt the humble, but will humble those who exalt themselves.
Matthew 23:13. “But Woe unto you, scribes and Pharisees, hypocrites!” seven times repeated in this chapter.
Were they not well warned?
They would not go into the blessing God had provided and they hindered others from going in, they shut up the kingdom of heaven against men.
Matthew 23:14. “Woe unto you,” for ye devour widows’ houses and for a pretense make long prayer; therefore ye shall receive the greater judgment.
Matthew 23:15. Great zeal to proselytize, only to make the convert more a child of hell than themselves, if possible!
Matthew 23:16-22. “Woe unto you, blind guides.” Their folly is shown out in making more of their gift than of God himself.
Matthew 23:23-24. They paid great attention to small things and neglected what was weighty and substantial – “judgment, mercy and faith.” Blind guides, straining out the gnat, and drinking down the camel.
Matthew 23:25-28. Clean outside, filthy inside, full of ‘extortion and excess. Begin to cleanse inside first, then the outside will follow; but they were whitewashed sepulchers – outwardly beautiful, within full of death and corruption and of all uncleanness. Outwardly appearing righteous, but within full of hypocrisy and iniquity.
What a difference from one washed white, owning his guilty condition before God, and receiving remission of sins! (Isa. 1:18).
Matthew 23:29-32. They build the tombs of the prophets and decorate the sepulchers of the righteous, and say, “If we had been in the (lays of our fathers, we would not have been partakers with them in the blood of the prophets.” Wherefore ye be witnesses unto yourselves, that ye are the children of them which killed the prophets. Fill ye up then the measure of your fathers. Ye serpents, ye generation of vipers, how can ye escape the damnation of hell? (Gehenna, the lake of fire).
Matthew 23:34. “Wherefore, behold, I send unto you prophets, and wise men, and scribes.” Some of those were the very disciples there at that moment, but spoken of in Jewish terms. This would test those leaders, and the Lord tells beforehand what would happen. “Some of them ye shall kill and crucify; and some of them shall ye scourge in your synagogues, and persecute them from city to city.”
Matthew 23:35-36. Frightful was the amount of guilt that was upon their heads. From Abel downwards they had heard the story of their wicked obduracy, and of God’s long-suffering mercy. Now this long suffering was run out and they were obdurate and rebellious still, and upon their heads would be poured out the accumulated wrath of God. “Verily I say unto you, all these things shall come upon this generation.”
Ecclesiastical assumption of authority, with profession of orthodoxy, has ever been the persecuting power and opposition to truth in every age and country. The Scriptures record it of Israel; the scribes and Pharisees finish the story here. The Lord Himself testifies to it.
Matthew 23:37-39. But with what tender pathos the Lord now pronounces judgment on His beloved city. “O Jerusalem, Jerusalem, thou that killest the prophets, and stonest them which are sent unto thee, how often would I have gathered thy children together, even as a hen gathereth her chickens under her wings and ye would not!” How often as Jehovah had He sent His warnings and entreaties? (Psa. 81:13-16; Isa. 48:18). Now ail was at an end. “Behold, your house is left unto you desolate. For I say unto you, ye shall not see me, henceforth, till ye shall say, Blessed is He that cometh in the name of the Lord” (Psa. 118). They rejected Him who is the Sun of Righteousness, so must lie desolate till repentance is wrought (proposed but still refused in Acts 3). In the appointed time they will as a nation realize their sin, and long for His coming and reign, and rejoice in that day when He comes. “This is the day Jehovah hath made; we will be glad and rejoice in it” (Psa. 118:21-29).

Nehemiah: The Building of the Wall, Part 6

We now come to another very interesting inquiry. And again, as of them, so it is of us. If they were neither to come down from the enclosure of those exclusive walls, and mingle with the craftsmen, nor yet to shut themselves up, what were they to do? If we are not to come down from that blessed. place our God has restored to us the ground of the one body, and the sovereign guidance of the Holy Spirit – if we are not to compromise God’s blessed truth by a truce with the clergy, and what is of man in the movements of the act; and, on the other hand, if we are not to shut ourselves up – then what are we to do?
Nehemiah 8 is an answer to this inquiry. The people are gathered together as one man. And Ezra, the priest, brought the law before the congregation. O, what reading of the book before both men and women, and those that could understand; and what attention to the book! The book, God’s book. And Ezra opened the book. And now what blessing and worship! And what causing the people to understand the book “So they read the book, in the law of God, distinctly, and gave the sense, and caused them to understand the reading” (Neh. 8:8).
This, my brethren, is the work of those separated to Christ, and this is what they have to do. Remember how little real regard there is in the camp for the Word of God. It must be far otherwise with them. They must be men of “the book.” They must open the book; read the book distinctly; make the people understand the book. It is God speaking to us. Then there will be lifting up of hands, and bowing of heads, and worshiping the Lord with faces to the ground. Yes, as the Tirshatha, which is the Holy Spirit, gives us understanding of the precious Word, there will be intelligent delight in the Lord; we joy in God.
But is it to be all for ourselves? O no. “Then he said, Go your way, eat the fat, and drink the sweet, and send portions unto them for whom nothing is prepared: for this day is holy unto our Lord; neither be ye sorry; for the joy of the Lord is your strength” (Neh. 8:10). And all the people did so, because they had understood the words.
It is a great mistake merely to seek our own personal blessing and edification. It is spiritual selfishness. We must be personal; we must feed on all the sweet perfections of Christ, that which the fat of the burnt offering pointed to – the inmost thoughts and affections of our own precious Lord; the loveliness of His walk here below; and His present unchanging love. Does not the sweet perfume of His adorable person fill the heaven of heavens? O, let us drink the sweet; let us be full of Christ! And then our happy work is to send portions to those for whom nothing is prepared. O child of God, this is to be thy constant work, even to those who do not understand thee, yea, who slander thee, who misrepresent thee, and who speak all manner of evil of thee ignorantly. Do not return evil for evil, railing for railing, but contrariwise, seek the spiritual good of all; “send portions to the whole church of God. Remember how the Lord met the mad persecutor, Saul of Tarsus. And not a few in our day who were bitter opponents, have been taught of the Father to come out of the camp to the Lord Jesus Christ, the true center. Let what is pleasing to Him be pleasing to us. “For the joy of the Lord is your strength.”
Now we will notice one striking effect of reading the book, and understanding the words that were declared unto them. On the second day gathering (Chapter 8:13-18), they found what was written concerning the feast of tabernacles – “That the children of Israel should dwell in booths in the feast of the seventh month.” “And all the congregation of them that were come again out of the captivity made booths, and sat under the booths: for since the days of Joshua, the son of Nun, unto that day, had not the children of Israel done so. And there was very great gladness.” Is not this very remarkable? They were only a handful of people compared with Israel in the days of Solomon; yet this feast had never been so kept. This feast, Israel in booths, was a beautiful symbol of the people waiting for the millennial reign of their long-expected Messiah and Lord. And for a thousand years Israel had never so waited in booths, as this feeble remnant now waited with “very great gladness.”
It is no less remarkable that the church had never kept the feast of tabernacles since the days of Paul, until God has in these our days gathered a feeble remnant outside the camp to Christ. This was the attitude of the church in the early days of Paul: “Turned to God from idols, to serve the living and true God, and to wait for His Son from heaven,” (1 Thess. 1:9; 2:19; 3:13; 4:15-18). Read prayerfully these scriptures). Must we not confess that for eighteen centuries we look in vain in what is called church history to find the church in this tabernacle feast again? No doubt there was a little of it during the sad days of persecution. But no sooner did the world cease to persecute than the church immediately became worldly – in the world and of the world. And while the bridegroom tarried for so many centuries, the church slept.
(Continued from page 134).
(To be continued)

What God Hath Said on the Second Coming of Christ and the End of the Present Age: Part 6

When, then, will this glorious redemption of the body take place?
1 Corinthians 15:23-25. “But every man in his own order: Christ the first-fruits; afterward they that are Christ’s at His coming. Then cometh the end.” Most clearly, then, the Spirit of God teaches that the resurrection of the sons of God, they who are Christ’s will take place at His coming. And for this event all believers waited at Corinth, as well as at Rome. “Waiting for the coming of our Lord Jesus Christ” (1 Cor. 1:7). Mark, it does not say all shall rise together; but “every man in his own order.” Christ has risen – the first-fruits. Blessed pledge of certainty! Then “afterward.” Who would have thought 1800 years were in that word “afterward”? Now, if there have been 1800 years, at least, between the resurrection of Christ, the Head, and the body – they that are His, may there net, as assuredly there will be 1000 years between the resurrection of the saved, the first resurrect; on, and the rest of the dead, who live not, again until the 1000 years are fulfilled? (Rev. 20).
It may be asked, But how does the resurrection of the dead in Christ at His coming, affect the question of the redemption of our poor groaning bodies who are alive in them, seeing we are not vet fallen asleep? How can we and all believers be waiting for the redemption of the body at the coming of Christ? As to that, “Behold I show you a mystery; we shall not all sleep, but we shall all be changed, in a moment, in the twinkling of an eve,” (1 Cor. 15:51-52). To this agree the words of the Spirit (1 Thess. 4), on which I hope to speak shortly. Now, as we go through the epistles, we shall find, it was for this very event that all believers, in all places, in the days of the apostles waited. Not for the unclothed state of the soul, blessed as that is. The apostle says, “Not for that we would be unclothed, but clothed upon, that mortality might be swallowed up of life” (2 Cor. 5:4).
The Epistle to the Galatians, being the defense of the blessed truth of justification, this subject is not dwelt upon. Also, as the Ephesians presents, that aspect of the church, as already raised and seated in Christ in heavenly places, of course, the subject of the church’s hope is not introduced. But in the Philippians, where the church is looked at more in the service of the gospel, and pressing forward through a weary world, then this blessed hope, and no other, is distinctly presented, “for our conversation (or citizenship) is in heaven; from whence also we look for the Savior, the Lord Jesus Christ; who shall change our vile body, that it may be fashioned like unto His glorious body, according to the working whereby He is able even to subdue all things unto Himself” (Phil. 3:20-21). Those who know the Greek tell us that this passage should be, “We look for the Lord Jesus Christ as Savior.” O! what a contrast this blessed hope of primitive days. to the modern dread of Jesus as a terrible judge. How sweet it is to a mother’s heart, when she returns from a journey, to see her little child’s longed-for face at the window; it claps its little hands, and would fly through the window to meet her. Surely, no mother would have her child dread her return as a terrible tyrant. When Jesus left His chosen ones on Olivet, He lifted up His hands and blessed them, and as He blessed them He was parted from them. In like manner will He return; while to the rejecting world He comes as a terrible judge. Yet, O sinner, saved by grace, the first sight thou shalt have of Him who loved thee, and washed thee in His own blood, will be with uplifted hands of blessing. O! view Him coming as Savior, to claim thee as His prize, bought with His own blood. In one moment, thy body of humiliation, sorrow and sin, shall be fashioned like unto His glorious body. What a moment! Thy last tear shall be gone. Thou shalt grieve Him no more. Thou shalt sin no more. O, what will it be to see His very face – to hear His voice; that face once wrung with deepest anguish, bearing thy sins on the tree!
Surely the certainly of all this is very precious. “When Christ, who is our life, shall appear, then shall ye also appear with Him in glory” (Col. 3:4).
Is it not strange that the church of God should have so sadly forgotten her blessed hope? While in the apostles’ days it was the immediate hope of the youngest converts, as we find in 1 Thessalonians 1:9-10, “How ye turned to God from idols, to serve the living and true God; and to wait for His Son from heaven, whom He raised from the dead.” Thus we see these young converts (for the church of God at Thessalonica was not more than about a year old) were not waiting either for the conversion of the world, or death and departure to be with the Lord; but for the Son of God from heaven. Indeed, the apostles had no other hope. “For what is our hope, or joy, or crown of rejoicing? Are not even ye in the presence of our Lord Jesus Christ at His coming? (1 Thess. 2:10). Great appearances on earth – swelling the numbers of a society on earth, raising funds, and building elegant (so-called) Christian temples; for these things the apostle had not a thought, much more a hope. His eye was fixed on the appearing of Christ. For this he labored night and day, that he might win souls to Christ; that they might be the crown of his rejoicing in the presence of our Lord Jesus Christ. And yet men say the coming of Christ is not a practical truth. The Lord give us more of this practical waiting for Christ.
(Continued from page 139). (To be Continued).
Thou art coming, loving Savior;
Coming first to claim Thine own.
Thou art coming, faithful Savior,
Thou couldst not abide alone.
In Thy Father’s house in glory,
Sinners saved shall dwell with
Thee; O, the sweetness of the story;
Love’s own record we shall be.

Correspondence: Acts 8; Matt. 28:9 and John 20:17

Question 99: Who is the one to take the place of Philip? (Acts 8).
Who has authority to baptize others?
W. B.
Answer: Christ in glory is the source of all true ministry (Eph. 4:8,11).
Paul was an Apostle, “not of men, neither by man, but by Jesus Christ, and God the Father, who raised Him from the dead” (Gal. 1:1).
Elders and deacons were appointed to these offices by apostolic authority.
All gifts come from Christ in glory.
The servants of Christ, therefore, local or otherwise, who can act on Mark 16:15-16, are by that word authorized to serve the Lord. Philip had to act on the Word alone (Acts 8:4-5).
Question 100: What is the difference between Matthew 28:9, “And they came and held Him by the feet,” and John 20:17, “Touch Me not”?
W. H. W.
Answer: Matthew presents the Messiah, risen from the dead, to His earthly people, the remnant of Israel, beginning anew with Israel, therefore you get no ascension. but He sends out the disciples to the nations, and it ends by His saying, “Lo, I am with you alway, even unto the end of the world” (or age).
In John 20:17-23, we have the blessing of the present time, when the Lord is ascended to His Father and our Father, His God and our God. We know Him by faith up there. We do not know Him after the flesh, or as a man down here. (2 Cor. 5:16). Not as the Messiah on earth, but as the glorified Christ at God’s right hand.

Inspiration of the Scriptures: The Bible - its Perfection, Part 2

But before leaving these examples of the predictions of the Bible, there is another city with which we are rather more familiar, which calls for a few remarks: we mean Jerusalem. A later prophet said, “Jerusalem shall become heaps” (Mic. 3:12); and does not this agree with every description we have of its state ever since its destruction by Titus? Are not our Lord’s words also truly fulfilled, “Behold your house is left unto you desolate”? Are not the stones of the temple so scattered, that there is not one stone upon another which has not been thrown down? Is not Jerusalem still trodden down of the Gentiles? Is it not well known that Arab boys break off pieces of stone from the heaps of scattered materials of the ancient temple to obtain a small gratuity from her visitors? And are we not assured that the treading down of the holy city by the uncircumcised will go on “till the times of the Gentiles be fulfilled”? How surpassingly solemn then is the Word of God, and how earnestly it admonishes us to adopt the motto of a less favored and far less instructed servant of God in a past age, “Believe in Jehovah your God, so shall ye be established; believe His prophets, so shall ye prosper!”
This subject – “The Times of the Gentiles” – now calls for a few remarks as a further and very striking example of the accurate fulfillment of the Word of God by the prophet.
When the last two tribes of Israel were delivered by Jehovah into captivity to the king of Babylon, the sword of God’s rule in the earth was handed over to Nebuchadnezzar, so that “whom he would he slew, and whom he would he kept alive,” and created things were also given unto his hands. The prophet said to him, “Thou art this head of gold” – the power derived in its purest state. The whole course of the times of the Gentiles” was revealed to Daniel by Jehovah, and made known to the king, extending from that moment till the Lord will be revealed from heaven to judge, as the stone cut out without hands.” As our space only admits of a brief glance at this Gentile image as another example of the perfection of Scripture, let it be especially noticed that it consisted of four empires, and is to terminate in ten kingdoms. It has been the ambition of some to have a fifth monarchy; and if it be true that the first Napoleon led an immense army into Russia with this view, it was painfully proved that God’s mind is four empires among Gentiles and not five, and they have all long since been and gone. The head of gold – the Babylonian. The next inferior to this – the Medo-Persian of silver; then the Grecian, of brass; then the Roman strong as iron; then the feet and toes of the image, part of potters’ clay, and part of iron, with which there can be no union, strikingly showing the political conservative and radical elements so manifest unto this day. It is remarkable also, that when the ten toes, or ten kingdoms, are fully developed, then judgment falls upon it. And it is most solemnly true, that for many years past most of the political changes that have taken place on the continent in that part of Europe comprehended in Nebuchadnezzar’s image, have more and more developed these ten kingdoms (Dan. 2).
We refer to this not merely to show how accurately Scripture has been fulfilled, and is still being fulfilled as to the Gentile kingdoms, but because of our Lord’s words, “Jerusalem shall be trodden down of the Gentiles, until the times of the Gentiles be fulfilled:” For when the Lord conies out of heaven with His saints in manifested glory, and every eye shall see Him, and they also who pierced Him, then He will turn away ungodliness from Jacob, be their Savior and Deliverer, and establish them in ‘their own land as their rightful King, and “the Sun of Righteousness with healing in His wings.” The Lord’s promised blessings to Israel will be accompanied with judgment; but when His saints are taken to heavenly glory, to the Father’s house, the translation will be all of divine grace and to them power and blessing without judgment.
(Continued from page 146).
(To be Continued).

The Christian Traveler

Having tarried a few days in a beautiful village of the West, I embarked in a vessel which was crossing one of the great lakes. Three other individuals had taken passage, and night coming on found us waiting for a breeze.
About nine o’clock, as the sails were hoisted, another passenger came on board. When we had cleared the harbor he entered the cabin, and seemed to suppose that he was alone; for we had all retired to our berths. The lamp was burning dimly on the table, but it afforded sufficient light for me to discover that he was young. Seating himself beside it, he drew a book from his pocket and read a few minutes. Suddenly from on deck, was heard the voice of the captain uttering oaths, terrific beyond description. The youth arose, laid his book in the chair, and kneeling beside it, in a low whisper engaged in prayer. I listened attentively, and though his soul seemed to burn within him, I could gather only an occasional word or part of a sentence, such as “mercy,” “dying heathen,” or “sinners.” Presently he seemed in an agony of spirit for these swearers, and could scarcely suppress his voice while pleading with God to have mercy on them. My soul was stirred within me. There was a sacredness in this place, and I was self-condemned, knowing that I also professed the name of Jesus, and had retired with my fellow-passengers to rest, not having spoken of God or committed myself to His care.
Early in the morning I was awakened by a loud voice at the door of the companion-way: “Here, whose tracts are these?” followed by other voices in threats and imprecations against tract-distributers, Bethels, and Temperance Societies.
I thought of the young stranger, and feared they would execute their threats upon him; but he calmly said, “Those tracts, sir, are mine. I have but a few, as you see, but they are very good, and you may take one if you wish. I brought them on board to distribute, but you were all too busy last night.” The sailor smiled and walked away, making no reply.
We were soon called to breakfast with the captain and mate. When we were seated at the table, “Captain,” said our young companion, “as the Lord supplies all our wants, if neither you nor the passengers object, I would like to ask His blessing on our repast.”
“If you please,” replied the captain, with apparent good-will. In a few minutes the cook was on deck, and informed the sailors, who were instantly in an uproar and their mouths filled with curses. The captain attempted to apologize for the profanity of his men, saying “It was perfectly common among sailors, and they meant no harm by it.”
“With your leave, captain,” said the young stranger, “I think we can put an end to it.”
Himself a swearer, and having just apologized for his men, the captain was puzzled for an answer, but after a little hesitation replied, “I might as well attempt to sail against a head-wind as to think of such a thing.”
“But I meant all I said,” added the young man. “Well, if you think it possible, you may try it,” said the captain.
As soon as breakfast was over, the oldest and most profane of the sailors seated himself on the quarterdeck to smoke his pipe. The young man entered into conversation with him, and soon drew from him a history of the adventures of his life. From his boyhood he had followed the ocean. He had been tossed on the billows in many a tempest; had visited several missionary stations in different parts of the world, and gave his testimony to the good effect of missionary efforts among the natives of the Sandwich Islands. Proud of his nautical skill, he at length boasted that he could do anything that could be done by a sailor.
“I doubt it,” said the young man.
“I can,” answered the hardy tar, “and will not be outdone, my word for it.”
“Well, when a sailor passes his word he ought to be believed. I know a sailor who resolved that he would stop swearing, and did so.”
“Ah,” said the old sailor, “you’ve anchored me; I’m fast – but I can do it.”
“I know you can,” said the young man, “and I hope you will anchor all your shipmates’ oaths with yours.”
Not a word of profanity was afterward heard on board the vessel. During the day, as opportunity presented itself, he conversed with each sailor singly on the subject of his soul’s salvation, and gained the hearts of all.
By this time I was much interested in the young stranger, and determined to know more of him. There was nothing prepossessing in his appearance; his dress was plain, his manner unassuming, but his influence had, by the blessing of God, in a few short hours greatly changed the aspect of our crew. The tiger seemed softened to a lamb, and peace and quiet had succeeded confusion and blasphemy.
After supper he requested of the captain the privilege of conducting service in the cabin. His wishes were complied with, and soon all on board, except the man at the helm, were assembled. The captain brought out a Bible, which he said was given him in early life by his father with a request that he would never part with it. We listened as our friend read Matthew’s account of Christ’s crucifixion and resurrection; and then, looking around upon us, he said, “He is risen – yes, Jesus lives; let us worship Him.”
It was a melting scene. Knees that seldom bowed before now knelt at the altar of prayer, while the solemnities of eternity seemed hanging over us. After prayer we went on deck and sang a hymn. It was a happy place, a floating Bethel. Instead of confusion and wrath there was sweet peace and solemnity. We ceased just as the setting sun was flinging upon us his last cheering rays.
“Look yonder!” he exclaimed. “You who have been nursed in the storm and cradled in the tempest, look at the setting sun, and learn a lesson that will make you happy when it shall set to rise no more. As rose that sun, this morning, to afford us light and comfort, so has the Son of God arisen to secure salvation to all who accept and love Him; and as that sun withdraws its beams, and we are veiled in darkness for a season, so will the Sun of Righteousness withdraw His offers of mercy from all who continue to neglect Him. But remember, that season is one that never ends-one dark, perpetual night.”
The captain, deeply affected, went into the cabin, lit his lamp, took his Bible, and was engaged in reading till we had retired to rest. In the morning as soon as we were seated at the breakfast table, the captain invited our friend to ask a blessing. “There, gentlemen,” said he, “this is the first time I ever made such a request; and never till this young man came on board have I been asked for the privilege of holding prayers, though I have a thousand times expected it, both on the ocean and the lake; and have as often, on being disappointed, cursed religion in my heart and believed that it was all delusion. Now I see the influence of the Bible, and though I make no claims to religion myself, I respect it, for my parents were Christians; and though I have never followed their counsels, I cannot forget them.”
After this, for three days, we regularly attended family worship, and had much interesting conversation on various subjects: for there was nothing in the religion of the young stranger to repress the cheerfulness of social intercourse. From his familiarity with the Bible, his readiness in illustrating its truths and presenting its motives, from his fearless but judicious and persevering steps, we concluded that he was a minister of the gospel. From all he saw he gathered laurels to cast at his Master’s feet, and in all his movements aimed to show that eternity was not to be trifled with. A few hours before we arrived in port we ascertained that he was a mechanic.
Before we reached the wharf the captain came forward and with much feeling bade him farewell and declared that he was resolved to live as he had done no longer; his wife, he said, was a Christian, and he meant to go and live with her; and added, “I have had ministers as passengers on my vessel Sabbath days and week days, but never before have been reminded of the family altar where my departed parents knelt.” As we left the vessel every countenance showed that our friend had, by his decided yet mild and Christian faithfulness, won the gratitude of many and the esteem of all.
(To be Continued).

Practical Conversations With Our Young People: Obedience

The normal state of a Christian is to desire to do something for, and to please, Him who has brought us out of nature’s darkness into His marvelous light. In order to do this we must be in communion with our blessed Lord, spending much time in our closets alone with Him, and in meditation on His Word. Then we receive strength and power for our walk and service, as He becomes the object of our hearts, and we rejoice in the fact that “obedience is better than sacrifice, and to hearken than the fat of rams.” “Walk before Me, and be thou perfect,” was the word given to Abram. There was not much about service in that, but it would result to God’s glory. What characterized Abram was, faith and obedience.
To obey with delight of heart for all the Lord has done for us, is what pleases Him above all else, and if He gives us some service to do for Him, it will meet with His approval and get His Well done, good and faithful servant” in that day so near.

Questions for July

Always give Scripture for answers, as well as your own expressions.
1. Should a Christian attend all the different kinds of religious meetings?
2. What is the lesson to be learned from Jer. 15:19?
3. What place does music get in God’s Word?
4. Give five proofs of the inspiration of Scripture.
5. Give five Scriptures that the unsaved are awaiting execution.

Answers to Questions for May

ANSWERS TO QUESTION 1.
Why can we not serve two masters?
(a) “No man can serve two masters; for either he will hate the one, and love the other: or else he will hold to the one, and despise the other. Ye cannot serve God and mammon” (Matt. 6:24).
(b) We cannot serve two earthly masters because one would be almost sure to want us to do something which the other would not want us to do; or, for personal reasons, we might be more anxious to please one more than the other. How much less can we serve God and Satan, whose every thought and purpose are entirely opposed the one to the other.
(c) We cannot be the servants of Christ and the servants of the devil, too.
(d) “For do I now persuade men, or God? or do I seek to please men? for if I yet pleased men, I should not be the servant of Christ” (Gal. 1:10).
(e) “Know ye not, that to whom ye yield yourselves servants to obey, his servants ye are to whom ye obey; whether of sin unto death, or of obedience unto righteousness?” (Rom. 6:16).
ANSWERS TO QUESTION 2.
What kind of fruit does the Lord look for from Christians?
(a) “The fruit of the Spirit is love, joy, peace, long-suffering, gentleness, goodness, faith, meekness, temperance” (Gal. 5:22-23).
(b) “Being filled with the fruits of righteousness, which is by Jesus Christ, unto the glory and praise of God” (Phil. 1:11).
(c) “The fruit of the righteous is a tree of life; and he that winneth souls is wise” (Prov. 11:30).
(d) “Walk in love, as Christ also hath loved us” (Eph. 5:2).
(e) “By Him, therefore, let us offer the sacrifice of praise to God continually, that is, the fruit of our lips giving thanks to His name” (Heb. 13:15).
ANSWERS TO QUESTION 3.
Will faithfulness to the Lord always bring success?
(a) “Be thou faithful unto death, and I will give thee a crown of life” (Rev. 2:10).
(b) “Them that honor Me I will honor” (1 Sam. 2:30).
(c) “Because thou hast kept the word of My patience, I will also keep thee from the hour of temptation, which shall come upon all the world” (Rev. 3:10).
(d) “When a man’s ways please the Lord, He maketh even his enemies to be at peace with him” (Prov. 16:7).
(e) “This book of the law shall not depart out of thy mouth: but thou shalt meditate therein day and night, that thou mayest observe to do according to all that is written ‘therein: for then thou shalt make thy way prosperous, and then thou shalt have good success” (Josh. 1:8).
(f) “Seek ye first the kingdom of God, and His righteousness; and all these things shall be added unto you” (Matt. 6:33).
(g) I take this question to mean earthly success, and do not think it is always so. But. still we know we are kept. “My God shall supply all your need” (Phil. 4:19). Success is often withheld to teach us patience and long-suffering. If we speak of going to glory as success, then indeed are our lives successful. “Lay not up for yourselves treasures upon earth, but lay up for yourselves treasures in heaven” (Matt. 6:19-20).
(h) Faithfulness to the Lord does not always bring success. In Paul’s case it brought the very opposite. He was the most faithful servant, yet no one suffered for the Lord as he did (2 Cor. 11:23-28). In the case of Ruth, faithfulness to God did bring success, from the time she made her choice and took her place among God’s earthly people (Ruth 1:16, 17; 2:14-16; 3:11-18; 4:10).
ANSWERS TO QUESTION 4.
Why should the Lord rather have obedience than mere religious activity?
(a) “Samuel said, Hath the Lord as great delight in burnt offerings and sacrifices, as in obeying the voice of the Lord? Behold, to obey is better than sacrifice, and to hearken than the fat of rams” (1 Sam. 15:22).
(b) Even a master or mistress in the world delights to have an obedient servant, and parents delight in obedient children. If Christ is the object of our hearts, it will be our delight to seek to please Him. It may cost us something, but we shall hear His “Well done.” Martha’s service would have been all right if she had had Christ as her object, but service got between her heart and Christ. “Mary hath chosen that good part” (Luke 10:38-42).
ANSWERS TO QUESTION 5.
Give five scriptures, showing that all have sinned.
Romans 3:23, Romans 3:10, Galatians 3:22, 1 John 1:8,
Romans 5:12, Ecclesiastes 7:20, Psalms 14:3, Romans 11:32,
Genesis 6:5, 1 Kings 8:46.

Truths for Young Christians: Service, Part 1

All that we do for the Lord, whether made effectual to others or not, is service, accounted so by Him, and by-and-by to be rewarded by Him. This is what we need to learn. We have to learn that we are so the Lord’s as to have no right to ourselves.
We often hear the expression “consecrating our talents,” “devoting ourselves,” “giving up ourselves,” forgetting that we do not possess ourselves. We had forfeited all, and Christ redeemed us, but as we had sold ourselves and were the slaves, or servants of Satan, Christ purchased us, that we might be His servants (slaves), absolutely His. This truth enhances the grace of Romans 12:1, 2 that can beseech us to yield our bodies, although we belong to Him, body and soul.
Therefore, whenever I do the least thing to please myself, right or wrong, I am sinning, for I am not my own. When I do anything because it is His will in obedience, it is service. Men may not so reckon it, the flesh in us may despise it saints may say “We do nothing,” Christ says, “You serve Me.”
If, in a large house, a servant is hired to attend only to one bell, she is paid for being in her place, waiting for that bell to ring, all the day long. While she waits ready to attend it she is a good servant, though she may do nothing else. She was hired for that one thing. The Lord wants us to be at His bidding, whether for activity or inactivity. We must learn this, I believe, before the Lord will have much use for us; and then we have to learn also that we are not agents to act for Him, but rather instruments by which He acts; pipes through which He waters. We must learn our nothingness in order that we may be simple, in no way occupied with ourselves, whether in success or failure. Our work is to deliver our message, and we are rewarded for doing our work. The success is not ours to control, and when we learn our nothingness, that we are only the instruments that He takes up as and when He pleases, and for what He pleases, we shall be content to work on simply.
Perhaps you will say, “Of course.” But I assure you that to me all this is new. I have worked with some diligence, but now I see how much self-will dictated. I was a willing, self-dictating agent.
When first I saw this, I was perplexed and sad at heart. How should I ever know what the Lord would have me do? How should I avoid running when not sent? It appeared to me the stream of service was at once cut off. I could not go back to old plans, and I could not see one step before me, but the Lord knew all. and when He had taught me the lesson, in a measure, He used me when I thought not of it, and showed me that life in healthy action necessarily includes all service, and all I had to do was to yield myself as one alive (in a new life which was a gift) and yield my members as instruments (Rom. 6), and He would speak through me, act by me, in a word use me.
I cannot tell you how this has simplified this matter, made me happy in strength or feebleness writing or speaking. Fear of man, and desire of man’s approbation is that which is judged as sin. I used to desire to live to some purpose, to be of some use. Why? “O, for God’s glory,” I would have said. But now I see it was for self. I had not learned my nothingness now and to all eternity. I worked as an individual believer, to be individualized to all eternity, not as a part of Christ. The Lord will fill the vessel, when He has prepared it, will use the tool when it knows itself to be one. Do not be discouraged, trust God and believe the love He has toward you.

Scripture Study: Matthew 24:1-20

Matthew 24:1-2. The Lord Jesus went out, and departed from the temple. He had already pronounced its judgment, but now He carries it out, a solemn act, the house is left desolate; the disciples do not enter into this, but want to show Him its grandeur. The Lord announces that even that shall be utterly destroyed.
Matthew 24:3. As He sat on the Mount of Olives, the disciples came privately, saying, “Tell us, when shall these things be? and what shall be the sign of Thy coming, and of the end of the world (or age)?”
In Luke 21 the Lord leads them on to what follows till Jerusalem was destroyed by Titus. In this chapter, it is told in words that apply to the future period, after the church is caught up to be with the Lord. The disciples are looked at as the godly remnant of Israel, spoken of in the Psalms and Prophets, that will be waiting for Christ as Son of David to reign over Israel’s kingdom.
Matthew 24:4-5. They were to take heed of men who would come as deceivers, saying, “I am Christ; and shall deceive many.” Those who know that Christ is at the right hand of God in heavenly glory, could not be deceived in that way. He will meet us on the cloud in the air (1 Thess. 4:17).
Matthew 24:6-13. They must expect wars and rumors of wars and yet not be disturbed; such things must be, but the end is not seen in them. Nation shall rise against nation, and kingdom against kingdom; with famines, and pestilences, and earthquakes, in divers places. All these are the beginning of sorrows.
All these have happened time after time during the church period, but we must remember there is no prophecy being fulfilled now.
The true disciples will be greatly tried, afflicted, some killed, and generally hated of all nations for My name’s sake. Not only that, but many shall be offended, and shall betray one another, and shall hate one another and many false prophets shall rise, and shall deceive many. And because iniquity abounds, the love of many shall wax cold. But he that endures to the end, the same shall be saved. The true believer endures to the end, then as now. He may grow cold and appear disheartened, but he cannot deny that Jesus is Lord and Christ. He may feel his own weakness and be sadly troubled, but God is faithful to carry him through.
The enduring to the end, here is during the time when all Satan’s power is exerted to get the godly Jews to deny Jesus as the Christ. The end is when He appears, and they are saved into the millennial reign of Christ as His subjects on earth.
Some are put to death during the beginning of sorrows, for “the Word of God and the testimony of Jesus.” And some during the great tribulation, because they refuse the mark of the beast and will not worship his image. They are distinguished in Revelation 20:4 (JND).
Matthew 24:14. And this gospel of the kingdom shall be preached in all the world for a witness unto all nations; and then shall the end come. This is not the gospel that is preached now. It is not the gospel of the grace of God to all alike, Jew and Gentile, repentance toward God and faith toward our Lord Jesus Christ. The gospel of the kingdom tells of the King coming to establish His dominion, and says, “Kiss ye the Son, lest He be angry, and ye perish from the way, when His anger is kindled but a little. Blessed are all they that trust in Him” (Psa. 2:12). It is this gospel that is to be preached to the nations. It demands their subjection (Psa. 110:2,5). It is a witness to all nations, that the Great King is about to claim His rights.
Matthew 24:15-20. But Satan now brings his masterpiece out. In Revelation 12 he is cast out of heaven; in Revelation 13 you have the satanic trinity – the beast, the antichrist or false prophet, and the dragon. The beast, by Satan’s power, sets himself up to be worshiped as the only God. He is the Head of the Roman Empire. The antichrist, king of the Jews, made a covenant that all the Jews were to be allowed to worship in the temple, but now, and suddenly it is declared that every one must worship the beast, whose image is set up in the holy place: this is the abomination spoken of by Daniel the prophet (Dan. 9:27), (whoso readeth let him understand), that is, take particular notice, it is a warning, “Flee to the mountains.” The covenant was a wrong one; it was made with the powers of darkness, and it could not stand (Isa. 28:15,18). Many of the Jews will worship the beast to save their lives, and will be lost forever (Rev. 14:11; Rev. 13:4-8,12-14). What can the godly do but flee?
(To be continued).

Nehemiah: The Building of the Wall, Part 7

Now what has taken place during these last years, since God has gathered a feeble remnant to Christ? Have not the Scriptures had a similar place and effect to that described in our chapter? Has not the effect been the same? The blessed long-lost hope of the church has been restored, and an attitude answering to the feast of tabernacles has been once more taken. The gathered remnant have been led, by the Spirit of God, to wait for the Son from heaven; and there is very great gladness. The blessed certainty, that as it is appointed to men to die, and after death the judgment, so Christ was once offered for our sins; and we. are now looking for Him without sin unto salvation. Yea, “we know that when He shall appear, we shall be like Him; for we shall see Him as He is” (see Heb. 9:27-28; 1 John 3:2), contrasted with the awful gloom of looking for a day of judgment, and the bar of God, about our sins. There is very great gladness; because we know that He has loved us, and washed us from our sins in His own blood; and it is our happy privilege now to be waiting for Him from heaven: O the untold joy of that triumphant moment! “Even so, come, Lord Jesus.”
And as Israel were to publish and proclaim in all their cities what they found written; so surely would the Lord have us make known, with holy boldness, what we have found written. Intelligent communion with God and with one another, understanding the words of God which are written, making all this known to the blood-bought church of God, and waiting for His Son from heaven – what could we have more?
Thus the wall was built. And all this great gladness more than made up for the hatred of men, and charges of exclusivism. I do not pursue this study much beyond the wall, but there is one thing I must notice.
Some of my readers may say, “Surely the result of all this would be self-complacency, conceit, pride. What, you, the only handful of people on the face of the earth on true ground – within God’s sacred enclosure – around the only true Center? This must produce narrow-minded self-satisfaction.”
You are wrong; it does not. Read Nehemiah 9. What a contrast to all human thought! “Israel were assembled with fasting, and with sackcloth, and earth upon them.” It might be thought that separation from others would produce a feeling of self-superiority. But no; it did not. The seed of Israel separated themselves from all strangers, “and stood and confessed their sins, and the iniquities of their fathers.” And then there is reading, confession, and worship. Ah, this is of God: it is the divine order. Separation from evil brings us into self-abhorrence before God. And the more we read His Word, the more we have to confess; and, wondrous to tell, the more we confess, the more we worship. And then you find the Levites cry to God. Self-judgment produces dependence on God, and faith in God. “Stand up, and bless the Lord your God forever and ever; and blessed be Thy glorious name; which is exalted above all blessing and praise.” Thus the Lord Jehovah is before their souls; while owning their utter failure, and the failure of their fathers, yet throughout this chapter, God, in all that He had done and was to them, shines out in every verse.
All this is so true in every case where a soul is truly gathered to Christ. “Mine eye seeth Thee, wherefore I abhor myself, and repent in dust and ashes.” I am sure, the nearer we to God, the more the flesh will be crushed, whether as individuals, or as saints gathered to the Lord. It is not what we are no, we have sinned; but it is what God is, and what He has done for us. Surely deep, real humility becomes those who can say there is nothing between our souls and the lake of fire, but the blood of Christ. To Him be all glory and praise. He is worthy to bring His redeemed, without spot or wrinkle, to His own place prepared for them. “Let us,” then, “go forth therefore unto Him without the camp, bearing His reproach.”
All this is surely truth for present guidance, and for testing. Where are you, reader? in the religious Babylon, afar off from God’s true ground of gathering? or have you, like the remnant, been brought back to the ground of what the church was in the beginning? Have you been exercised before the Lord about the present condition of Christendom, as Nehemiah was about the holy city? Have you found any seeking alone the good of the church of God? Do you know anything of that sevenfold opposition to the present work of God? The grief, the laughter, wrath, mocking, fighting, subtlety without and within, of those who are in the professing church? Have you the certainty that your name is written in heaven? or have you searched, and cannot find your register? Do you know whether you are a child of God, or not? This being settled, have you been led to search the book – to understand the book – to eat the fat, and drink the sweet?
Is it your joy to send portions to them for whom nothing is prepared? Has the searching the Word led you to wait for Christ from heaven? Are you charged with exclusivism, because of that hated wall of separation? And has all this brought you lowly before the Lord in confession, and then worship? And, finally, is God before your soul, as He was before the remnant in chapter 9? Has your soul found the sabbath of rest within the sacred wall, even Christ Himself?
Then beware of the men of Tyre, who will offer their tempting wares before the wall. Keep the gates shut – O, keep the gates closed. Let nothing come in to break your rest in Christ – your joy in God. We need much the lesson of the last chapter to keep the gates shut; it will be most offensive to men of Tyre, but most pleasing to our God. He alone could have given us such a picture of the day in which we live, and He alone could give us such a light for our feet. May He sanctify us by His Word – His Word is truth!
(Continued from page 163).
(Concluded).

What God Hath Said on the Second Coming of Christ and the End of the Present Age: Part 7

If there be one thing more powerful for practical holiness than another set before us in the Word, it is the constant expectation of Christ. This was the prayer of the apostle, for these young converts, night and day. “To the end He may establish your hearts unblamable in holiness, before God, even our Father, at the coming of our Lord Jesus Christ with all His saints” (1 Thess. 3:13). This is the desire of every minister of Christ, who longs himself for the coming of the Lord; while others forget the connection there is between conversion and the coming of Christ. He will think of it night and day, and pray night and day, that every convert may be found unblamable in holiness in that moment.
Some have a very great difficulty as to waiting for Christ. They would say, How can I wait for or expect Christ this very day, seeing so many things have yet to be fulfilled? I am told the Roman Empire has to be reconstructed. The Jews have to be restored to Judea. The man of sin has to be manifested, reigning at Jerusalem – and all this before Christ comes to this earth again. Must I not, then, of necessity, say in my heart, “My Lord delayeth His coming;” at least, until after all these events. I cannot make it out how I am to be waiting for Christ today, since all these things have to take place.
There is no doubt the Roman Empire has to be reconstructed – (I shall have to speak of these things shortly). The Jews have to be restored: the wicked man of sin has to be revealed. Europe has to sink in grossest darkness – to become worshipers of devils or idolaters again. But let us look at the next chapter, and every difficulty will vanish.
1 Thessalonians 4:15-18. “For this we say unto you, by the word of the Lord, that we which are alive and remain unto the coming of the Lord shall not prevent them who are asleep. For the Lord Himself shall descend from heaven with a shout, with the voice of the archangel, and the trump of God: and the dead in Christ shall rise first. Then we which are alive and remain, shall be caught up together with them in the clouds, to meet the Lord in the air; and so shall we ever be with the Lord.” Now, observe, in the gradual unfolding of prophetic truth, up to this passage the coming of the Lord, in a general sense only, has been presented. But in this place there is a point of detail never noticed before. The special object of the apostle was evidently to comfort the hearts of these young converts, who were sorrowing because some of their number had fallen asleep. Bear in mind, they had only had three weeks’ preaching. (Acts 17:2). Paul’s manner was always to set forth the passing away of all things of the old creation, in the death of Jesus; and Jesus the beginning and head of the new creation, as raised from the dead. Thus these young converts were filled with joy – being raised from the dead, and in God the Father. (See Chapter 1). As we may imagine, these newly-converted heathen were sorely perplexed at the death of the body. Well, this passage is evidently written to show them the resurrection of the saints who sleep in Jesus – that they will lose nothing, but be raised first; then we who are alive and remain – changed in a moment, as noticed in 1 Corinthians 15.
And then a new fact is revealed – we shall be caught up together, to meet the Lord in the air. Now this may take place before any of the events which have to be fulfilled; yea, we shall find this event is the very first that will take place; and therefore the last elect soul being brought to God, may take place while you read this paper. No person can quote me a single verse, which has to be fulfilled, before the sleeping and living saints, be caught up to meet Christ.
Now if this be so. we should expect the Spirit of God, having made known this new fact, of believers being caught up to meet Christ, then to speak of this day of the Lord, which shall follow that event, the taking of the church to be ever with the Lord. And this is exactly what He does do (1 Thess. 5). As the believer’s hope is to be caught up to meet Christ, there is no need of writing on times and seasons. In fact there are no dates of Scripture that refer to this event (the moment of taking the saints) at all. All dates refer to Israel. “For yourselves know perfectly, that the day of the Lord so cometh as a thief in the night. For when they shall say, Peace and safety, then sudden destruction cometh upon them, as travail upon a woman with child and ye shall not escape. But ye, brethren, are not in darkness that that day should overtake you as a thief,” etc. Thus this day of destruction and vengeance is introduced in dark contrast with the bright and blessed hope of the church. The church may be taken at any moment. Then sets in the day of vengeance. The acceptable year of the Lord shall close; the day of vengeance begin. (Isa. 61). Seeing this, how earnest the apostle was in prayer: “And I pray God your whole spirit, and soul, and body be preserved blameless unto the coming of our Lord Jesus Christ” (1 Thess. 5:23). We find persecution and tribulations soon overtaking these dear young converts, and to add to their deep distress, deceivers come, as though sent by the apostle, to tell them the day of the Lord was come. Their trouble and sorrow seemed to favor the report. They seemed to have been greatly shaken by this stratagem of Satan.
(Continued from page 167).
(To be continued).

Correspondence: Rom. 8:16 and 1 John 5:10, The Witness of the Spirit

Question 101:
(a) How does the Spirit bear witness with our spirit? (Rom. 8:16).
(b) Where does it (my spirit) get its witness? (1 John 5:10).
To whom do these bear witness?
K. C.
Answer: (a) “The Holy Spirit (acting in us in life) has produced the affections, the consciousness of being a child of God, so He does not separate Himself from this, but, by His powerful presence, He bears witness Himself that we are children. We have this testimony in our hearts in our relationship with God; but the Holy Spirit Himself, as distinct from us, bears His testimony to us in whom He dwells. The true freed Christian knows that his heart recognizes God as Father, but he knows also that the Holy Spirit Himself bears testimony to him. That which is founded in the Word is realized and verified in the heart.”
(J. N. Darby, Synopsis).
(b) 1 John 5:10. The believer gets his assurance from the Word of God; he knows no greater testimony (vs. 9); he rests on it and has the confidence of it in testimony in his heart.
(c) In the above passages, the witness is in and to his own soul.

Inspiration of the Scriptures: The Bible - its Perfection, Part 3

In nothing, perhaps, is the perfect accuracy of Scripture more manifest than in the present state of the children of Israel; not but that the word of Jehovah as to the Gentile nations is also very solemnly being fulfilled, “I will overturn, overturn, overturn it; and it shall be no more, until He come whose right it is” (Ezek. 21:27). But as to Israel, the ten tribes are scattered to the four winds – “outcasts,” no man knows where; a few of them were not carried away in the Assyrian captivity, and that is why we read of Anna, who was of the tribe of Asher, and now and then we meet with one of the remnant of these ten tribes which were left behind. (See 2 Chron. 34:9). But most of the Israelites with whom we have intercourse are of Judah and Benjamin. This distinction is kept up in the prophets, and is important; for our Lord taught, that the ten tribes would not be gathered till He appears in manifested glory. Isaiah remarkably distinguishes them by speaking of the “outcasts of Israel,” and of the “dispersed of Judah,” instead of preserving their own nationality.
But though one here and there repents, receives Christ as his Savior, and openly confesses Him, and is really born anew and saved, “according to the election of grace,” as the apostle tells us, yet, as a people, they are still in unbelief; some are pious Jews, and others openly infidel, yet at this moment fulfilling the words of the prophet Hosea. “The children of Israel shall abide many days without a king, and without a prince, and without a sacrifice, and without an image [or pillar, margin], and without an ephod, and without teraphim.” Thus it is to this day, though they have much of the wealth of the world, the Word of God must be fulfilled, for it endures forever; so the Jews have no king, no prince, no proper sacrifice, no pillar, no high priest with ephod, and no teraphim or idol. How appalling are these facts, and how the eternal verity of the written Word should be endeared to our hearts! It is a serious blunder to suppose that the Jews, as a people, are to be converted by the present ministry of the gospel, for both Old and New Testaments assure us, that it will be by seeing their Savior, and not like us by believing in one whom we have not seen, but for whom we look and wait. Hence Hosea goes on to say, “Afterward shall the children of Israel return, and seek Jehovah their God, and David their king, and shall fear Jehovah and His goodness in the latter days” (Chapter 3:4, 5). Nothing is more plainly stated in Scripture than Israel’s future blessing, and that by the Lord’s personal coming. The prophet Isaiah said, “The Redeemer shall come to Zion, and unto them that turn from transgression in Jacob, saith Jehovah” (Chapter 59:20); and an apostle taught that, “all Israel [that is, all the twelve tribes] shall be saved, as it is written, There shall come out of Sion the Deliverer, and shall turn away ungodliness from Jacob; for this is My covenant unto them, when I shall take away their sins” (Rom. 11:26-27).
The faithfulness of God to His own Word in all these ways is very precious to our souls. A story is published of two Rabbis approaching Jerusalem, who observed a fox running on the hill of Zion, when Rabbi Joshua wept, and Rabbi Eliezer laughed. “Wherefore dost thou laugh?” said he who wept. “Nay, wherefore dost thou weep?” demanded Eliezer. “I weep,” replied the Rabbi Joshua, “because I see what is written in the Lamentations fulfilled, ‘because of the mountain of Zion, which is desolate, the foxes walk upon it.’” (Chapter 5:18). “And therefore,” said Rabbi Eliezer, “do I laugh, for when I see with mine own eyes that God has fulfilled His threatenings to the very letter, I have thereby a pledge that not one of His promises shall fail, for He is ever more ready to show mercy than judgment.”
In nothing, however, is the perfection of the Scriptures more strikingly seen than in the use of the divine titles. As we have before noticed, God’s creatorial title is simply God-Elohim; but when man and God’s relationship to him are brought in, it is then Lord God, or Jehovah Elohim. Afterward when Abraham is called out from idolatry to trust God, He then is revealed to him as “the Almighty God” (Gen. 17:1). It is also remarked, that in the brief account of King Melchisedec he is also spoken of as priest of the Most High God – king and priest; and he was, as we learn from Heb. 7, eminently typical of the Lord Jesus in a future day, when He will “be a Priest upon His throne;” and there God is spoken of as “the Most High God,” and also that He is “possessor of heaven and earth.” These titles will be asserted by our Lord in the millennial age, when He comes forth wearing His many crowns, and reigns in power and great glory (Gen. 14:18,22).
(Continued from page 171).
(To be Continued).

The Christian Traveler

We soon found ourselves in a canal-boat where were about thirty passengers of various ages and characters; and my curiosity was not a little excited to learn how my companion would proceed among them. The afternoon had nearly passed away, and he had conversed with no one but myself. At length he inquired of the captain if he were willing to have prayers on board.
“I have no objection,’ said he, “if the passengers have not: but I sha’n’t attend.”
At an early hour the passengers were invited into the cabin, and in a few minutes the captain was seated among them. After reading a short portion of Scripture our friend made a few appropriate remarks and earnestly commended us to God.
As soon as we rose from prayer, a gentleman whose head was whitening for the grave said, “Sir, I should like to converse with you. I profess to be a Deist. I once professed religion, but now I believe it is all delusion.”
“Sir,” said the young man, “I respect age and will listen to you; and as you proceed, may perhaps ask a few questions; but I cannot debate; I can only say that I must love Jesus Christ. He died to save me and I am a great sinner.”
“I do not deny that men are sinners,” said the old man, “but I don’t believe in Christ.”
“Will you then tell us how sinners can be saved in some other way, and God’s law be honored?”
We waited in vain for a reply, when my friend proceeded: “Not many years since I was an infidel, because I did not love the truth and was unwilling to examine it. Now I see my error; and the more I study the Bible, the firmer is my conviction of its truth, and that there is no way of salvation but through a crucified Redeemer.”
As the passengers sat engaged in conversation, one of them related the circumstances of a murder recently perpetrated by a man in the neighborhood while in a fit of intoxication. To this all paid the strictest attention. The captain joined them to hear the story, the conclusion of which afforded an opportunity for the stranger to begin his work. He was the fearless advocate of temperance as well as religion, and here gained some friends to this cause.
“But,” said he at length, “though alcohol occasions an immense amount of crime and misery in our world. I recollect one instance of murder with which it had no connection.” He then related as nearly as I can remember the following story: “In a populous city at the East was a man who seemed to live only for the good of others. He daily exhibited the most perfect benevolence towards his fellow men: sought out the poor and needy and relieved their wants; sympathized with and comforted the sick and the afflicted: and though he was rich, his unsparing beneficence reduced him to poverty. He deserved the esteem of all, yet he had enemies. He took no part in politics, yet many feared that his generosity was a cloak of ambition, and that he was making friends in order to secure to himself the reins of government. Others feared that his religious sentiments, connected with his consistent life, would expose their hypocrisy. At length a mock trial was held by an infuriated mob, and he was condemned and put to death.”
“Where was that?” “When was it?” “Who was it?” was heard from several voices.
“It was in the city of Jerusalem, and the person was none other than the Lord Jesus Christ, By His enemies He was hung upon the cross, and for us guilty sinners He died.”
Every eye was fixed upon the young man and a solemn awe rested on every countenance. He opened a Bible which lay upon the table and read the account of Christ’s condemnation and death; the captain nodded to him a signal for prayer, and we all again fell on our knees, while he wept over the condition of sinners, and, for the sake of Christ, besought God’s mercy upon them. Here again was a floating Bethel.
In the morning the stranger was not forgotten, and he evidently did not forget that there were immortal souls around him, hastening with him to the bar of God. During the day he conversed separately with each individual, except an elderly gentleman who had followed him from seat to seat and showed much uneasiness of mind; the realities of eternity were set before us, and the Holy Spirit seemed to be striving with many.
Towards evening our friend requested an interview with the aged man.
“Yes, yes,” said he, “I have been wishing all day to see you, but you were talking with others.”
He acknowledged that he had tried to be a Universalist; and though he could not rest in that belief, he never, until the previous evening, saw his lost condition. “And now,” said he, “I want you to tell me what I shall do.”
The young man raised his eyes to heaven, as if asking for the Lord’s guidance, and then briefly explained the nature and reasonableness of repentance and faith, accompanied by a few striking illustrations in proof of the justice of God in condemning and His mercy in pardoning sinners.
The old man saw the plan of redemption so clearly that he burst into tears, and exclaimed, “O my soul, my soul! How have I sinned against God! I see it – I feel it; yes I have sinned all my days!”
“But Jesus died to save sinners,” replied the young man. “Will you, my friend, give Him your heart?”
“O yes, yes, if I had a thousand hearts He should have them all,” was the answer.
The young man turned away and wept. For some minutes silence was broken only by the deep sighs of the aged penitent. There was something, in an hour like this, awfully solemn. Heaven was rejoicing, I doubt not, over a returning prodigal. As he stood alone and wept, he reiterated again and again, “Yes, I will serve God, I will, I will.” After a time his feelings became more calm, and lifting his eyes towards heaven, with both hands raised he broke out in singing,
“There shall I bathe my weary soul
In seas of heavenly rest,
And not a wave of trouble roll
Across my peaceful breast.”
And then again he wept, and said,
“Yes, O Jesus, precious Savior!”
The time had come for our young friend to leave us. By his zeal in his Master’s service he had stolen our hearts, and all pressed forward to express their friendship in an affectionate farewell.
Such was the influence of one individual whose unwavering purpose it was to live for God. He felt for dying sinners, and relying on the guidance of the Holy Spirit for success, labored for the salvation of souls around him. Will not the reader solemnly resolve, in God’s strength, that henceforth, whether at home or abroad, he will make the glory of Christ, in the salvation of men, the object of his life?
(Continued from page 177).

Practical Conversations With Our Young People: Questions for August

Always give Scripture for answers, as well as your own expressions.
1. What is required for true service?
2. What should our object be in service?
3. Why should we serve the Lord?
4. Why is it our reasonable (intelligent – New Trans). service to present our bodies as a living sacrifice?
5. Give five scriptures, showing that God is unwilling that any should perish.

Answers to Questions for June

ANSWERS TO QUESTION 1.
How did Paul and Daniel use their education for God’s glory?
(a) “For Christ sent me not to baptize, but to preach the gospel: not with wisdom of words, lest the cross of Christ should be made of none effect” (1 Cor. 1:17).
(b) “As for these four children, God gave them knowledge and skill in all learning and wisdom; and Daniel had understanding in all wisdom and dreams” (Dan. 1:17)
(c) “I thank my God, I speak with tongues more than ye all: yet in, the church I had rather speak five words with my understanding, that by my voice I might teach others also, than ten thousand words in an unknown tongue” (1 Cor. 14:18-19)
“The king communed with them; and among them all was found none like Daniel .... and in all matters of wisdom and understanding, that the king inquired of them, he found them ten times better than all the magicians and astrologers that were in all his realm” (Dan. 1:19-20).
Paul, who was taught according to the perfect manner of the law of the fathers (Acts 22:3), after he was converted, preached, and taught publicly, and from house to house. (Acts 20:7,20). For him to live was Christ. (Phil. 1:21). He did not boast of his knowledge, but counted all things loss for Christ. (Phil. 3:7-8).
Daniel, who was skilful in all wisdom, and cunning in knowledge, and understanding science (Dan. 1:4), gave God the praise by owning that wisdom and might come from Him. (Dan. 2:20,23).
ANSWERS TO QUESTION 2.
How does James tell a wise man to act?
(a) “Who is a wise man and endued with knowledge among you? let him show out of a good conversation his works with meatiness of wisdom” (James 3:13).
(b) A wise man would be “swift to hear, slow to speak, slow to wrath” (James 1:19).
ANSWERS TO QUESTION 3.
What was Timothy told to avoid?
(a) “O Timothy, keep that which is committed to thy trust, avoiding profane and vain babblings, and oppositions of science falsely so called” (1 Tim. 6:20).
(b)”Flee youthful lusts: but follow righteousness, faith, charity, peace, with them that call on the Lord out of a pure heart. But foolish and unlearned questions avoid, knowing that they do gender strifes. (2 Tim. 2:22-23).
ANSWERS TO QUESTION 4.
What is the wisdom from above?
(a) The wisdom from above is the wisdom given us when we feel our own weakness and dependance, and we ask God’s guidance and direction. “If any of you lack wisdom, let him ask of God, that giveth to all men liberally, and upraideth not; and it shall be given him” (James 1:5).
(b)It was what Solomon asked of the Lord (1 Kings 3:7-12). “But the wisdom that is from above is first pure, then peaceable, gentle, and easy to be entreated, full of mercy and good fruits, without partiality, and without hypocrisy” (James 3:17).
(c) “The fear of the Lord is the beginning of wisdom: a good understanding have all they that do His commandments” (Psa. 111:10).
ANSWERS TO QUESTION 5.
Give five scriptures showing that all are under condemnation.
John 3:18. John 3:36. Romans 5:1. Romans 5:18. Galatians 3:10.

Truths for Young Christians: Service, Part 2

True service begins with Christ, who is the Head, and when Christ is forgotten, then the service is defective; it has lost connection with the spring and fountain of all service, because it is from the Head that all the body by joints and bands having nourishment ministered, increaseth. The body is of Christ, and He loves it as He loves Himself, and everyone who would serve it, will best learn to do so, by knowing His heart and purpose towards it. In a word, it is Christ who serves, though it may be through us. We are but “joints and bands.” If we are not derivative and communicative from Christ, we are useless. To be useful, our eye and heart must be on Christ, and not on the issue of our service; though, if true to Him, the end will vindicate us, too, however disheartening the interval. He who judges of his service by present appearances will judge by the blossom, and not by the fruit; and, after all, the service is not for the sake of the church, but for the sake of Christ; and if He be served in the church, though the church own it not, yet, Christ being served, He will own it. Now the constant effort of Satan is to disconnect, in our minds, Christ from our service, and this he does much more than any of us, perhaps, have fully discovered. Whether in reading, or praying, or speaking, how seldom, if we judge ourselves, do we find that we act simply as towards Christ, and Him alone! How often may sentimentality and natural feeling affect us in our service, instead of simple love to Him!
“Thrice happy he who serveth
The Lord with heart and soul!
Whose purpose never swerveth,
Who loves the Lord’s control.
With single eye – unfearing –
With simple child-like faith –
The Master’s accents hearing;
He doth whate’er He saith’.”

Steadfast, Unmovable, Always Abounding

“Be ye steadfast, unmovable, always abounding in the work of the Lord, forasmuch as ye know that your labor is not in vain in the word” (1 Cor. 15:58).
“Always abounding” is Thy call, O Lord,
We cry for power to obey Thy Word;
And as we stand upon redemption ground
We would in Thy most blessed work abound.
For Thou art worthy of our very best,
Fain would we give it, leave to Thee the rest,
We know, O Lord, our labor’s not in vain,
Sure the reward, for Thou wilt come again.
Thou wast a servant, and in Thee we see
The perfect pattern of what we should be.
Doing Thy Father’s will, Thy one intent,
In wondrous works of grace Thy life was spent.
Help us, O Lord, to imitate Thy ways,
To live to serve Thee all our future days.
O that we ever may abounding be!
Our aim Thy glory for eternity.
“Always abounding,” Master teach us how;
Call, send, equip, empower us just now.
Ourselves we unreservedly would yield,
Appoint our task in Thy great harvest field.
Through storm, or sunshine, forth for Thee we go,
With weeping, bearing precious seed to sow,
Our only object to extol Thy fame,
Our one ambition to exalt Thy Name.
“Always abounding” – quicken our zeal for Thee,
To render service true, wholeheartedly;
A burning zeal, yet to Thy will resigned,
Still laboring on, forgetting things behind.
Thy guiding hand alone we ask to see,
As time and strength we gladly yield to Thee,
Abounding in Thy service, at Thy call,
Our Lord, our Master, and our All in all.
Abounding still, valiant for Thee we go,
Withstanding scorn, advancing to the foe.
With girded loins, and armor clear and bright.
With lights aflame in this world’s darkening night,
May we not falter for the battle’s Thine,
And we can conquer in Thy might divine;
And when our warfare’s past – our race is run,
Then shall we hear Thy blessed words – “Well done.”

Scripture Study: Matthew 24:32-51

In Luke 21, the Christians were to flee when they saw Jerusalem encompassed with armies. They were warned, and got out in time. Here it is plainly future, as spoken by Daniel the prophet. Sudden and sharp would be the enemy’s action; no time to be lost; escape to the mountain from off the roof, without anything. Do not go to the field, for the clothes left there. Woe to those who had to care for their young; how hard for them to escape, or in the winter, when the roads and gullies would be running streams, or on the Sabbath when only allowed to go a short distance, by the law. Here is another mark that it is Jewish (the Sabbath is not mentioned in Luke 21). Some of the Gentiles join in the persecution, others help the persecuted (Matt. 25:40,45).
Matthew 24:21-22. This brings us into the night of the great tribulation (Dan. 12:1; Jer. 30:7), such as was not from the beginning of the world to this time, no, nor ever shall be.
The Lord will prepare a place of safety for His people (Rev. 12:6,14), and will encourage them with His promises (Rev. 14:12-13). Some will suffer much, others will be killed, but the Word will encourage them. A voice from heaven bade John write for them, “Blessed are the dead which die in the Lord from henceforth: Yea, saith the Spirit, that they may rest from their labors; and their works do follow them.” If they lose their lives on earth, they will gain a heavenly place, and their works will follow them there. And then they will rest.
What comfort the suffering saints of that time will find in the Psalms, such as, “My times are in Thy hand: deliver me from the hand of mine enemies, and from them that persecute me” (Psa. 31). The Lord’s eye will he upon them, like the refiner of silver, watching till their dross is purged – He will shorten the days for His elect’s sake.
Matthew 24:23-24. False prophets and false Christs arise with great signs and wonders, if possible to deceive the very elect. How easily cheated men are with a show of power, God has warned His people about this. (Deut. 13:1-3). Satan’s works of power are getting very plentiful now, but it will be full blown in that day. (2 Thess. 2:9). By such means souls are enticed and destroyed, led from the truth of God’s Word and ruined for eternity. Those that hearken to God’s Word will be kept in His path.
Matthew 24:26-27. They, being warned of God, and knowing that Jesus is on high, will not look for Him in the desert, or in the secret chamber; they will look for Him coming as the lightning cometh out of the east and shineth even unto the west; coming as the Son of Man from heaven, appearing in glory.
How blessed this divine certainty is, as we, the church, know it. We will be caught up to be with Him in the air. This is our blessed hope. (1 Thess. 4:15-17).
Matthew 24:28. With what unerring precision, and swiftness, the judgment will fall on the ungodly. There is no escape for them then.
Matthew 24:29-30. Now moral chaos and physical derangements, causing dire perplexity and fear in men’s hearts (see Luke 21:25-27), and for looking after those things which are coming on the earth: for the powers of heaven shall be shaken. (Joel 2:30-31). And then appears the sign of the Son of Man in heaven. He appears, to the great disappointment of all those who looked for an earthly Messiah to arise, who find after all it is the despised Jesus; but what joy to those who believed, and waited for Him to come from heaven. (Isa. 25:9). Then shall all the tribes of the land mourn, and they shall see the Son of Man coming in the clouds of heaven with power and great glory.
In Zechariah 12:10-14, grace is at work in giving repentance; here it is but worldly disappointment. The one is faith; the other, unbelief’s bitter fruit.
Matthew 24:31. And He shall send His angels with a great sound of a trumpet, from one end of heaven to the other (Isa. 27:13; Zech. 2:16). The elect, in this chapter we learn by the context, is Israel. (Isa. 65:9,22). There are others who are elect, but the heavenly saints are caught up before this to meet the Lord in the air. There are also elect angels. (1 Tim. 5:21). The object of this prophecy is to show His coming for the relief and ingathering of Israel. He is King and Judge, and the angels sound the trumpet loudly to gather them.
(Continued from page 188).
(To be Continued).

What Is the Camp? Hebrews 13:12-13, Part 1

Hebrews 13:12-13. In the last chapter of this epistle the Spirit of God, speaking of the fact that “the bodies of those beasts, whose blood is brought into the sanctuary by the high-priest for sin, are burned without the camp,” goes on to say, “Wherefore Jesus also, that He might sanctify [set apart] the people with His own blood, suffered without the gate. Let us go forth therefore unto Him without the camp, bearing His reproach.”
Many are at a loss to know what is referred to by the term “camp”; yet it is very important to understand it, because those who love the Lord Jesus are exhorted to go forth from it, unto Him.
In order to learn what is meant, then (as is our duty to do), by this expression, we should first of all find out to whom the epistle is primarily and especially addressed, and with God’s blessing it will soon be made plain.
The Epistle to the Hebrews was written to professedly converted Jews who had been born and trained up under the law given by God to Israel at Mount Sinai when in camp in the wilderness, where and when a system of worship was ordained of God for this nation in the flesh, which was perfect in its place. but which did not suppose or require that the worshipers should be born again, and under which system they as a nation utterly failed.
Next, we must get clear on what the Spirit. of God calls “the camp”, when this was written.
The ninth chapter tells us that there were ordinances of divine service, and a worldly sanctuary; consisting of a tabernacle made with men’s hands, and pitched on earth (afterward, when in the land, it was a temple at Jerusalem), which was their place of worship.
Then they had an ordained priesthood – men of a certain family set apart to come between the worshipers and God, of whom Aaron was the high-priest. And this man was the minister of this sanctuary, without whom the worshipers could not worship.
This high-priest offered up an atonement for these people’s sins once every year, besides the other sacrifices that were offered up continually; and, as already noticed, these worshipers did not require to be converted people, though some of them were so, but were a company of believers and unbelievers mixed together; all of them on the ground of law keeping for righteousness.
Now this system embraced as worshipers all the nation of Israel then in camp, and was at that time going on at Jerusalem when this epistle was written; and the Spirit of God calls this “the camp”, out of which the “holy brethren, partakers of the heavenly calling,” were to go forth.
I learn, then, that Judaism, or this system of worship in the flesh, was in Paul’s day “the camp.” And, dear Christian reader, is it not clear at a glance that any system of worship of a Jewish nature and character, and, in consequence, a system that the flesh and sight – the world in fact – can more or less join in, is the camp in our day? We know, as revealed by the Son of God Himself, that “God is a Spirit, and they that worship Him must worship Him in Spirit and in truth” – a thing impossible for the flesh to do. It requires men to be born of the Spirit and sealed with the Spirit. In John 4:21, our Lord says: “Woman, believe Me, the hour cometh, when ye shall neither in this mountain, nor pet at Jerusalem, worship the Father.” The time had come when places of worship on earth – mountains, and temples made with hands – should be done away with. Men in the flesh had been thoroughly tested and proved to be utterly incapable of obeying God’s law, or worshiping Him either; and now a new order of things comes in: men born of the Spirit, indwelt by the Spirit, worshiping in Spirit and in truth, not in a temple or tabernacle on earth, but in the holiest, that is, heaven itself – the only worshiping place now – having boldness to enter there by the blood of Jesus.
Therefore if any system of worship exists now which has a worldly sanctuary – a temple made with hands, with an ordained priesthood, or class of men set apart to take a special place between the worshipers and God, without whom they cannot worship; where the worshipers are a mixture of converted and unconverted people under the law; that is – must be, indeed, – ”the camp.” It is Judaism, having its nature and character as a system of worship, though more or less Christians be in it, and the gospel be still preached more or less faithfully by some in it too.
It is the Holy Spirit – it is God – that says, through the apostle, to any of His dear children who are in any kind of system such as this, “Let us go forth unto Him (Christ) without the camp, bearing His reproach.”
“But,” say some, “why not stay inside and do all the good you can where you are?”
Because, “to obey is better than sacrifice, and to hearken than the fat of rams” (1 Sam. 15:22).
Because, God will not have Christianity and Judaism mixed up together – the new wine in the old bottles.
Because, God will not have the world and the church unequally yoked together (2 Cor. 6).
Because, Jesus Christ has come an High Priest of good things to come, by a greater and more perfect tabernacle, not made with hands. He has been cast out by worshipers in the flesh (Jewish worshipers), and suffered outside the gate, on Calvary; proving that worship in the flesh is not a real thing; for if they had loved God, they would have reverenced His Son. Yea, though these worshipers in the flesh could point out in the Scriptures to the wise men of the east where Christ was to be born, not one foot did they go to find Him, but were troubled and distressed at the very thought of His being come.
And now, risen from the dead, He has entered into the holy place made without hands, even into heaven itself; not with the blood of bulls and goats, “but by His own blood,” “having obtained (not redemption for a year only, but) eternal redemption for us.” So now the Man in the glory, the God-man, is our Minister – “A minister of the sanctuary, and of the true tabernacle, which the Lord pitched and not man” (Heb. 8:1-2; 9:11-12,24).
And though it is true that God at sundry times and in divers manners spake in time past unto the fathers by the prophets, He has more highly honored us in these last days, says Hebrews 1:2 – He has spoken unto us by His Son. Here, the Spirit of God puts God’s Son above and before all other prophets.
Again, though true that Moses was the apostle (and was faithful in all His house as a servant), and Aaron was God’s called, chosen high-priest for this nation of worshipers in the flesh – Israel; we Christians are to consider the Apostle and High Priest of our profession, Christ Jesus; thus exalting Christ above and before Moses and Aaron. (Heb. 1:1-2; 3:1-6). Under the law, it was commanded and instituted by God Himself that sacrifices of bulls and goats should be offered – a shadow of better things to come; but now, Christ has offered Himself, and risen and gone in and presented His own blood to God for us, having thus by one offering perfected forever them that are sanctified (Heb. 10). Thus Christ’s sacrifice is put above, and in the place of, all these other sacrifices which could never take away sins. The Substance has come and taken the place of the shadows.
Again, though perfectly true that God told Moses to make the tabernacle, and Solomon the temple, for a place of worship on earth under the law; now, He has opened heaven to us, and we have boldness to enter into the holiest by the blood of Jesus – heaven itself, taking the place of the tabernacle, and temple at at Jerusalem, as the Christian’s worshiping place, which we enter by faith and in Spirit. (Heb. 8:1-2; 9:24).
(To be continued).

What God Hath Said on the Second Coming of Christ and the End of the Present Age: Part 8

The Second Epistle to the Thessalonians was written to deliver them from this mistake and sorrow. Instead of the coming of the Lord being a day of trouble to them, the apostle says, “And to you who are troubled, rest with us; when the Lord Jesus shall be revealed from heaven, with His mighty angels” (2 Thess. 1:7). So far from the world persecuting you in the day of vengeance, you shall rest with us, caught up; as he had taught them. Flaming “vengeance shall be taken on them that know not God, and that obey not the gospel of our Lord Jesus Christ.” Punished with everlasting destruction from His presence, “When He shall come to be glorified in His saints, and to be admired in all them that believe (because our testimony was believed), in that day” (Ver. 10). Still further to assure them, he says, “Now we beseech you, brethren, by the coming of our Lord Jesus Christ, and by our gathering together unto Him, that ye be not soon shaken in mind, or troubled, neither by spirit, nor by letter, as from us, as that the day of the Lord is at hand” (2 Thess. 2:1-2).
Two things were certain before the great and terrible day of the Lord – His coming for them, and their gathering to Him, as taught them in the first epistle.
O! could one think it possible, as we walk the streets, and watch the busy crowd, that destinies so vastly opposite await that crowd – the believer to be caught up to meet the Lord (perhaps this very day), the unbeliever to be left to the fierceness of that day of vengeance.
In 2 Thessalonians 2:3-12 we have some of the terrible features of the end of this age. “The falling away first.” However sadly the professing church has departed, yet what will it be when the real church of God is taken up! The full character of this falling away is described in Revelation 17. One terrible feature is the revelation of the wicked one, “who opposeth and exalteth himself above all that is called God, or that is worshiped; so that he, as God, sitteth in the temple of God, showing himself that he is God” (2 Thess. 2:4).
Some, not observing that this fearful character appears on the closing scene of human wickedness, after the true church is taken up to be with the Lord, have thought this man of sin is Popery, or the Pope. But do not you see this passage says, the man of sin shall sit in the temple of God. Now God never has, and never will have, a temple built on earth in any place except Mount Zion, or Mount Moriah – the place in which he appeared to Abraham. But that temple is now destroyed. It must, then, be rebuilt, as many scriptures show it will. And the terrible man of sin is evidently one of Daniel’s people, that is, a Jew – who shall come in his own name – whom the Jews shall receive (John 5:43).
As Satan entered into Judas, so will he enter into this son of perdition. St. Peter’s at Rome cannot be the temple of God; neither can the Pope be this man of sin. Daniel plainly describes him as the wicked Jewish king at the time of the end: “And the king shall do according to his will; and he shall exalt himself, and magnify himself above every god, and shall speak marvelous things against the God of gods, and shall prosper till the indignation be accomplished: for that that is determined shall be done.” We all know the Pope is not the King of the Jews. It is quite clear that all this is Jewish, and cannot take place while the Spirit and the church are here. As Paul had well taught these converts, he reminds them how he had told them, “And now ye know what withholdeth, that he might be revealed in his time.”
“The mystery of iniquity doth already work” – the leaven foretold in Matthew 13. “Only He who now letteth (or hindereth) will let until He is taken away.” But O! what will it be when the Spirit of God is taken, and the church caught up to meet the Lord? “And then shall that wicked one be revealed.” And, now, how clearly this proves this wicked one is not Popery, for it is he “whom the Lord shall consume with the spirit of His mouth, and shall destroy with the brightness of His coming.” Popery, and the whole of the ecclesiastical apostacy, will be destroyed by the ten kings (Rev. 17:16). But this wicked one “is after the working of Satan, with all power and signs, and lying wonders.” The present work of Satan, in leading men to have to do with devils, and familiar spirits, by table-turning and the like, may be preparing the way. But these terrible events cannot possibly take place during this day of gospel grace. For in those days of darkness, God will send them strong delusion, that they may believe a lie, that they may all be damned. This will be assuredly the case when this day of mercy closes. God will arise and shake terribly the earth. “For this cause God shall send them strong delusion;” that is, because they received not the love of the truth that they might be saved. Man is damned because he receives not the truth. These are God’s words as to the end of the present age. Fellow-believers, we are saved “because God hath from the beginning chosen us to salvation, through sanctification of the Spirit and belief of the truth” (vs. 13). O! it is this that makes the coming of the Lord so precious – God’s eternal love. The apostle closes the subject in this epistle with these words, “And the Lord direct your hearts into the love of God, and into the patient waiting for Christ” (2 Thess. 3:5).
(Continued from page 196).
(To be continued).

Correspondence: Daniel 7:9, 13; 1 Cor. 4:4

Question 102: Who is the “Ancient of Days” in Daniel 7:9,13; and who is the “One like the Son of Man”? E. M. S.
Answer: Jehovah, also called the Most High, is the judge of all the earth (Gen. 18:25).
Our Lord Jesus is the true Son of man who gave Himself up to glorify God in the work of atonement, and who is appointed judge (John 5:27; Acts 17:31).
Revelation 5:9 declares, He is worthy to take the book of God’s judgments, and to open the seals thereof, because “Thou wast slain”.
His is the everlasting kingdom spoken of in Daniel, and throughout the Scriptures.
In Revelation 11:15, the voices in heaven say: “The kingdom of the world of our Lord and of His Christ is come, and He shall reign to the ages of ages” (JND Trans).
“He is King of kings, and Lord of lords” (Rev. 19:11-16).
In Daniel 7:13, He comes to the Ancient of Days, and in verse 22 He is the Ancient of Days.
Even down here He was God manifest in the flesh.
“Worthy by all to be adored.”
Question 103: What is the meaning of “I know nothing by myself”? (1 Cor. 4:4). H. F.
Answer: J. N. Darby Trans. reads: “For I am conscious of nothing in myself.”
The Corinthians had been judging the motives of the laborers who are stewards for God. Paul committed his own behavior to the Lord, and while not conscious of doing wrong, said, this did not justify him. The Lord is the only true judge, and He would judge every motive and counsel of the heart at the judgment seat of Christ.
We must act in sincerity before the Lord for ourselves, and yet at the judgment seat we may find, in some things we were greatly mistaken.

Inspiration of the Scriptures: The Bible - its Perfection, Part 4

THE BIBLE – ITS PERFECTION.
When God has a people on earth, He is made known as Jehovah, or I AM; and when they are redeemed out of Egypt, He dwells among them, and maintains covenant relationship with them as Jehovah. This goes on as long as He can own them as His people, and when He can no longer say of them, “My people,” even then, instead of utterly giving them up, Jesus is born into the world to “save His people from their sins.” He is called Jesus, or Jehovah Savior. He is spoken of in Scripture as that Holy Thing born of Mary, Son of God, Son of Man, Son of the Highest, the Christ, Messiah or Anointed. the Lord Jesus Christ, Christ Jesus, Jesus Christ, the Word, Lamb of God, King of Israel, and each title is used with the most perfect accuracy and point. Take a few examples. Stephen full of the Holy Spirit looked up steadfastly into heaven, and saw the glory of God, and Jesus standing on the right hand of God; and why did he say that he saw Jesus? Why not use another of His many titles? Because Jesus is His name as incarnate, and Stephen’s special comfort when suffering for His truth was the contemplation of Him as Man, who after suffering for the truth unto death was now on the throne of God. His confession therefore was, “Behold I see the heavens opened, and the Son of Man standing on the right hand of God.” Though the faithful martyr beholds Him as glorified Man, yet be it also observed, that he bows to Him there as “Lord of all,” and says, “Lord Jesus, receive my spirit!” and again, “Lord, lay not this sin to their charge. And when he had said this he fell asleep” (Acts 7:55-60). All true believers bow to Jesus as Lord.
Again, when an alarmed and sin-convicted man cried out, “What must I do to be saved?” Paul’s reply was, “Believe on the Lord Jesus Christ, and thou shalt be saved, and thy house;” but why does he here repeat all these titles of the Savior? Because he is presenting Him to this anxious soul as the object of faith, and in such a way as to fully satisfy his awakened conscience. Jesus is not only Son of Man in the glory, but has been there made “both Lord, and Christ” (Acts 2:36). If the question of the troubled heart be, Is He willing to save me? The answer is, Yes; He is Jesus, and came into the world for the very purpose of saving sinners. If the inquiry be, Is He able to save me? The answer is, Yes; He is “Lord of all,” and has all power in heaven and in earth; and His being the Christ, the anointed one in heaven. consequent upon His finished work on the cross, is the unquestionable proof that every one that has come as a sinner to Him the Savior, is forgiven and blessed. How perfectly accurate, therefore, was the apostle’s reply, “Believe on the Lord Jesus Christ, and thou shalt be saved.”
Again, when the same apostle is writing about the Lord’s coming, he says, “We look for the Savior, the Lord Jesus Christ;” because he is then contemplating Him not in humiliation but in power; that power as Lord of all, by which “He is able even to subdue all things unto Himself,” and yet Savior even to the changing and saving of our body.
Look at another instance, when saving faith is referred to. It is said, “If thou shalt confess with thy mouth the Lord Jesus” – not simply Jesus, for many speak of Him as such who never savingly know Him, but Lord Jesus – the Son of Man in the glory in whom dwells “the fullness of the Godhead bodily,” and who is “Head of all principality and power” – “and shalt believe in thine heart that God hath raised Him from the dead, thou shalt be saved.” Could anything be more precise and pointed in the use of titles?
It is well known that the book of Psalms consists of five divisions, or books. The Hebrew Bible is so arranged; and it is very striking to see how remarkably the titles of God are used in them. Look, for instance, at the first and second books. The first book consists of Psalm 1 to Psalm 41, and the second book from Psalm 42 to the end of Psalm 72. In the first book the prevailing title by far is Jehovah, because the godly Jews are looked at as in Jerusalem, and still in association with the temple; whereas in the second book the title most commonly used is not Jehovah, but God. And why? Because the remnant of Jews are looked at there as having fled from Jerusalem, in fulfillment of our Lord’s words. “Then let them which be in Judea flee to the mountains,” and their covenant relationship with Jehovah is little realized by them. Far away from their beloved city and temple, in the hill Mizar, they feel cast down, as if God had forgotten them. Take a psalm in the first book, say Psalm 34, and Jehovah is mentioned sixteen times; and in a psalm in the second book, say Psalm 41 or Psalm 42, and we find God used nearly as many times as there are verses. The accuracy is most striking.
The precision and depth of meaning with which the divine titles are used all through Scripture is very manifest. If in Proverbs we have Jehovah, is it not because those addressed are looked at in relationship with Him? If in Ecclesiastes we have God, is it not because it is for the most part man seeking on earth an object for his heart. and finds vanity and vexation of spirit? And if in the Song of Solomon we have neither, is it not because the heart has there found a satisfying Object? No doubt in its primary application it is Jewish, but who among the children of God has not delighted in our Lord Jesus Christ as the “altogether lovely,” and rejoiced at realizing that He has brought us into His banqueting house, and that His banner over us is love? By the Spirit we are able, while
“Gazing on the Lord in glory,”
to sing, “I am my Beloved’s, and my Beloved is mine,” and can surely add, “I sat down under His shadow with great delight, and His fruit was sweet unto my taste.”
(Continued from page 200).
(To be Continued).

The Mission of Tracts

Tracts can go everywhere. Tracts know no fear. Tracts never tire. Tracts can be multiplied without end by the press. Tracts can travel at little expense. They run up and down, like the angels of God, blessing all, giving to all, asking no gift in return. They can talk to one as well as to a multitude, and to a multitude as well as to one. They require no public room to tell their story in. They can tell it in the kitchen or the shop, the parlor or the closet, in the railway coach or in the omnibus, on the broad highway or in the footpath through the fields. They take no note of scoffs, or jeers, or taunts. No one can betray them into hasty or random expressions. Though they will not always answer questions, they will tell their stories twice over, or thrice, or four times, if you wish them. And they can be made to speak on every subject, and on every subject they may be made to speak wisely and well. They can, in short, be made the vehicles of truth, the teachers of all classes, the benefactors of all saints.
“Cast thy bread upon the waters: for thou shalt find it after many days” (Eccl. 11:1).
To encourage God’s people to give out or send out tracts, “sowing beside all waters,’ I give a fact which has come to my knowledge.
In 1785, Dr. Cook gave a tract to a family in Virginia. The family numbered fourteen, and that tract was the instrumentality used in the conversion of the whole family. Dr. Chickering’s tract, “What it is to believe in Christ”, has been a wonderful power for good. Nearly two thousand people have written to, or told him personally, that they owe their conversion to that tract. In 1876, a wealthy manufacturer of Cincinnati (in whose employ the writer was at that time), took with him to Florida a few copies of a tract, “Good News for You”. On meeting that gentleman several years later, he said that he gave some of those gospel tracts to a Methodist evangelist who was holding meetings in Florida. The evangelist thought that the tracts would be very useful at his meetings, and he had an edition printed which he distributed at all the services. He subsequently stated the grand result was, that by the blessing of God’s Holy Spirit, they were the means of the conversion of over three hundred persons.
All these years our God has been watching over these silent messengers, and who can tell into how many hands they have fallen, and how many hearts have been moved to receive the truth as to God’s salvation.
One has truly said: “Whether we see results or not, it is our blessed privilege to know and rejoice in the fact, that, if we sow good seed, the harvest will certainly be for His glory.” “Know now that there shall fall unto the earth nothing of the Word of the Lord” (2 Kings 10:10).
Again, books (“precious seed”) were left in the hands of a Christian lady in Kansas, who packed them away until mold came upon them. For six years they remained there, when God, by His Spirit, awakened her at two o’clock a. m., and in a moment showed her that that which had been pressed upon her was truth. The books were brought out and read and she was delivered of all that was of man.
Beloved, are we thus laying up treasures in heaven to meet us in that day, and to receive at His hand the reward for such service? If we cannot do this work in person, because of the worldly service in which we are engaged, we can have fellowship with the Lord by putting into the hands of such as go forth to “sow beside the waters.”
Let us not miss the rich blessedness which will surely come to our own souls as we thus stand in our lot, before Him to do thus His bidding.
May our God mightily stir up His saints everywhere to do what they can. The time is short and the days are evil. Satan’s hosts are exceedingly active in filling the land with that which leads souls down to perdition.
Beloved, think of the tens of thousands of tracts (silent messengers which speak for Him many times, in many places, and to many hearts) which you may have fellowship in sending forth. And know assuredly, that in the day of manifestation a host shall come before Him, saved through such instrumentalities; besides the many saints who have been refreshed by the way through a tract or book given them.
O, beloved! “It is more blessed to give than to receive.”

Practical Conversations With Our Young People: The Need of Dependence

THE NEED OF DEPENDENCE
Is God known, loved and trusted? If He be, the heart will delight in the most absolute dependence upon Him; if not, such dependence would be perfectly insufferable. The unrenewed man loves to think himself independent – loves to fancy himself free – loves to believe that he may do what he likes. Alas! it is the merest delusion. Man is not free: he is the slave of Satan. It is now well-nigh six thousand years since he sold himself into the hands of that great spiritual slave-holder, who has held him ever since, and who holds him still. Satan rules man by means of his lusts, his passions and his pleasures. There is no freedom save that with which Christ makes His people free. He it is who says: “Ye shall know the truth, and the truth shall make you free.” And again: “If the Son shall make you free, ye shall be free indeed” (John 8).
Here is true liberty. It is the liberty which the new nature finds in walking in the Spirit, and doing those things that are pleasing in the sight of God. The service of the Lord is perfect freedom. But this service, in all its departments, involves the most simple dependence upon the living God. Thus it was with our blessed Lord, the perfect Servant.

Questions for September

Always give Scripture for answers, as well as your own expressions.
1. What scriptures prove the humanity of the Lord Jesus Christ?
2. What scriptures prove the deity of the Lord Jesus Christ?
3. Give a few of the divine names that are given to Him, and their meanings.
4. Give Scripture proof that the Lord Jesus is a man in heaven now.
5. Give five scriptures that Christ is the only way.

Answers to Questions for July

ANSWERS TO QUESTION 1.
Should a Christian attend all the different kinds of religious meetings?
(a) No, it is not right for a Christian to attend all the different kinds of religious meetings, for the word is, “In a great house there are not only vessels of gold and silver, but also of wood and of earth; and some to honor, and some to dishonor. If a man therefore purge himself from these, he shall be a vessel unto honor, sanctified, and meet for the master’s use, and prepared unto every good work” (2 Tim. 2:20, 21). “Let us go forth therefore unto Him without the camp, bearing His reproach” (Heb. 13:13). Outside the religious camp.
(b) 2 Corinthians 6:7. Christians should not attend all kinds of religious meetings, because the Lord, would have us to live in separation, and He has a place where He would have His children to gather. Matthew 18:20 is the only place where He promised to be in the midst, and it is His desire to have us there. Going to all kinds of meetings will bring one into confusion. We will notice that when the Lord sent his disciples to prepare the Passover, He directed them where to go. I am sure He will not leave any one in doubt where he should go to meet Him if such a one is only willing to obey. They followed the man with the pitcher of water, and He is still the Person who will guide – the Holy Spirit.
ANSWERS TO QUESTION 2.
What is the lesson to be learned from Jeremiah 15:19?
(a) The lesson to be learned from Jeremiah 15:19 is, that it is only when walking in the path of obedience and separation that the Lord can really use us, or we be a real testimony for Him.
(b) I believe we get separation in Jeremiah 15:19, and it is a good answer to the first question. It shows how God delights to have His people to be separate even if one should stand alone. I believe it is not His desire to have us go back to the old thing we have left behind, which is indicated by the last clause of the verse “but return not thou unto them.”
ANSWERS TO QUESTION 3.
What place does music get in God’s Word?
(a) Musical instruments were invented by the children of Cain (Gen. 4:21). But with only a few exceptions, such as Daniel 3:5, Daniel 6:18, 1 Samuel 18:6, 1 Samuel 16:23, both, musical instruments and singing, are spoken of in connection with the praise and worship of Jehovah and in connection with the temple worship (1 Chron. 6:13; 1 Chron. 25:1).
(b) If I understand the question right, music is given the place in connection with worship, as in Psalm 150. When we find music we generally find joy and rejoicing. It is when one is happy in the Lord that he can praise God most for His mercies and goodness.
ANSWERS TO QUESTION 4.
Give five proofs of the inspiration of Scripture (a) 2 Timothy 3:16, 17; 2 Peter 1:20, 21; 1 Peter 1:23; Hebrews 4:12; 2 Peter 3:15,16; Luke 24:27.
(b) Isaiah 53:3; Mark 14:65;16:14; Jeremiah 6:8; Jeremiah 39:2-8; Isaiah 53:5; 1 Peter 2:24; Psalm 22:7-8; Luke 23:35; Psalm 22:18; Luke 23:36. These scriptures were prophesied many years before they happened, and they are so perfect, that if they had not been inspired of God, it would not have entered into the mind of man.
ANSWERS TO QUESTION 5.
Give five scriptures that the unsaved are awaiting judgement.
(a) Revelation 21:8; 2 Peter 2:9: John 5:29; John 3:36; Matthew 25:41; Mark 16:16; Daniel 12:2; Isaiah 66:24; Proverbs 16:4; Job 21:30; Matthew 13:41-42; Luke 13:27-28.
(b) Acts 17:31; 1 Thessalonians 5:3; 1 Peter 4:18; Revelation 20:12-15; John 3:15.

Truths for Young Christians: Three Sore Evils

There are three things from which many of the people of God suffer severely, and which may truly be called “sore evils,” namely a legal mind, a morbid conscience, a self-occupied heart. We can do little more in these few lines than name these things and point out the remedies for them, praying the Lord in His great mercy, to give full deliverance to any of our readers who may be tried by any or all of these things.
1. And, first, as to a legal mind. This is a very common evil, and one hard to be laid aside. In many cases it cleaves to the very last, and robs the soul of that peace and liberty which are the proper portion of all the children of God. It exhibits itself in various ways. It hinders the soul in its enjoyment of the free grace of God, and of the salvation which that grace has accomplished, and lowers the whole tone of the life and character. Furthermore, it falsifies the character of God, by presenting Him as an Exactor demanding a certain amount of duty, instead of a giver delighting in praise. In a word, a legal mind, in so far as it is allowed to work, spoils everything. It creates a dark cloud between the soul and God, and in doing this it throws everything into confusion. There may be the most scrupulous attention to the letter of Scripture – the most earnest desire to keep the standard of conduct up to what that letter enjoins, all right enough, no doubt, but the legal mind renders all cold, formal, heavy and ungenial. Service is put as a duty instead of a delight. It chills the affections and hinders their going out after God Himself.
Thus much as to this first sore evil. And now one word as to the remedy. What is it? Grace. Yes; grace is the grand remedy for a legal mind. Let the free grace of God, in all its sweetness and heavenly power, enter into the soul. Let God be known and enjoyed in His true character as the Giver – the one delighting in worship – inhabiting the praises of His ransomed people. Let grace possess the whole being. Let it be known and realized that we stand in absolute grace, that we are not under law but under grace, that every yoke is broken, and every fetter burst, that we are looked at in Christ, and loved as He is loved, washed in His blood and brought nigh to God. Let these divine realities be laid hold of in the power of simple, childlike faith, and the shadows of a legal mind will be chased away, and all its hateful workings counteracted. A heart established with grace is the sovereign remedy, the divine specific, for the sore disease of a legal mind.
2. We shall now dwell for a moment on a morbid conscience. How does this evil work? It, too, works in various ways, and cuts out a vast amount of sorrowful work for the soul. It is continually creating difficulties and suggesting doubts. Instead of being governed by the plain precepts of the Word of God, it is ever and only governed by its own fears. No one who has not been troubled with a morbid conscience can have any idea of the amount of suffering it entails upon its possessor. If it should so happen, and it often does, that a morbid conscience stands connected with a legal mind, the poor harassed soul must be a stranger to peace and joy in believing.
Now, what is the remedy for this sad and afflicted disease? Truth. The plain truth of God, the authority of holy Scripture, the conscience brought into immediate contact with the Word, and subjection to it alone – this is the remedy for a morbid conscience. In this way, the soul is governed simply by the claims of divine truth, and not by its own scrupulous fears – an immense deliverance!
3. Finally, as to the grievous evil of a self-occupied heart. It would be utterly impossible to trace its workings, so manifold and various are they. There are few who do not know something of this, even though they may not suffer from a morbid conscience or a legal mind. A self-occupied heart leads us to look at things and think of things and estimate things in reference to ourselves.
We value people in proportion as they adapt themselves or are agreeable to us. There is, though we may not be fully aware of it, a manifest leaning towards person’s who suit us in tastes, feelings, opinions and habits of thought. We like those who agree with us in all our peculiar views and prejudices. Men and things are not looked at in simple reference to Christ and His interests, but rather to self and its interests.
This is indeed a sore evil. We may say, and say it, too, with much decision, that self-occupation is the death blow to fellowship-fellowship with God, and fellowship with God’s people. And what is the remedy? What is the infallible cure, the divine specific for self-occupation? The person of Christ. Grace is the remedy for a legal mind, truth for a morbid conscience, and the embodiment of grace and truth, even Christ Himself, for the self-occupied heart. May we know the real power and blessedness of these things.

Matthew 24: Scripture Study

Matthew 24:32-33. Now learn a parable of the fig tree. Israel, who can produce no fruit forever, will show signs of life and prosperity, and have all the appearance of outward success, and this will be a sign to the disciples that the time is near.
Matthew 24:34-35. But, alas! it is the same perverse, unbelieving generation, as perverse as ever. In the days of Christ’s sojourn on earth, they stumbled over Him; but when He comes, He will fall upon them, and grind them to powder. (Matt. 19:44). Between these two points all the present interval has come in, but Israel, distinct from the nations, is unbelieving Israel still, and will so remain till all these things be fulfilled. “Heaven and earth shall pass away, but My words shall not pass away.” All shall be fulfilled.
Matthew 24:36-41. Signs may show it is near, but the time of His coming is hidden. Faith depends upon it, and expects Him, but this faith may be so small that it is said, “When the Son of Man cometh, shall He find faith on the earth?” (Luke 18:7-8). Patience must be exercised amid the sufferings of that time, for of that day and that hour knoweth no man, no, not the angels of heaven, but My Father only. With all Israel’s dates and signs, the exact time of the Son of Man’s coming is not known on earth. How much less could anyone presume to say when the Lord will come for the church, where neither dates nor signs are given?
“The wise shall understand,” and wait with expectant hearts, but the wicked will make it a time like the days of Noe – eating and drinking, marrying and giving in marriage, until the day that Noe entered into the ark, and knew not till the flood came, and took them all away; so shall also the coming of the Son of man be. But discrimination will be shown, and no righteous will be carried away with the wicked. Judgment will take the wicked one away, and leave the other one in the field; or those working together at the mill, one shall be taken and the other left. There can be no hiding from Him then. When the Lord comes for His church, all His own will be caught up to meet Him in the air. Christless professors will be left behind to perish with everlasting torments (2 Thess. 1:7-9; 2:10-12). It is just the opposite, at Christ’s coming for His saints: the good are taken, the wicked left. At His coming to the earth as Son of Man, the wicked are gathered out; the just are left to possess the kingdom. (See Matt. 13:41-42,49-50).
Matthew 24:42-44. Here is an exhortation to the believers of that time, warning them to be ready for the sudden appearing of their Messiah as the Son of Man from heaven. There is great temptation to yield to Satan in the hope of getting an easier path. Many of them shall fall, but “they that be wise shall shine as the brightness of the firmament; and they that turn many to righteousness, as the stars forever and ever” (Dan. 12:10,1-3).
From Matthew 24:45 to chapter 25:30, we have truth that applies to the present time. From Matthew 25:31 gives the coming of the Son of Man.
Matthew 24:45-47. The Lord speaks now to the heart of His servants, that they might wisely and faithfully feed His flock (Acts 20:28), a work precious to Him who loved and gave Himself for His church; and still loves them, even unto the end. All the service of Christ is precious to Him, but this is not so much gospel work which is sometimes allowed to displace Christ and the church because of its importance, but here we are reminded how the Lord desires the wise and faithful servant to give to His household food in due season, putting us in mind of that word in Luke 10:35: “And whatsoever thou spendest more, when I come again, I will repay thee.” And here, “Blessed is that servant, whom His Lord, when He comes, shall find so doing. Verily I say unto you, that He shall make Him ruler over all His goods.” He wears our names upon His breast (Ex. 28:29).
Matthew 24:48-51. There are evil servants also, who know not the character of the Master they profess to serve, who are not waiting for the Lord; their hearts say, my Lord delayeth His coming. They assume authority that belongs to Christ, and abuse their fellow-servants; they eat and drink with the drunken, that is, have fellowship with the world that are such. Such servants get the doom of the hypocrite, where there shall be weeping and gnashing of teeth.
This would apply to the profession as a whole, describing the kingdom. The professing church has settled down into the world, follows its ways and customs, and is unfaithful to Christ. There are faithful individual servants who will be rewarded, but this is brought out in the next two parables.
(Continued from page 214).
(To be continued).

What Is the Camp? Hebrews 13:12-13, Part 2

Then, it was true that unconverted as well as converted ones were in the congregation of worshipers under the law; now He who is a spirit, and must have worship in spirit and in truth, is seeking such to worship Him.
Who can do so? Those, and those only, who, converted and sealed by the Spirit of God, have the power and heart for it.
In short, Christ and Christianity has taken the place of, and is a direct contrast to, the law and Judaism, and God will not allow us to mix the two, or rather, I should have said, try to do so.
So the word is, “Let us go forth unto Him without the camp, bearing His reproach.” Now, my reader must take notice that this is not a question of salvation – it is not that souls inside the camp are not saved, – indeed it is saved ones inside that are called upon to go outside (and no doubt that many souls are being saved where the gospel is truly preached inside); but it is a question of obedience, and of worshiping God according to the principles of Christianity, or not – surely a very important thing in the eyes of those who love the Lord Jesus.
When the Epistle to the Hebrews was written, there were many thousands of Jews that believed, and they were all zealous of the law, the apostle James, who says so, one of them. (See Acts 21:20). If we compare dates, too, we shall find that God, in long-suffering goodness and mercy, allowed the believing Jews to remain in “the camp” for perhaps thirty years after Pentecost, and so Christianity and Judaism were going on together for some years. (Acts 2:46-47; 21:20-25). But God had not told them to leave it as yet. However, after Paul had gone to Jerusalem, and, persuaded by James, gone to the temple worship again, to please the many thousands of Jews that believed, and had nearly lost his life at the hands of the unbelieving portion of Jewish worshipers, he was sent a prisoner to Rome, and probably from there wrote this very epistle (judging from dates); the Spirit of God using the apostle to call on the Christians in Judaism – Apostle James and all – to separate from the camp altogether; the time having come when God would no longer allow the two to go on, or appear to go on, together.
It was very trying for one brought up as a Jew in Paul’s day to obey this call, and so it is now in these days. But if we love Him who gave Himself for us, is not the way to show our love to Him to obey His commandments and His words? (See John 14:21,23). And if there is reproach promised us on obedience, let us not forget that it is His reproach. He has been through Himself, far worse than any of us ever did or ever will have to do; and His love and His power are with us and for us to carry us through – yes, and even to enable us to rejoice that we are counted worthy to suffer for His sake.
But we are to “go forth unto Him without the camp.” Where, outside the camp, shall I find Him, and in what special way is He to be found there? Christian brother or sister, there is a special place outside the camp where the Lord vouchsafes to be, and to manifest Himself in a very special and peculiar manner to faith, and it is “where two or three are gathered together unto My name, there am I in the midst of them” (Matt. 18:20). The place, then, is the two or three, or twenty or thirty, or two or three hundred, gathered together unto His name; and the special way and manner He vouchsafes to manifest Himself there to faith is not to be described in words. It is to be felt and enjoyed by those who believe and obey His words – His actual, but not, of course, bodily, presence in the midst. O, that all His dear people believed it!
I would add that this does not do away with ministry – divinely appointed ministry – in the church of God. There is a ministry; there are gifts given unto men; there are evangelists, pastors and teachers,”for the perfecting of the saints, for the work of the ministry, for the edifying of (not a sect, but) the body of Christ.” The apostles we have in their writings. (See Eph. 4:11-12). But these gifts are not given to lead Christians in worship; they are as the members of one’s own natural body, for the use of the whole body; but the head is alone that which all are to look to, to lead and guide, especially in worship. And it is Christ who is the Head of His body, the church, of which all true Christians are members (1 Cor. 12:12). And He really does lead by His Spirit where He is owned as present and looked to as in the midst, and given His proper place – the place that only He has a right to, or is qualified to fill.
(Continued from page 220).
(To be Continued).

What God Hath Said - on the Second Coming of Christ and the End of the Present Age: Part 9

Still, whether writing to an assembly or to an individual saint, with the apostle it is the great practical truth. In fact, it is just as opposite to modern thoughts of men as possible. With men, the appearing of Christ is the least practical – their most distant thought. With the apostle, it is the great practical truth – the ever-present theme of hope. He says to his son Timothy, after telling him of the sad departure and iniquity of the professing church in the last days, his concern for him was, “That thou keep this commandment without spot, unrebukable, until the appearing of our Lord Jesus Christ,” (1 Tim. 6:14-16). And again, though misjudged and forsaken, and fully aware of the terrible character of these last days, yet what was the stay of his heart – “Henceforth there is laid up for me a crown of righteousness, which the Lord, the righteous judge, shall give me at that clay: and not unto me only, but unto all them that love His appearing” (2 Tim. 4:8).
And again, writing to Titus, he says, “For the grace of God that bringeth salvation hath appeared to all men.” Ah! this is very precious. God does not expect to find anything, except sin and misery, in a poor sinner. But grace bringeth.
Christ has died. Salvation is all of grace: that gives all and asks for nothing. Then the effect of this grace is to teach us holiness of life in every way, “Looking for that blessed hope and the glorious appearing of the great God and our Savior Jesus Christ” (Titus 2:13). Men now look for death: and so “it is appointed unto men once to die and after death the judgment.” But how blessed the contrast, “So Christ was once offered to bear the sins of many; and unto them that look for Him shall He appear the second time without sin unto salvation” (Heb. 9:27-28). Have you, my reader, this unspeakable consolation, that Christ has once borne your sins on the cross, and consequently, that He having borne the full judgment due to you once, there can be no more condemnation to you? And that He has made full atonement is proved, in that God hath raised Him from the dead. Then you are justified in Him, the risen Christ, from all things; yea, so justified that God says He will remember your sins and iniquities no more. Then how can you be judged for your sins again! Impossible, unless Christ has died in vain. O, what blessed, settled ‘peace this gives to the long-perplexed soul!
But you ask, Shall we not all stand before the judgment-seat of Christ? We shall do so. Yes, it is most happy for us who lived in a world where the nearer a man lives to God the more is he misjudged and hated. Yes, I say, it is most blessed that we shall soon stand before the Beemah or seat of Him who will reward every man according to his works. Yes, while the whole question of sin, and judgment due to sin, has been eternally settled by the blood of the Lamb, yet He who gave His blood for us, has promised that the gift of a cup of cold water shall not lose its reward. ‘To them that look for Him shall He appear the second time, without sin” – no question of sin then, but – “unto salvation.” What encouragement to confidence and patience this gives. “For yet a little while and He that shall come will come and will not tarry” (Heb. 10:37).
Indeed nothing gives the soul more quiet patience, in the midst of sore trial and temptation, than this blessed hope, “Be patient therefore, brethren, unto the coming of the Lord. Be ye also patient; stablish your heart; for the coming of the Lord draweth nigh” (James 5:7-8). “That the trial of your faith, being much more precious than of gold that perisheth, though it be tried with fire, might be found unto praise, and honor, and glory, at the appearing of Jesus Christ” (1 Peter 1:7). “Wherefore, gird up the loins of your mind, be sober, and hope to the end for the grace that is to be brought unto you at the revelation of Jesus Christ” (1 Peter 1:13). “And when the chief Shepherd shall appear, ye shall receive a crown of glory that fadeth not away” (1 Peter 5:4).
Now, is it not strange that, with Scripture so full of this precious subject, men should say it is a dark, mysterions, bewildering subject and that those do well who never look into it? What God says is this, “We have a more sure word of prophecy, whereunto ye do well that ye take heed, as unto a light that shineth in a dark place, until the day dawn, and the day star arise in your hearts.” Men would say our reasoning on the future is light. God says it is all darkness. Men say prophecy is all darkness. God says it is a light. But alas! men will even go so far as to say, “Where is the promise of His coming?” (2 Peter 3:1-15). In this chapter we have a glorious view of the whole future in a general way, right onward to the creation of new heavens and a new earth – more in reference to the world than the church.
One thing is very manifest in all these words of the Spirit of God – the coming of the Lord is not looked at as a mere doctrine. It is either a blessed hope – that is, Jesus Himself is presented as the object of the heart’s utmost desire; or a terrible fact, fast approaching to a doomed world. Of the former, we have a sweet example in the next words before us, “Beloved, now are we the sons of God; and it doth not yet appear what we shall be; but we know that when He shall appear, we shall be like Him; for we shall see Him as He is. And every man that hath this hope in Him purifieth himself, even as He is pure” (1 John 3:2-3). Ah! it is not he that believeth this doctrine – our precious Lord claims the heart. O! is this the one desire and hope of my reader’s heart to be like Him, to see Him as He is? Then sure it is sweet “to abide in Him; that, when He shall appear, we may have confidence, and not be ashamed before Him at His coming” (1 John 2:28). Surely nothing can have a more purifying effect upon the believer, than his constant looking, longing, desiring, the coming of his precious Lord.
And as to the fact of judgment on an ungodly world, even Enoch, the seventh from Adam, prophesied, saying, “Behold the Lord cometh with ten thousand of His saints, to execute judgment upon all,” Jude 14). And how solemn are those words, “Behold He cometh with clouds; and every eye shall see Him, and they also which pierced Him: and all kindreds of the earth shall wail because of Him. Even so, Amen” (Rev. 1:2).
(Continued from page 223).
(To be continued).

Extracts: Selfishness

Selfishness delights to be served and wants to be great. Love delights to serve and is great.
Our hearts are too large to be filled by this poor world and what it has to offer. But Christ is sufficient, and more than sufficient, to fill them to overflowing.
Take time over the Bible daily – unhurried time. Time enough not to think of time. (Psa. 1:7).

Correspondence: The Lord's Will; the Epistle of James; Rom. 5:3

Question 104: Can we find out the Lord’s will in everything by His Word? F. S. D.
Answer: “Thy Word is a lamp unto my feet and a light unto my path.” (Psa. 119:105). “O Lord, I know that the way of man is not in himself: it is not in man that walkway to direct his steps” (Jer. 10:23).
The Word of God teaches us what our manner of life should be in all things, and leads us in dependence upon God, in submission to His will, and in obedience to His Word, but our path itself is found in waiting upon the Lord. Prayer is needed to find out God’s will for us from day to day. In prayer He lays upon our hearts what we should do, and gives us confidence that in some way he will open a door for us. In Acts 16:6, we find Paul and Silas forbidden of the Holy Spirit to preach the Word in Asia. In the 7th verse they tried to go to Bithynia, but the Spirit suffered them not; that is, He hindered them from going. Then, in verses 9 and 10, Paul gets a vision that shows him they are to go to Philippi. So that in these few verses we get various ways of guidance from the Lord.
It is waiting upon God in prayer that gives us true guidance, but when we are guided of God it is always in ways consistent with His holy Word.
Question 105: Is James’ Epistle addressed to the Jews? If so, is it to them all or just to the Christian Jews? E. N. Y.
Answer: James recognizes and addresses the twelve tribes, though scattered.
The Christians are not yet separated from them in his epistle.
The truth in it is what is needed for practical life, and while speaking mainly to the Christians (James 2:1), he also speaks to others, walking in wickedness (James 5:1-6).
The truth as to practical righteousness applies at all times and to everyone.
Question 106: What are the tribulations of Romans 5:3? C. R.
Answer: All that is painful and trying to us can be called tribulations. We do not glory or boast that we have tribulations, but in what they work in us. The exercise of heart is to produce in us the blessed fruits of patience in submission to the will of God. This gives us experience of what God is for us, and hope which ever points us on to a blessed future; and hope maketh not ashamed; because the love of God is shed abroad in our hearts by the Holy Spirit which is given unto us.
What comfort is given us for our wilderness journey; and the proof of His love is the gift of His Son (Rom. 8:32).

Inspiration of the Scriptures: The Bible - its Perfection, Part 5

We have already observed how minutely and accurately the Scripture has been fulfilled as to the birth, life, sufferings, atoning death, resurrection and ascension of our Lord Jesus Christ; and we may be assured that not one jot or tittle shall fail of all that is written concerning His coming for His saints, His glorious appearing, judgments and reign. Few things have so obscured the simplicity of the truth, or been more misleading than the traditions that there will be one general resurrection, and a general judgment, till which no one can have the assurance of his eternal safety. The reception of such doctrines, which are unknown in Scripture, can only be accounted for by persons going to the Reformation as the source of orthodoxy, instead of to the Word of God, to that which was at the beginning, and has been treasured up for us in the inspired writings. As to the Reformation we have much to thank God for in the wonderful recovery of the grand foundation truth of justification by faith, and the diffusion of copies of the Bible for general reading; but with these and other blessings, a mass of doctrines were received from Papists, which are held by many Protestants to this day; and among them that the church will convert the world; that Christianity will triumph over infidelity and every opposing power. That a time will come when “the earth shall be full of the knowledge of the Lord, as the waters cover the sea,” is unquestionable, as we have seen; but in the same chapter we are told it is to be brought about by Christ reigning in “righteousness,” and not therefore by the preaching of the “gospel of the grace of God” (Isa. 11). From Scripture we learn that Christ is “heir of all things,” that, as the glorified Man, all things are to be put under His feet, and that all judgment is committed unto Him because He is the Son of Man. His rightful place also, as having died for all, and triumphed over death and Satan is, that to Him every knee should bow in heaven, in earth, and under the earth, and that every tongue should confess that Jesus Christ is Lord, to the glory of God the Father. How is it possible that these things can be otherwise than strictly and accurately fulfilled by His power, whereby He is able to subdue all things unto Himself! It is, we repeat, God’s unfailing faithfulness to His own Word that comforts our hearts and strengthens our confidence in Himself, for “He is faithful that promised: He cannot deny Himself.” Was He ever truer to the soul that seeketh Him than now? Was it ever a greater reality to souls than now, that, “Where two or three are gathered together in My name, there am I in the midst of them”? (Matt. 18:20). Is it not to thousands in these days as real to their hearts as if they saw Him in the midst? O the untold blessedness of having to do with the God. and Father of our Lord Jesus Christ, receiving His testimony, and setting to our seal that God is true!
Our gracious God having caused His Word to be written for our instruction, how could we expect it otherwise than its being accurately fulfilled according to His own will? Nor is it surprising, because it is His Word, the revelation of His own mind, that both Old and New Testaments give a note of most solemn warning to any who add to it or take from it. (Deut. 4:2; 12:32; Prov. 30:6; Rev. 22:18-19) Nothing surely could more truly authenticate the sacred writings, or more thoroughly show the infinite perfection of the Bible.
“What Christ hath said must be fulfilled:
On this firm rock believers build:
His word shall stand, His truth prevail,
And not one jot nor tittle fail.”
(Continued from page 229).

Their Sins and Iniquities Will I Remember No More: Hebrews 10:17

What though the great accuser roar
Of ills that I have done,
And oft they grieve my spirit sore,
God looks but findeth none.
Christ’s precious blood has done its work,
For all my sins He bore
When hanging on the accursed tree
Beneath God’s judgment sore.
God’s waves of wrath and judgment broke
Upon His holy head,
And when that cup of wrath was drained,
He’s numbered with the dead.
Of such eternal value was
That work upon the tree,
That God now says my sins are gone,
Not one that He can see.
O love divine that gave for me
Thy precious blood to save,
To set me free from all my sins,
By which I was enslaved.
From this poor heart of mine should flow
Unceasing songs of praise,
Begun on earth, but sweeter far
Through heaven’s eternal days.

Practical Conversations With Our Young People: Assembling Ourselves Together

Assembling Ourselves Together
It is quite certain that those who are wholehearted for Christ desire to be in His company. They instinctively wend their way to the spot where He is known to be.
Is there such a spot on earth? Yes.
“Where two or three are gathered together in My name, there am I in the midst of them.” Mathew 18:20.
No one who is truly conscious of the greatness and excellency of His person and of the blessedness of communion with Him, can be willingly absent from that favored place. A neglected Lord’s table and a neglected prayer meeting, speak aloud of a Laodicean state of the heart towards Him. We read that of old “They continued steadfastly in the apostles’ doctrine and fellowship, and in breaking of bread and in prayers” (Acts 2:42). Alas! that there should be such a lack of continuing steadfastly now.
Does not the Lord say to God, “In the midst of the church will I sing praise unto Thee” (Heb. 2:12), and can we suppose that He fails to notice whether we are there, or not, to join in the song He leads? In the coming day of review before the judgment seat of Christ, how shall we like the disclosure that self-indulgence, a little unfavorable weather, or a passing tiff with a brother or sister in Christ, has outweighed with us all the mighty motives for a loving response to His wish, “This do in remembrance of Me”? (Luke 22:19).
It is deeply humbling to think that any who have tasted the Lord’s love, can take advantage of not having to work on the Lord’s day, to spend its morning hours in bed, and that others can excuse their absence from the meetings on the ground of visiting, or receiving visits from, friends. Priceless opportunities of gratifying the heart of the Lord, and of showing our attachment to Him, in the scene of His rejection, are thus wasted and lost.
It is mere mockery to repeat, “Come, Lord Jesus,” and use glowing expressions of desire to be with Him in glory, if, by our absence from His table, we betray our indifference to His presence here.
Beloved, “It is high time to awake out of sleep” (Rom. 13:11). May we take to heart the solemn and impressive exhortation of the Word, “Not forsaking the assembling of ourselves together, as the manner of some is; but exhorting one another: and so much the more as ye see the day approaching” (Heb. 10:25).
Closely connected with the foregoing is the question of TIME.
Where is our reverence for the Lord, or our sense of His grace – where our responsive love, if we come with lagging steps, five, ten, or fifteen minutes after He has taken His place in the midst of His own?
It was easy, in the freshness of first love, to come early to the place where He manifests Himself in such a peculiarly blessed way. Excuses were not made. Has He become less precious?
The moments we may thus spend together with Him on the earth stained with His blood, are swiftly passing away. Let us not willingly lose one of them.
It is touching to remember that no thought of all the sufferings that awaited Him, of Himself presently becoming the true passover, “sacrificed for us,” delayed the Lord’s appearing at His last paschal feast.
“WHEN THE HOUR WAS COME, He sat down, and the twelve apostles with Him.” (Luke 22:14).
O! for a holy eagerness to be where He is!

Questions for October

Always give Scripture for answers, as well as your own expressions.
1. Give scriptures, showing that the Lord Jesus possessed a human nature.
2. What do we get in the Scriptures as to His incarnation?
3. Give the purpose of the Lord in coming into the world.
4. In what different aspects do the gospels show forth the Lord?
5. Give five scriptures, showing that Christ became man.

Answers to Questions for August

ANSWERS TO QUESTION 1.
What is required for true service?
(a) For true service we must have a single eye. “Behold to obey is better than sacrifice, and to hearken than the fat of rams” (1 Sam. 15:22).
The Lord Jesus was the perfect servant, and in all His path down here “He pleased not Himself” (Rom. 15:3), but in all things sought His Father’s will (John 5:30; 6:38), and for true service we must fulfill His will. Like Mary, we must first sit at His feet and hear His words, to know what His will is for us (Luke 10:39).
(b) I think obedience is required for true service, not with eye-service, as men-pleasers, but as the servants of Christ, doing the will of God from the heart” (Eph. 1:6).
ANSWERS TO QUESTION 2.
What should our object be in service?
(a) Our object should be Christ Himself. “For to me to live is Christ” (Phil. 1:21). “That I may win Christ” (Phil. 3:8).
(b) Our object should be to follow the Lord. “If any man serve Me, let him follow Me, and where I am, there shall also My servant be: if any man serve Me, him will My Father honor” (John 12:26).
ANSWERS TO QUESTION 3.
Why should we serve the Lord?
(a) This is the way we can show our love for the Lord and express in some small measure our gratitude for all He has done for us (Luke 18:22).
He has delivered us from this world (Gal. 1:4), and then left us here to live for Him (John 17:18).
“And that He died for all, that they which live should not henceforth live unto themselves, but unto Him who died for them, and rose again” (2 Cor. 5:15).
“God hath given to us the ministry of reconciliation....now then we are ambassadors for Christ” (2 Cor. 5:18,20).
(b) We should serve the Lord for what He has done for us. “We love Him because He first loved us” (1 John 4:19). “Christ died for our sins” (1 Cor. 15:3).
ANSWERS TO QUESTION 4.
Why is it our reasonable service to present our bodies as a living sacrifice?
(a) “Ye are not your own, for ye are bought with a price, therefore glorify God in your body and your spirit which are God’s” (1 Cor. 6:19-20).
This is the only way for the Christian to be truly happy, for “There is only joy as we seek His pleasure, There is only rest as we do His will.”
(b) “That ye may prove what is that good, and acceptable, and perfect will of God” (Rom. 12:2).
ANSWERS TO QUESTION 5.
Give five scriptures, showing that God is unwilling that any should perish.
(a) 1 Timothy 2:4; 2 Peter 3:9; Ezekiel 33:11.
(b) John 3:16,17; 1 John 4:14. John 3:16; Isaiah 45:22; Matthew 11:28; Matthew 18:14; 2 Peter 3:9.

My Desire

Take Thou my life and let it be,
Completely given up to Thee.
Take Thou my mind and let it rove,
Through those bright scenes on things above.
Take Thou my lips and let them tell
Thy love immense, unsearchable.
Take Thou my hands, and set them free
To labor in this world for Thee.
Take Thou my feet, and let them run
Till all my work for Thee is done.
Until my path on earth is trod,
Yielding my members unto God;
May this my happy service be,
While waiting, Lord, Thy face to see.

Truths for Young Christians: Christian Devotedness, Part 1

If there is one thing of importance now, it is Christian devotedness. I do not separate this from Christian doctrine, but found it on that. I do not surely separate it from the presence and power of the Spirit (one of the most important of these doctrines), for it is produced by it. But Christian devotedness founded on the truth, and produced by the power of the Spirit, I believe to be of the utmost importance for the saints themselves and for the testimony of God. I believe surely that doctrine is of deep importance now; clearness as to redemption, and the peace that belongs to the Christian through divine righteousness; the presence and living power of the Comforter sent down from heaven; the sure and blessed hope of Christ’s coming again to receive us to Himself, that where He is we shall be also, that we shall be like Himself seeing Him as He is, and that if we die we shall be present with Him; the knowledge that risen with Him we shall be blessed not only through but with Christ; the deep practical identification with Him through our being united with Him by the Holy Spirit. All these things, and many truths connected with them, held in the power of the Holy Spirit separate us from the world, shelter the soul (by the spiritual possession of Christ glorified, the conscious possession of Christ) from the cavils of current infidelity, and give a living spring to the joy and hope of the whole Christian life. But the expression of the power of them in the heart will manifest itself in devotedness.
Christianity has exercised a mighty influence over the world, even where it is openly rejected, as well as where it is professedly received. Care of the poor and the supply of temporal wants have become recognized duties of society. And where the truth is not known and Christianity is corrupted, diligent devotedness to this, on the false ground of merit, is largely used to propagate that corruption. And even where infidelity prevails, the habits of feeling produced by Christianity prevail, and man becomes the object of diligent, though often of perverted care. The testimony of the true saint surely should not be wanting where falsehood has imitated the good effects of truth. But there are higher motives than these; and it is of the true character of devotedness I would speak.
I accept as the general rule that, any special call of God apart, Christians should abide in the calling wherein they are called. This is only the place of their walk, its motives and character are behind. These are summed up in one word – Christ. He is at once the life and the object or motive of life in us, giving thus its character to our walk. “To me,” says the apostle, “to live is Christ.” There are two great parts of divine life of which devotedness is one. Both are infinite and unspeakable privileges for us and both perfected by, manifested in, Christ. The one God Himself, the other the actings and display of His nature, as love, the divine witness of His nature which is love. This was seen in Christ. His communion with His Father was perfect, as was His desire to glorify Him. But He was the display, at all cost to Himself, of divine love to men. These could not be separated in His soul. His Father was His continual delight and object, His exercise of love and display of His Father, of the divine nature by it, constant and perfect. But this was His devotedness.
Another principle must be added to this to complete those which governed His walk: undivided obedience to His Father’s will, His having that will for His constant motive. Love to the Father and obedience to Him gave form and character to His love to us. And so it is with us, only that He Himself comes in as the more immediate object, but this in no way hindering the display of the divine nature in love. “Be ye imitators of God as dear children. and walk in love, even as Christ hath loved us and given Himself for us, an offering and a sacrifice to God for a sweet-smelling savor.” Note here the fullness of motive and character which is shown, and how high and blessed that motive and character is. We are followers and imitators of God. We walk in love as Christ loved us. It is the exercise of divine love as displayed in Christ. There is no stint in it. He gave Himself, nothing short. of Himself, wholly; a principle often repeated as to Christ, His love to us, for He gave Himself for us. Yet God was the object and motive constituting its perfection: “an offering and a sacrifice to God for a sweet-smelling savor.” It is thus we are called to walk, to imitate God, to follow Him as He displayed Himself in Christ.
If it be blessed to joy in God, who is love, it is blessed to follow Him in the love He has exercised. Yet as displayed in Christ as a man, it has God Himself for its object: and so with us. The love that descends down from God working in man rises up always towards and to God as its just and necessary object. It can have nothing lower as its spring, towards whomsoever it is exercised. All the incense of the meat-offering was burnt on the altar, however sweet the savor to others. This constitutes, as I have said, its essential character and excellence; nor do its just actings in us come short of its actings in Christ. “Hereby,” says ‘John, “know we love, because He laid down His life for us; and we ought to lay down our lives for the brethren.” There is no question of any cup of wrath for us. Here Christ stood, of course, alone, but all self-sacrifice displayed in Him we are called upon to display, as having His life, Himself, in us.
But I will consider this a little more methodically before I press it hortatively on my brethren.
As to my reward, as motive, or merit, it is clear that any such thought destroys the whole truth of devotedness, because there is no love in it. It is self, looking like “James and John,” for a good place in the kingdom. Reward there is in Scripture, but it is used to encourage us in the difficulties and dangers which higher and truer motives bring us into. So Christ Himself, “who for the joy that was set before Him endured the cross, despising the shame.” Yet we well know that His motive was love. So Moses: “He endured as seeing Him who is invisible, for he had respect to the recompense of reward.” His motive was caring for his brethren. So reward is ever used, and it is a great mercy in this way. And every man receives his reward according to his own labor.
The spring and source of all true devotedness is divine love filling and operating in our hearts: as Paul says: “the love of Christ constraineth us.” Its form and character must be drawn from Christ’s actings. Hence grace must first be known for oneself, for thus it is I know love. Thus it is that this love is shed abroad in the heart. We learn divine love in divine redemption. This redemption sets us, too. in divine righteousness before God. Thus all question of merit, of self-righteousness, is shut out, and self-seeking in our labor set aside. “Grace,” we have learned, “reigns through righteousness unto eternal life by Jesus Christ.” The infinite perfect love of God towards us has wrought; has done so when we were mere sinners; has thought of our need; given us eternal life in Christ when we were dead in sins-forgiveness and divine righteousness when we were guilty; given us now to enjoy divine love, to enjoy God by His Spirit dwelling in us, and boldness in the day of judgment, because as Christ, the Judge, is, so are we in this world. I speak of all this now in view of the love shown in it. True, that could not have been divinely without righteousness. That is gloriously made good through Christ, and the heart is free to enjoy God’s unhindered love: a love shown to men in man. For the very angels learn “the exceeding riches of His grace in His kindness towards us in Christ Jesus.” This knits the heart to Christ, bringing it to God in Him, God in Him to us. We say nothing separates us from this love. The first effect is to lead the heart up, thus sanctifying it: we bless God, adore God, thus known; our delight, adoring delight, is in Jesus.
(To be Continued).

Scripture Study: Matthew 25

The servant at the end of the previous chapter pictures collective responsibility; in this parable. it is individual readiness to go in with the Bridegroom, and pictures the end of this present period of time by the Lord’s coming as a Bridegroom for His people. He could not present the bride here, for it is individual responsibility represented by ten virgins.
Matthew 25:1. Then shall the kingdom of heaven be likened unto ten virgins, which took their lamps, and went forth to meet the Bridegroom, and five of them were wise and five were foolish: They that were foolish took their lamps, and took no oil with them; but the wise took oil in their vessels with their lamps. They all had lamps, that is profession; the difference is, the wise had oil in their vessels with their lamps; they all seem the same till the testing time comes.
They all went forth to meet the Bridegroom, they all are nominally in the same position, the position of the church is outside this world’s religion and ways, it goes forth.
Matthew 25:5. While the Bridegroom tarried they all slumbered and slept, they lost their separate character and went in to sleep again in worldly religion and ways, but still keeping their profession. But though the church has lost its heavenly character, saints are always saints though sleeping among the dead (Eph. 5:14). A worldly saint is very much like the world.
This parable corresponds to the professing church and what has happened in it.
Matthew 25:6. At midnight there was a cry made, “Behold the Bridegroom (cometh), go ye out to meet Him.” “Go ye out to meet Him” had to be said again to call them out of their wrong state. Then all those virgins arose and trimmed their lamps.
The thought of the Lord’s return wakens them up and activity begins. In trimming their lamps the foolish find out they lack the oil, their lamps (or torches) were going out (see margin). An empty profession gives no light. The foolish need oil, but seek it from man, their neighbors; they cannot find it there. Their religion does not satisfy them now; man might think them all right, but they have no grace of life or the Spirit; the reality is wanting; they must have to do with God. If they go to Him they can buy it without money and without price.
Since the cry has gone forth, “Behold the Bridegroom,” what religious activity is seen, but nothing avails except Christ and His finished work. It is having done with self and resting on Him as the Savior that is the way to obtain this oil in the vessel. Not even God’s people can impart it; only the Lord Himself. How near His coming is now.
Matthew 25:10. And while they went to buy, the Bridegroom came, and they that were ready went in with Him to the marriage; and the door was shut. The foolish have no part in this, they are left out and kept out, for the door was shut.
Matthew 25:11. They plead, “Lord, Lord, open to us,” but He answers “Verily, I say unto you, I know you not.” It is too late. Church members, professing Christians, but not born of God and not having the Spirit of Christ, these are beyond hope when left behind. Do not believe Satan’s lie; believe God’s Word, and come to Christ now.
The midnight cry has gone forth, the Lord is coming. He may come today. Are you ready? Are you sheltered from the judgment of God by the work of Christ? Only those who have the oil go in with the Bridegroom to the marriage. He refuses to own all others. They had no light, their lamps were useless, they had no title to the feast like those who went in with the Bridegroom.
Quite true, the faithful ones had fallen asleep for a while, but it was a joy to them to be wakened, for they were ready, they had the oil, and the truth was revived in their souls.
None of the Lord’s own will fail to hear the shout (1 Thess. 4:16). All His own will go in with Him.
Will you be there or will you be left out?
Watch, therefore, for ye know neither the day nor the hour. This is a warning. The Lord may come at any moment. We are to be watching as well as waiting for Him. The verse ends here. (See JND or Revised Trans).
Matthew 25:14. In this parable each servant’s individual faithfulness is tested. It is as a man traveling into a far country, who called his own servants and delivered unto them his goods. And unto one he gave five talents, to another two, and to another one; to every man according to his several ability; and straightway took his journey. The Lord has gone into heaven, the goods to trade with are the gifts He gives for His service. The Lord notices the ability or capacity of each vessel, and gives accordingly. (Eph. 4:7-8,11). The servant is fitted by the Lord for what He gives him to do, and he needs no other authority than the Lord’s to serve Him. The true servant can count on the goodness and love he has seen in his Master, and labors in confidence of His approval, counting on Him to bless his labors. In this the wicked and slothful servant fails; he misjudges his Master, and has no confidence in Him to serve Him.
Matthew 25:19. After a long time the Lord of those servants cometh, and reckoneth with them. And so he that had received five talents came and brought other five talents, saying, Lord, Thou deliveredst unto me five talents; behold I have gained beside them five talents more. His Lord said unto him, Well done, thou good and faithful servant; thou hast been faithful over a few things, I will make thee ruler over many things; enter thou into the joy of thy Lord.
Matthew 25:22-23. He goes over the same to the one with two talents and He gets the same blessed word of approval; they share alike the joy of their Lord.
Matthew 25:24. The third one had digged in the earth and hid his Lord’s money; then when called to account, he calls his Master a hard man, dealing in unrighteousness, and he was afraid. What did he know of the grace of his Master? Nothing at all. After all, he is a wicked man, and because he is in the profession of a servant, he is a wicked and slothful servant. What he has is taken from him and given to the one who gained the ten talents; but for him there is outer darkness, weeping and gnashing of teeth. What a sad end for one professedly serving Christ. (1 Cor. 3:17).
“That which we gain spiritually here, in spiritual intelligence and in the knowledge of God in power, is not lost in the other world. On the contrary, we receive more, and the glory of the inheritance is given us in proportion to our work. All is grace” (J. N D. Synopsis. Luke 19).
There is also danger of a true child of God neglecting the ministry given to him of the Lord. (See Col. 4:17).
Matthew 25:31. We are again looking at the coming of the Son of Man as King, coming in heavenly glory, and all the holy angels with Him, to sit upon the throne of His glory. In 24:30, 31, He appeared with the great sound of a trumpet calling together His elect earthly people. Here it is the living nations who had taken part in helping or persecuting the suffering Jews during the tribulation.
There is no resurrection of the dead here; the unsaved dead will be raised after the thousand years of Christ’s reign are over. This is at the commencement of it.
The gospel of the kingdom has been preached; in the sheep we see those who received it; in the goats those who did not believe in the Lord’s coming as King. Now He divides them asunder, as a shepherd divides his sheep from the goats. The King speaks: “Come, ye blessed of My Father, inherit the kingdom prepared for you from the foundation of the world: (God’s purposes for the church are before the foundation of the world). For’ was an hungered, and ye gave Me meat; I was thirsty, and ye gave Me drink; was a stranger, and ye took Me in; naked, and ye clothed Me; I was sick, and ye visited Me; I was in prison, and ye came unto Me.”
And in answer to their question when did they do these things, He answers: “Verily, I say unto you, inasmuch as ye have done it unto the least of these, My brethren, ye have done it unto Me.”
The church is the Lord’s brethren; the Jews here are the King’s brethren. These sheep had helped them during their sufferings from their enemies. Their love to His people speaks to Him of their faith in Him.
How solemn the word to those on the left hand: “Depart from Me, ye cursed, into everlasting fire, prepared for the devil and his angels;” they were enemies of His brethren, and thus His enemies. “Inasmuch as ye did it not to one of the least of these, ye did it not to Me. And these shall go away into everlasting punishment; but the righteous into life eternal.” These righteous possess eternal life on earth under the reign of Christ, and on the new earth in the eternal state. For the wicked it is the lake of fire forever.

What Is the Camp? Hebrews 13:12-13, Part 3

He who places the members of the body in it as it pleases Him, uses evangelists, pastors and teachers, when and where He chooses to do so; they looking to Him for guidance where to go, and where and how long to remain, to whom they are alone responsible for the use of their gift or gifts.
But in worship, we come together, not to hear a gospel address, not to be taught by a teacher, not to be exhorted by a pastor, but to give, as led of the Lord in the midst by His Spirit, praise, adoration, and thanksgiving to our God and Father, or it may be. to the Lord Jesus Himself – something that requires no special gift in any one, but something that every true Christian walking with God has in him, and which God is alone worthy to receive, that is, worship.
It may be one or two or three simple, but true-hearted souls, may each in turn be led, in simple, unlettered words, to be the mouthpiece of all the Christians present; but it will be so, not because more fitted or more gifted by even spiritual gifts, or because called upon by any one to do so, but simply because His Spirit is leading, to whom we are to look and own as our Head, our High-Priest, our Minister of the true tabernacle, which the Lord pitched and not man, our Lord Jesus Christ; through whom, too, our songs and words of praise come up acceptable unto God.
And this system of worship will never be done away with. It has begun on earth, though, we being still in bodies of humiliation, it is never what it should be or will be. When we get the redemption of our bodies – when no longer “through a glass darkly, but then face to face,” knowing as we are known – when, no longer scattered and divided, and mixed up with the world, as many are, in worship, all Christ’s blood-bought ones are gathered around Himself in glory, then shall worship begun on earth go on there in glory forever, and be what it ought to be fully.
The holiest we enter
In perfect peace with God,
Through whom we found our center
In Jesus and His blood;
Though great may be our dullness
In thought and word and deed,
We glory in the fullness
Of Him that meets our need.
Much incense is ascending
Before th’ eternal throne;
God graciously is bending
To hear each feeble groan;
To all our prayers and praises
Christ adds His sweet perfume,
And love the censer raises,
These odors to consume.
(Read the Epistle to the Hebrews prayerfully with this).
(Continued from page 246).
(Concluded).

What God Hath Said on the Second Coming of Christ and the End of the Present Age: Part 10

In the seven addresses to the churches, these are searching words in the midst of much outward profession – “Hold fast till I come” (Rev. 2:25). And again, “If therefore thou shalt not watch, I will come on thee as a thief, and thou shalt not know what hour I will come upon thee” (Rev. 3:3). And again, “Behold I come quickly: hold that fast which thou halt, that no man take thy crown” (Rev. 3:11). Woe be to the carnal worldly professor. who sets light by these solemn warnings, so suited to the state of the church. during its seven-fold history, of the things that are now. In Revelation 4 things are revealed that will take place after these – “I will show thee things that must be hereafter,” or after these (Rev. 4:1). And then, in blissful vision, in Revelation 4 and 5, the redeemed are seen gone from the earth, and seated around the glorified Lamb.
Revelation 6 to 19 contain the words of the Spirit of God as to the end of the present age, the church being at that time taken to be with the Lord. “Then sets in the great and terrible day of the Lord. Peace is taken from the earth that they should kill one another” (Rev. 11:4). Who can describe the terrors of that fearful day!
I do not go into the detail of the woes and judgments of this day of vengeance, answering to all the prophets have said, and all the passages we have read, in the words of Jesus, as to this time of great tribulation, such as never was – no, and never will be again. More gifted servants of the Lord have written on these things.
I may just remark, in perfect keeping with every other part of Scripture. the whole scene becomes Jewish in character during this day of wrath. Satan is cast down to the earth in Revelation 12 and persecutes the Jewish remnant. In Revelation 13 Satan is worshiped, and the head of the Roman Empire, to whom Satan gives his power, is worshiped. (Rev. 13:3-8). In Revelation 17 the ecclesiastical apostacy, having lost her temporalities in the empire. now in her last most blasphemous character, sits upon the beast; that is, guides the imperial head in its last acts of wickedness. The Roman Empire which was, and is not, shall again appear in its most terrible character. Ten kings are seen confederate with the imperial head; and as the Roman people formed the empire, when it was, so assuredly they shall be again. It would seem, however, not by conquest; for the ten kings or kingdoms give their power to the beast. The reconstructed empire for a time carries the whore; but being infidel at heart, throws her off, and the ten kingdoms which will exist in that day “hate the whore, and shall make her desolate and naked, and shall eat her flesh and burn her with fire” (Rev. 17:16). Revelation 18 is occupied with a description of her burning.
The church having been with the Lord from Revelation 6 now returns with the Lord in Revelation 19. The full number of the first resurrection being completed in the beginning of Revelation 20, then takes place the millennium, or thousand years’ rest. with Christ. The rest of the dead live not until this thousand years’ rest is completed. During this thousand years, every promise of blessing to this earth will be fulfilled – Satan bound – sin not allowed, but immediately judged. Then comes the end. Satan is loosed a little while; and then the great judgment of the dead takes place (Revelation 20:11-15). And this over, the eternal state of inexpressible blessedness sets in – new heavens and new earth,
Where God shall shine in light divine.
In glory everlasting.
I will conclude with the last closing sounds of the words of the Lord Jesus on this solemn subject: – “Behold I come quickly: blessed is he that keepeth the sayings of the prophecy of this book” (Rev. 22:7). “And behold I come quickly; and My reward is with Me, to give to every man according as his work shall be” (Rev. 22:12). “He which testifieth these things saith, Surely I come quickly. Amen. Even so come. Lord Jesus” (Rev. 22:20).
(Continued from page 251).
(Concluded).

Correspondence: Sabbath; Giving Thanks Always; John 21:11

Question 107: Why was the Sabbath changed from Saturday to Sunday? L. S. H.
Answer: The names of the days of the week, including Saturday and Sunday, are heathen names.
In the Word of God the days are numbered.
The seventh day (Saturday), is the day given by God to Israel as the Sabbath (Ex. 20:8-11; 31:15-17).
The Jews under the law professed to keep it; they blamed the Lord Jesus for doing works of mercy on the Sabbath day (John 9:16). They had rejected Him, the Lord of the Sabbath, whose pleasure it was to do good to men. He did not honor their keeping of the Sabbath, for they were living in sin, and He could not be bound by the law which He had given, from doing works of mercy (Mark 2:23-28). He wrought with God, His Father (John 5:17). In His death He was the whole Sabbath day in the grave. Then He arose triumphant from among the dead on the first day of the week (Matt. 28:1; Mark 16:9). It was the same day He came in the midst of His disciples. (John 20:19). It was upon the first day of the week the Holy Spirit came down and formed the church, (Pentecost means fifty), fifty days after Christ’s resurrection. Leviticus 23:16 calls it the morrow after the Sabbath. It was on the first day of the week the disciples assembled to remember the Lord in the breaking of the bread and in drinking of the cup. The Apostle John, by the Spirit, gave it its name when he wrote: “I was in the Spirit on the Lord’s day” (Rev. 1:10).
Christians, intelligent in the Scriptures, can see that the Sabbath is one of the shadows of things to come. It is a shadow of God’s rest when sin will be done away. In the Millennium Israel will again observe it (Ezek. 46). But Christians are not under law (Rom. 6:14), they do not keep the Sabbath, the Sabbath was never given to them. They have the first day of the week, the Lord’s day, and if the heart is right, they will use it for the Lord, and not for worldly gain or pleasure.
The church, set up at the beginning as God’s witness on earth, soon fell into the world’s ways and worldly religion, and adopted much of paganism and Judaism. They keep days and ordinances and put themselves under law (Gal. 3:10; 4:10-11; Col. 2:20). Calling the first day of the week sabbath is only an invention of man, and is not according to the Word of God.
“Let us therefore go forth unto Him without the camp, bearing His reproach” (Heb. 13:13).
Question 108: What is the difference between “Giving thanks always for all things” (Eph. 5:20), and “In everything give thanks” (1 Thess. 5:18)? S. J.
Answer: I do not know of any practical difference except the viewpoint of the different epistles, and in this we might include Colossians and Philippians.
What a benefit to the soul to be filled with the Spirit, as in Ephesians, enjoying our heavenly portion; and how it cheers us on our pilgrim way, as in Thessalonians, to know the hand and heart that guides us along, to rejoice evermore, to pray without ceasing, and in everything to give thanks, doing the will of God.
And does not Colossians make much of Christ to our souls? (Col. 2:7; 3:11-17). “Be thankful.” We will be if we let “the Word of Christ dwell in us richly” and do all things in the name of our Lord Jesus Christ, giving thanks to God and the Father by Him.
But Philippians 4:6 tells how to gain such experience, for there are many and great difficulties to overcome, before we can be filled with thanksgiving. Notice then – we begin with prayer and continue until in deep supplication we make our requests known unto God; then the soul receives its answer, and is clothed (or garrisoned) with peace that passes all understanding. O, that our hearts and minds might be kept in this lowly, dependent, thankful condition.
Question 109: What does the number 153 in John 21:11 mean? M. C.
Answer: We are right in saying some numbers in Scripture have meaning, but we must avoid letting our imagination put on meaning. (See 2 Cor. 10:5).
We are safe in drawing lessons in accordance with all Scripture. So, while we do not know any meaning to the number 153, we see that it represents a multitude (ver. 6), and we can think how precious we are to the Lord who has numbered us, yes, even the hairs of our head are all numbered. He knows each of us.
In Luke 5:6 the net brake, here the net is not broken. It has been pointed out that now the gospel net breaks but at the beginning of the Millennial reign of Christ the net will not break. When the disciples came ashore they found that the Lord had some fish there, already caught; these represent the believers saved through the tribulation period, what is called the (believing) remnant of Israel.

Inspiration of the Scriptures: The Bible - its Blessedness, Part 1

It is impossible to tell out in human language the full blessedness of having the holy Scriptures. But what is most striking in it is the revelation it gives us of God, so that we now know Him not only as Creator, and one who is kind to the unthankful and the unholy in providing for His creatures (for “His tender mercies are over all His work;”), but we also know Him in the exceeding riches of His grace as a Savior-God, in and through our Lord Jesus Christ. Having received “the words of God,” and “the Spirit of God,” we know that we are brought to God, and are “in the light as He is in the light.” Precious grace! It is not merely that we have title to glory through the blood of the cross, but we are brought to God who is “light” and “love” in Christ, who is our Life, Righteousness, and Peace.
Before the death of Christ, God was not so revealed. Till Jesus the Son of God came, God was hid behind a veil, and little known except by His acts; then Christ revealed the Father, and made Himself known as the Son, and on leaving, promised to send the Holy Spirit to abide with us forever. “No man hath seen God at any time; the only-begotten Son, who is in the bosom of the Father, He hath declared Him.” The Son, the effulgence of His glory, has fully manifested God in flesh. We have the Father so perfectly represented in the Son, that He could say, “If ye had known Me, ye should have known My Father also.... he that hath seen Me hath seen the Father”; and He declares that His rejecters were guilty, and had no cloak for their sin, because as He said, Ye have “both seen and hated both Me and My Father” (John 1:18; 14:7,9; 15:24).
Though God had been so far made known in Old Testament times, that He visited Adam and Abraham, and dwelt among His redeemed earthly people; yet it was not till the Savior’s baptism that God, in the plurality of Persons – Father, Son, and Holy Spirit – was made known. The Spirit came down as a dove, and abode on the spotless One, and the voice from heaven, “This is My beloved Son, in whom I am well pleased,” manifested also the personal glories of the Father and the Son. From that time the unfathomable blessedness of knowing God as Father, Son, and Holy Spirit – one God – abounds in Holy Scripture; and the believing knowledge of redemption, through grace, according to divine counsel and eternal purpose, has brought us into conscious relationship with God as our Father, with the Son as our eternal life, and the Spirit given to guide us into all the truth, to strengthen us with might in the inner man, and also as the anointing, seal, and earnest of our inheritance.
It is because the Scriptures give us God’s revelation of Himself and of His mind and will. and so constantly testify of Christ, that by the teaching and ministry of the Holy Spirit, they are the food of our souls. And here observe, it is not our own thoughts about Scripture, or our reasonings about it, or opinions of it, or deductions from it, but what God says. “It is written,” was the word so often uttered by our adorable Savior and His apostles; and we may be assured that we can only “resist” Satan by being “steadfast in the faith.” And what is this, but believingly holding and using Scripture as the Word of God? How else could we be steadfast in the faith?
Again, we see the untold blessedness of the Scriptures in giving us divine intelligence as to ourselves, our state, our path, our circumstances, and everything around us, as to Jews, Gentiles, and church of God. They open up to us the past, present, and future; things heavenly and earthly, things temporal and eternal,. the two Adams and all in connection with them as heads of races; and the curtain is so drawn now and then, that the Spirit-led soul can survey the coming glories and their felicity, and also solemnly contemplate the infernal regions of unending misery and punishment.
In the Old Testament, we have the sons of Israel, a people God called out to Himself; also the Gentiles, with promise that God’s blessing should even reach out to them – that in Christ, the Seed, all nations should be blessed. But the church, the body of Christ, was not revealed there. The prophets went from “the sufferings of Christ” to “the glories which should follow,” and entirely passed over the marvelous work of forming and removing the church to her destined heavenly glory, before the Lord comes out in blessing to His ancient people, and judgment of the quick and the dead at His appearing and kingdom. Typical intimations and shadows there were now and then of Christ and the church as in Adam and Eve, Isaac and Rebecca, Joseph and Asenath, Moses and his wife; but the assembly as “the body of Christ” on earth, united to Christ the Head in heaven, and formed into one body by the gift of the Holy Spirit, was not revealed till Paul was called by divine grace. Ephesians 3 plainly shows this, and that the mystery of the church was “hid in God,” “not made known,” “kept secret since the world began.” (See also Rom. 16:25-26; for “scriptures of the prophets” read “prophetic scriptures.”)
Believers are now, by the Holy Spirit, in union with Christ ascended; for “by one Spirit are we all baptized into one body.” “He that is joined unto the Lord is one spirit.” All this most blessed workmanship of the Holy Spirit, His present ministry through gifts bestowed by Christ ascended, all the affection and care of Christ for His assembly, and the perfection of the Father’s love to His children, loving them as He loved His Son, are richly and blessedly brought to us through the apostolic writings, and especially by those of Paul, who was emphatically a minister of the church or assembly (Col. 1:23). This, therefore, gives them a sacred charm to the believer, and through faith they necessarily produce a walk of separation and devotedness to the Lord.
(To be Continued).

An Hour of Prayer

How sweet the hour alone with God,
In earnest prayer we spend,
Alone with Him who knows our needs,
As friend would plead with friend.
We tell out all our needs to Him,
Assured that He will hear
The faintest, feeblest cry we raise,
Will reach His holy ear.
No care too great, no need too small,
He would not have us tell,
E’en though assured we are, those needs
To Him are known so well.
Then oft from toil and conflict here,
Our hearts would turn aside
To be in company with Him
Who once was crucified.
As we in faith to Him draw nigh
And make our wishes known,
He bends His ear to hear our cry,
Though high upon the throne.
No prayer of faith will ever be
By Him our God unheard.
The answer’s sure to come,
For this He tells us in His Word.
A little while He may defer
And hold the answer back,
But come it will, the answer’s sure,
Not one part will it lack.
And oft the answer does not come
In ways we’re looking for,
For He would have us know that He
Has many an open door.
Sometimes He answers right away
And proves His presence nigh,
And to our troubled spirits sends
His comfort from on high.
And then at times before we cry,
His answer has begun,
Our every need is known to Him
Long ere our path is run.
O, what a God we have to trust,
And God of love and grace,
Who never will refuse the cry
Of those who seek His face.
Then may we oft while here we wait
To meet Him in the air,
In earnest supplication spend,
That hour, the hour of prayer.

Practical Converstions With Our Young People: A Precious Service in Matthew 27-28

“The part that women take in all this history is very instructive, especially to them. The activity of public service, that which may he called “work,” belongs naturally to men (all that appertains to what is generally termed ministry), although women share a very precious activity in private. But there is another side of Christian life which is particularly theirs; and that is personal and loving devotedness to Christ. It is a woman who anointed the Lord while the disciples murmured; women who were at the cross when all except John had forsaken Him; women who came to the sepulcher, and who were sent to announce the truth to the apostles who had gone, after all, to their own home; women who ministered to the Lord’s need. And, indeed, this goes farther.
Devotedness in service is perhaps the part of man, but the instinct of affection, that which enters more intimately into Christ’s position, and is thus more immediately in connection with His sentiments, in closer communion with the sufferings of His heart – this is the part of woman; assuredly a happy part. The activity of service for Christ puts man a little out of this position, at least if the Christian is not watchful. Everything has, however, its place. I speak of that which is characteristic; for there are women who have served much, and men who have felt much. Note also here, what I believe I have remarked, that this clinging of heart to Jesus is the position where the communications of true knowledge are received. The first full gospel is announced to the poor woman that was a sinner who washed His feet; the embalming for His death to Mary; our highest position to Mary Magdalene; the communion Peter desired, to John who was in His bosom. And here the women have a large share.”

Questions for November

Always give Scripture for answers, as well as your own expressions.
Give scriptures for each of the following definitions of the death of Christ:
1st As a ransom.
2nd As a propitiation.
3rd As a reconciliation.
4th As a substitution.
5th Give five scriptures that Christ died for sinners.

Answers to Questions for August

ANSWERS TO QUESTION 1.
What scriptures prove the humanity of the Lord Jesus Christ?
(a) “And the Word was made flesh, and dwelt among us, (and we beheld His glory as of the only begotten of the Father,) full of grace and truth” (John 1:14).
(b) “Jesus therefore, being wearied with His journey” (John 4:6).
(c) “I thirst” (John 19:28).
(d) “And in those days He did eat nothing: and when they were ended, He afterward hungered” (Luke 4:2).
(e) “But as they sailed He fell asleep” (Luke 8:23).
(f) “And being found in fashion as a man, He humbled himself, and became obedient unto death” (Phil. 2:8).
ANSWERS TO QUESTION 2.
What scriptures prove the deity of the Lord Jesus Christ?
(a) “Who being in the form of God, thought it not robbery to be equal with God” (Phil. 2:6).
(b) “In the beginning was the Word, and the Word was with God, and the Word was God.” “All things were made by Him; and without Him was not anything made that was made” (John 1:1,3). (See Gen. 1: “In the beginning God created.”)
(c) “But unto the Son He saith, Thy throne, O God, is forever and ever” (Heb. 1:8). (See Col. 1:15-17).
“Jesus said unto them, Verily, verily, I say unto you, Before Abraham was, I am” (John 8:58). Compare Exodus 3:13-14: “What is His name?” “And God said... I am that I am.... thus shalt thou say.... I am hath sent me unto you.”
“I have power to lay it down, and I have power to take it again” (John 10:18). No one but God could have this power.
ANSWERS TO QUESTION 3.
Give a few of the divine names that are given to Him, and their meanings.
(a) Jesus-Savior. “And she shall bring forth a Son, and thou shalt call His name Jesus: for He shall save His people from their sins” (Matt. 1:21).
Lord – If we call Him Lord, we own Him as Master, and ought to obey Him, for He says: “And why call ye Me, Lord, Lord, and do not the things which I say?” (Luke 6:46).
(b) Lord – The one who has authority (Rev. 4:11).
Christ – the anointed one, the Messiah (John 1:41).
Emmanuel – God with us (Matt. 1:23).
Alpha and Omega – the Beginning and the ending (Rev. 1:8).
I am – ever existent (John 8:58). This name also seems to suggest: All-powerful, Infinity, Pre-eminence.
The Morning Star – The hope of the church (Rev. 22:16-17). The morning star is seen before sunrise, so the church will see the Lord before He comes in blessing to Israel (1 Thess. 4:16-17).
Sun of Righteousness – the hope of Israel. (Mal. 4:2).
The Faithful Witness. Rev. 1:5.
The above are just a very few.
ANSWERS TO QUESTION 4.
Give Scripture proof that the Lord Jesus is a man in heaven now.
(a) “Who is gone into heaven, and is on the right hand of God; angels, and authorities and powers being made subject unto Him” (1 Peter 3:22).
(b) “And it came to pass, while He blessed them, He was parted from them, and carried up into heaven” (Luke 24:51).
(c) “Behold My hands and My feet, that it is I Myself: handle Me and see; for a spirit hath not flesh and bones as ye see Me have” (Luke 24:39).
(d) “Ye men of Galilee, why stand ye gazing up into heaven? this same Jesus which is taken up from you into heaven, shall so come in like manner as ye have seen Him go into heaven” (Acts 1:11).
(e) “But he, being full of the Holy Ghost, looked up steadfastly into heaven, and saw the glory of God, and Jesus standing on the right hand of God, and said, Behold I see the heavens opened, and the Son of Man standing on the right hand of God” (Acts 7:55-56).
ANSWERS TO QUESTION 5.
Give five scriptures that Christ is the only way..
John 6:51-54; John 14:6; Acts 4:12; Acts 11:43; Romans 5:15,18-19; Hebrews 9:12.

Truths for Young Christians: Christian Devotedness, Part 2

But thus near to God and in communion with Him, thus not only united, but consciously united, to Christ by the Holy Spirit, divine love flows into and through our hearts. We become animated by it through our enjoyment of it. It is really “God dwelling in us,” as John expresses it; “His love shed abroad in our hearts,” as Paul does. It flows thus forth as it did in Christ. Its objects and motives are in Him, save that He Himself comes in as revealing it. It is the love of God in Christ Jesus our Lord; not the less God, but God revealed in Christ, for there we have learned love. Thus, in all true devotedness, Christ is the first and governing object; next, “His own which are in the world”; and then our fellow-men. First their souls, then their bodies, and every want they are in. His life of good to man governs ours, but His death governs the heart. “Hereby know we love because He laid down His life for us.” “The love of Christ constraineth us, because we thus judge, that if one died for all, then were all dead: and that He died for all, that they which live should not henceforth live unto themselves, but unto Him which died for them, and rose again.”
We must note, too, that as redemption and divine righteousness are that through which grace reigns and love is known, all idea of merit and self-righteousness is utterly excluded, so it is a new life in us which both enjoys God and to which His love is precious; which alone is capable of delighting, as a like nature, in the blessedness that is in Him, and in which His divine love operates towards others. It is not the benevolence of nature, but the activity of divine love in the new man. Its genuineness is thus tested, because Christ has necessarily the first place with this nature, and its working is in that estimate of right and wrong which the new man alone has, and of which Christ is the measure and motive. “Not as we hoped,” says Paul (it was more than he hoped), speaking of active charity; “but first gave their ownselves to the Lord, and to us by the will of God.”
But it is more than a new nature. Our bodies are the temples of the Holy Spirit; and God’s love is shed abroad in the heart by the Holy Spirit which is given to us. And as it springs up like a well in tis unto eternal life, so also living waters flow out from us by the Holy Spirit which we have received. All true devotedness, then, is the action of divine love in the redeemed, through the Holy Spirit given to them.
There may be a zeal which compasses sea and land, but it is in the interest of a prejudice, or the work of Satan. There may be natural benevolence clothed with a fairer name, and irritated if it be not accepted for its own sake. There may he the sense of obligation and legal activity, which, through grace, may lead farther, though it be the pressure of conscience, not the activity of love. The activity of love does not destroy the sense of obligation in the saint, but alters the whole character of his work. “Where the Spirit of the Lord is, there is liberty.” In God love is active, but sovereign; in the saint it is active, but a duty, because of ‘grace. It must be free to have the divine character – to be love. Yet we owe it all and more than all, to Him that loved us. The Spirit of God which dwells in us is a Spirit of adoption, and so of liberty with God, but it fixes the heart on God’s love in a constraining way. Every right feeling in a creature must have an object, and, to be right, that object must be God, and God revealed in Christ as the Father; for in that way God possesses our souls.
Hence Paul, speaking of himself, says, “I am crucified with Christ, nevertheless I live; yet not I, but Christ liveth in me; and the life which I now live in the flesh, I live by the faith of the Son of God, who loved me and gave Himself for me.” His life was a divine life. Christ lived in him, but it was a life of faith, a life living wholly by an object, and that object Christ; and known as the Son of God loving and giving Himself for him. Here we get the practical character and motive of Christian devotedness – living to Christ. We live on account of Christ: He is the object and reason of our life (all outside is the sphere of death); but this is the constraining power of the sense of His giving Himself for us. So, in a passage already referred to, “The love of Christ constraineth us, because we thus judge, if one died for all, then were all dead: and He died for all, that they which live should not live to themselves, but to Him who died for them and rose again.” They live to and for that, and nothing else. It may be a motive for various duties, but it is the motive and end of life. “We are not our own, but bought with a price,” and have to “glorify God in our bodies.”
What is supposed here is not a law contending or arresting a will seeking its own pleasure, but the blessed and thankful sense of our owning ourselves to the love of the blessed Son of God, and a heart entering into that love and its object by a life which flows from Christ and the power of the Holy Spirit. Hence it is a law of liberty. Hence, too, it can only have objects of service which that life can have, and the Holy Spirit can fix the heart on; and that service will be the free service of delight. Flesh may seek to hinder, but its objects cannot be those the new man and the Holy Spirit seek. The heart ranges in the sphere in which Christ does. It loves the brethren, for Christ does; and all the saints, for He does. It seeks the all for whom Christ died, vet knowing that only grace can bring any of them; and endures “all things for the elect’s sake, that they may obtain the salvation which is in Christ Jesus with eternal glory.” It seeks “to present every man perfect in Christ Jesus”; to see the saints grow up to Him who is the Head in all things, and walk worthy of the Lord. It seeks to see the church presented as a chaste virgin unto Christ. It continues in love, though the more abundantly it loves the less it be loved. It is ready to endure hardness as a good soldier of Jesus Christ.
The governing motive characterizes all our walk: all is judged by it. A man of pleasure flings away money; so does an ambitious man. They judge of the value of things by pleasure and power. The covetous man thinks their path folly, judges of everything by its tendency to enrich. The Christian judges of everything by Christ. If it hinders His glory in oneself or another, it is cast away. It is judged of not as sacrifice, but cast away as a hindrance. All is dross and dung for the excellency of the knowledge of Christ Jesus our Lord. To cast away dross is no great sacrifice. How blessedly self is gone here! “Gain to me” has disappeared. What a deliverance that is! Unspeakably precious for ourselves and morally elevating! Christ gave Himself. We have the privilege of forgetting self and living to Christ. It will be rewarded, our service in grace; but love has its own joys in serving in love. Self likes to be served. Love delights to serve. So we see, in Christ, on earth, now; when we are in glory, He girds Himself and serves us. And shall not we, if we have the privilege, imitate, serve, give ourselves to Him, who so loves us? Living to God inwardly is the only possible means of living to Him outwardly. All outward activity not moved and governed by this is fleshly and even a danger to the soul – tends to make us do without Christ and brings in self. It is not devotedness, for devotedness is devotedness to Christ, and this must be in looking to being with Him. I dread great activity without great communion; but I believe that, when the heart is with Christ, it will live to Him.
The form of devotedness, of external activity, will be governed by God’s will and the competency to serve; ‘for devotedness is a humble holy thing, doing its Master’s will; but the spirit of undivided service to Christ is the true part of every Christian. We want wisdom: God gives it liberally. Christ is our true wisdom. We want power: we learn it in dependence through Him who strengthens us. Devotedness is a dependent, as it is a humble, spirit. So it was in Christ. It waits on its Lord. It has courage and confidence in the path of God’s will, because it leans on divine strength in Christ. He can do all things. Hence it is patient and does what it has to do according to His will and word: for then He can work; and He does all that is done which is good.
(Continued from page 267).
(To be continued).

Scripture Study: Matthew 26:1-30

Matthew 26
Matthew 26:1. As the great prophet, foretold by Moses (Deut. 18:15-19), the Lord has now finished His discourses. We enter now on the last circumstances of His life, and that led on to His death. He knows what is before Him, and said to His disciples: Verse 2. “Ye know that after two days is the Passover, and the Son of Man is betrayed to be crucified.” No excitement appears, but the calm, quiet spirit of one doing the Father’s will.
Matthew 26:3-5. It was decided by Israel’s leaders to put Him to death, and the high priest’s palace is their meeting place. They mean to murder the Son of God. It is their intention to do it with subtlety, and not on the feast day, for they feared the people. But that was the very day decreed of God. The Lord has announced it to His disciples. Man’s wickedness is seen, but God overrules; it was the will of God that Jesus should suffer at that very time.
Matthew 26:6-13. What a comfort to the Lord’s spirit the act of this woman must have been. It is at Bethany – the house of dates – in the house of Simon the leper. There she pours the precious ointment on His head; there she anoints His body for its burial. Did she not discern the storm clouds gathering over His head? Did she not see the looks, and hear the words of spite and hatred uttered against Him? And did she not hear His own words, “The Son of Man is delivered to be crucified”? One soul at least is feeling sympathy with Him.
But the disciples, seeing it, had indignation; first Judas Iscariot (John 12:4), then the disciples join in saying, “To what purpose is this waste? for this ointment might have been sold for much, and given to the poor.” The world and worldly-minded Christians count waste what is expended on Jesus only; and good men fall under the influence of one altogether bad. Man’s good is man’s highest object. Christ is the Father’s object of delight; the world has self in some form as its object. Judas Iscariot posed as a philanthropist, but he was actually a thief, a covetous man, yet the disciples fell in with his thoughts; they were earthly-minded.
King David appreciated the devoted love of those three men (2 Sam. 23:13-17) who risked their lives to fetch him the drink of water he longed for, but he would not drink it; it was so precious in his sight, he poured it out before the Lord. How much more precious is this woman’s act to the Lord! He is her anointed King, although rejected by men. He defends her and rebukes them, “Why trouble ye the woman? for she hath wrought a good work upon Me. For ye have the poor always with you, but Me ye have not always. For in that she hath poured this ointment on My body, she did it for My burial. Verily, I say unto you, Wheresoever this gospel shall be preached in the whole world, there shall also this, that this woman hath done, be told for a memorial of her.” What a cheer to her this was. The Lord knew her love for Him, and appreciated it; gave her His “well done,” and rebuked the faultfinders. He declared it a good work, let others say what they please. The sense of His approval, “I know thy works” (Rev. 3:8), may be ours, should be ours, also.
She was thinking of His sufferings and of His death. She was in communion with Him, and this was a special time that they did not notice. It is good to help the poor, but to confess the Lord, to be with Him in the time of His rejection, is a privilege we will not have in heaven. She was ahead of all the rest. Joseph of Arimathea, Cornelius, and the other women, all showed their love and did their part, but this woman anticipated His death, and her Lord understood her. What matters then the frown of the world or of men, since He is pleased and refreshed?
And what honorable mention is given this deed? “Verily, I say unto you, Wheresoever this gospel shall be preached in the whole world, there shall also this, that this woman hath done, be told for a memorial of her.” How well worth repeating this story is! It is not doing good to man, nor is it quantity of work. It is what is done for and in communion with Christ that gains His “well done.” It needs no gift; it is not too high for a child to reach. We can refresh His heart, but it commences with sitting at His feet; it is the outcome of being in His company and of hearing His words. May our souls know more of this.
Matthew 26:14-16. Judas Iscariot’s covetousness now leads him to offer his services in the work of Satan, and these so-called leaders of Israel accept his services, and the Lord of glory is sold for thirty pieces of silver, the price of a common slave. From that time Judas watches his opportunity to deliver Him up. The love of money is a root from which every evil springs. No teaching of Jesus entered his heart to hinder his downward course. It had been good for that man if he had not been born. How great his privileges! How terrible his judgment!
Matthew 26:17-25. The first day of the feast of unleavened bread, the disciples ask Him, “Where wilt Thou that we prepare for Thee to eat the passover? And He said, Go into the city to such a man and say unto him, The Master saith My time is at hand; I will keep the passover at thy house with My disciples.” Here was another He counted on to own His claim. In Mark and Luke it is a man bearing a pitcher of water (the figure of the word for his path) that guides them to the place. There they made ready the passover. “When the evening was come, He sat down with the twelve, and as they did eat, He said, Verily I say unto you, that one of you shall betray Me. And they were exceedingly sorrowful, and began everyone of them to say unto Him, Lord, is it I? And He answered and said, He that dippeth his hand with Me in the dish, the same shall betray Me. The Son of Man goeth as it is written of Him, but woe unto that man by whom the Son of Man is betrayed! It had been good for that man if he had not been born. Then Judas, which betrayed Him, answered and said, Master is it I? He said unto him, Thou hast said.”
What an appeal and warning to Judas this is! The Lord knows his hypocrisy, but he is given up to Satan (John 13:2,27). And after the sop Satan enters into him to carry out his foul purpose.
Matthew 26:26. “And as they were eating, Jesus took bread, and blessed it, and brake it, and gave it to the disciples, and said, Take, eat, this is My body. And He took the cup, and gave thanks, and gave it to them, saying, Drink ye all of it; for this is My blood of the new testament (covenant), which was shed for many for the remission of sins.”
This is the memorial supper of the true passover, for Christ, our passover, is sacrificed for us (1 Cor. 5:7). It is a Savior who died for us that is to be remembered. It is not the remembrance of Israel brought out of Egyptian bondage, but of the Lord, the true Lamb of God, whose blood was shed for many for the remission of sins – the blood on which the new covenant with Israel will be founded. “Shed for many” tells of others besides Israel, and takes us in, though by nature Gentile dogs. It was not like the blood of Exodus 24, to ratify their promise of obedience. It was because they were sinners, and was for the remission of sins. In Luke it is added, “Was shed for you” (Luke 22:20).
In the supper, we remember the Lord in His death; the bread and wine, His body and His blood are separate, bringing Him before us in death. He is now glorified at the Father’s right hand. They are to expect to be with Him there, for He says, “I will not drink henceforth of this fruit of the vine, until that day when I drink it new (in a new way) with you in My Father’s Kingdom.” And having sung an hymn, they went out, into the Mount of Olives. Does this not bear witness that the joys of redemption rise over all the sorrows and trials of this present life?
(To be Continued).

Worship

We are needy enough, perhaps, to know what prayer is; our needs have been so_ richly met that we can gladly join in thanksgiving for benefits received; but, though worship does not exclude the latter, vet in its fullest, highest sense, it goes far beyond it.
Christian worship is the adoring homage of the soul in presence of God fully revealed. Our frames, feelings, trials, experiences, or service have no place here. We may have abundant cause to pray about all these; but when we worship, ourselves and our circumstances are forgotten as we admire and adore the deep and manifold perfections of God, revealed as the Father in Christ.
Have we not often to mourn over the poverty of worship in our own souls, and in the assemblies? Is there not a cause? I believe the secret to be that in spite of all our high talk and much intelligence, we have but little real appreciation of Christ.
There is wonderful force, to my soul, in the close connection between “worshiping God in the spirit” and “rejoicing in Christ Jesus” (Phil. 3:3). If we ourselves were actually in the enjoyment of Christ; if our hearts were finding continual delight in Himself and what He has done; not merely resting on Him and thus getting relief about the past and deliverance from all fear of the future (alas! many never seem to get beyond this), but rejoicing in Him in all His excellence and perfections, our hearts would be like censers continually giving out His fragrance in true spiritual worship.
I put the question solemnly to my own heart, and I put it to yours – Are you enjoying Christ?
I have not a doubt, thank God, of your salvation or of my own, and this very assurance enables us to welcome everything that produces exercise in our hearts before God. Remember, that intelligence is not enjoyment. Has it not been manifest again and again that where there has been the greatest amount of intellectual acquaintance with the Word, there has been little enjoyment of God’s favor in the soul?
Are you rejoicing in Christ as the Sin Offering? Is your heart occupied with thoughts of that blessed spotless Victim who has been in the awful place of distance, and death, and judgment for you and your sins? Do you meditate with adoring wonder on that cry of unparalleled anguish – “My God, My God, why hast Thou forsaken Me.” All the horrors of your place of distance from God were felt by His holy soul as none beside could ever feel them, during the three hours of His substitutionary sufferings on the cross. When you were without strength and ungodly He died for you. Yes, sinner as you were, He died for you! He gave Himself for your sins – ah! those dark, crimson, soul-defiling, and God-dishonoring sins and died for them according to the Scriptures, bearing them in His own body on the tree, and suffering for them every one. Jehovah’s sword of judgment awoke from its sheath of mercy when the Man who was His Fellow took your place upon the cross. Judgment without mercy, wrath without relief were His portion, and He has exhausted the flame; it has burnt itself out upon Him: and you are forgiven, cleansed, justified, redeemed, purged in your conscience, and made nigh by His blood. Are you enjoying it?
(To, be continued).

They That Are Christ's at His Coming: Part 1

1 Corinthians 15:23
What a scene of indescribable glory that will be, when the Lord Himself shall descend to gather His redeemed to the home He has prepared for them! What a moment of unsullied delight, when at the shout, the voice of the archangel, and the trump of God, in a moment, in the twinkling of an eye, His sleeping saints, raised, and living saints, changed, “shall be caught up to meet the Lord in the air, and so shall they ever be with the Lord”!
All the hosts of the redeemed shall be marshaled there, in bodies of glory, instinct with divine life; – the saints of old, who on the faith of a promise, were worshipers, pilgrims, soldiers; those to whose faith dens and caves bore witness; “of whom the world was not worthy”; the elders, and just men, who “died in faith, not having received the promise,” shall be there; “Abraham, Isaac and Jacob”; “Noah, Daniel and Job”; “Moses and Elias,” shall be there; Abel, and the long line of martyrs; Aaron, and the Lord’s priests; Samuel, and the Lord’s prophets; David, and the men of faith who sat on his throne; all God’s renowned ones, the perfected just, shall stand in that scene for which they in faith waited. “The church of the first-born,” too, as the bride prepared for her Lord, shall take her place there; all down to the last re-born soul, who shall form the completion of the mystery. “This is a great mystery; but I speak concerning Christ and the church” (Eph. 5:32). She, too, will recount her worthies in that morning; the many who have stood forth in other days, and who stand forth in our own day, as the witnesses of God’s truth, and the heralds of God’s salvation, all shall ascend together and swell the countless multitude of Christ’s own shall take their place, too, in their respective glories – “every man in his own order” – star differing from star in glory, and each reflecting the image of Jesus.
There will be seats, too, in the kingdom; thrones for rulership over the tribes of Israel; abodes in the Father’s house; thrones around the throne of God; all shall be occupied by the redeemed, each invested with the insignia sovereign love has assigned him. All will “know even as they are known” – each known to each – all known to all. What a season of unutterable joy! of holy intercourse! of uninterrupted communion! But the rapturous thought of each one of this innumerable company will be, they are Christ’s, “I am My beloved’s, and His desire is towards me.” To be Christ’s own, will be a source of deep, unmixed pleasure then (should it not be now?). The absorbing object of their heaven-inspired vision will be Christ; to be forever with Him; to behold Him; to cast their crowns at His feet, paying the heart’s deep homage to Him in one united utterance of “Thou art worthy, for Thou vast slain, and hast redeemed us to God by Thy blood.”
The power of Christ’s resurrection will be applied to the bodies of His saints; they will be raised, because He has been raised, by virtue of having His life, and being indwelt by His Spirit they will be presented in the perfection of that life, in its full triumph over death, and him who had the power of death; they are raised – not for judgment, that to them is passed, Christ bore it for them, but because they are Christ’s – Christ’s resurrection was the first-fruits, and the pledge of that abundant ingathering. He was the first sheaf presented to the Lord, the sample and earnest of the harvest that shall then be gathered into the garner of God; they will be raised up, and presented in the glory with Him. He is the expression of the glory, and they stand in Him. The reunited dust shall be reanimated and vivified with divine life; the weakness shall be transformed into power, corruption into in-corruption, dishonor into glory, the natural body into a spiritual body; it will bear the impress of the heavenly, even as it has borne the image of the earthy. Where is the sting of death? Gone! Where the grave’s victory? Gone! Victory, full, complete, eternal, is theirs – Satan bruised under their feet forever.
(To be continued).

Correspondence: Life at the Judgement; Great White Throne

Question 110: Is it our life after our conversion that is manifested at the judgment seat of Christ? or, Is it all our life from our birth? If the latter, how are we to understand, “Your sins and iniquities will I remember no more”? M. H.
Answer: The judgment seat of Christ (2 Cor. 5:10) takes place after the sleeping and changed saints have been caught up. They are glorified, therefore no thought of sins imputed can be raised. They are with and like the Lord. Their whole life is manifested there; all they did in the body, whether good or bad. The Lord will approve of what He can, and disapprove of what He sees was wrong, even to the hidden motives of the heart (1 Cor. 4:5). This will show us, as we never knew before, what grace has done for us, and how the Lord has borne with us all through our lives. Knowing we are to be manifested, should exercise us now to judge in the Lord’s presence our present ways (Psa. 138: 23-24).
Hebrews 10:17 is a quotation from Jeremiah 31, to show that the blessings of the new covenant already belong to the believer through the finished work of Christ. “By one offering He hath perfected forever them that are sanctified” (Heb. 10:14).
Question 111: Are only the unsaved to stand at the Great White Throne? What does it mean by “books” and “another book”? T. N. L.
Answer: If you read the chapter you will see that the first resurrection, that is, of the blessed and holy, took place a thousand years before this scene. That was the resurrection of life This is the resurrection of the dead, the resurrection of judgment (John 5:29). Those seen standing at the great white throne are judged according to their works; they have no substitute, no Savior. There is no hope for them now. The book of life can only witness that their names are not written in it. One book contains each person’s account of what they have done. The other is the Word of God. (See John 12:48). They are therefore judged, and they get justice, fearful doom, fully merited; they are cast into the lake of fire. The believers are saved by grace.

Inspiration of the Scriptures: The Bible - its Blessedness, Part 2

The Bible – Its Blessedness
The Scriptures give us divine intelligence about everything necessary to completely furnish the believer unto every good work. Do I inquire what God’s present ways on earth for blessing are? I learn from His word that He is calling out and forming a bride for His own Son, whose heart is set upon her as His own body, and who “nourishes and cherishes it, even as the Lord the church”; and that this will go on till we hear the “shout” and are caught up to meet the Lord in the air, and so are “forever with the Lord.” Do we inquire as to the Jews? We still find the “outcasts of Israel – the ten tribes are cast out, no man knows where; and the two tribes are still “dispersed” among the nations, fulfilling the Scriptures of the prophets as to their state; and one now and then believing the gospel, and forming part of God’s assembly. If a question be asked about the European nations, which once formed part of the Roman empire? According to the Word of God by Daniel, we learn that all is going on to the development of the ten kingdoms before divine judgment falls upon them. Does any one inquire whether Scripture tells us anything about Protestantism? Most surely; it is referred to in the epistle of the church at Sardis, and is described as having a name to live and being dead. Protesting against error may be with those who are quite devoid of faith in God’s revealed truth concerning His Son, and therefore such have not life – “He that hath not the Son of God hath not life” (1 John 5:12). And it would be impossible that such a huge system as Popery, established on the earth for so main, centuries, should be omitted in God’s Word written for our instruction; we find, therefore, much in Scripture concerning it, and what is so striking, it stands in the end of this age in rival antagonism with infidelity; and it needs but little spiritual discernment to see how rapidly this ill-feeling and fear of each other is growing. Nevertheless, as before observed, the result must be that the infidel power will be too strong for the Papal power, and will spoil and destroy this unchaste woman. (Rev. 17:16-17; 18:8). The great whore may include all in Christendom who are untrue in affection to the Lord Himself. Thank God not one child of His can be lost, and He knoweth them that trust in Him.
With regard to the church, or assembly of God in the true sense (which is doubtless the most blessed testimony, that ever emanated from God, and the workmanship of the Holy Spirit), it must abide forever, because each true believer is by one Spirit united to Christ in ascension as a member of His body, and is always an object of His care and blessing. On the other hand, the assembly looked at as God’s corporate witness on the earth, it has terribly fallen from the Pentecostal character of blessing and power in which He set it. Too often it is a witness of division, carnality, and strife, instead of keeping diligently the Spirit’s unity in the uniting bond of peace, through holding the Head, subjection to the word, and owning the presence and power of the one Spirit for all true blessing and unity.
And so with our open Bible we can look around on every hand, and see things just as they are set forth there for our guidance and instruction. One thing is certain, that while God looks for individual faithfulness and separation to Himself, and will always honor few or many who are really gathered to the name of our Lord Jesus Christ, yet He gives no hope of the reconstruction and return of unity in general of His assembly on earth; but enjoins us individually to purge ourselves from vessels to dishonor, and be with those (if we can find them), who call on the Lord out of a pure heart (2 Tim. 2:19-22).
And further, not only are we told that “God... hath in due times manifested His word through. preaching,” but it is also written, “Thou hast magnified Thy word, above all Thy name” (Titus 1:3; Psa. 138:2). So that, because Scripture is God’s Word, it assures us, without any question, of what is His mind and will concerning us. By it every one who has truly looked as a lost sinner to the Lord Jesus Christ as the object of faith and trust, is assured that his sins are forgiven, and blotted out forever. “To Him [Christ] give all the prophets witness, that through His name, whosoever believeth in Him shall receive remission of sins:” and “their sins, and iniquities will I remember no more” (Acts 10:43; Heb. 10:17)
Such know, also, on the authority of the Scriptures, that they have eternal life given to them, and that they possess it. We never find an apostle saying, “I feel that I have this and that, and because I feel it I know I have it.” Never; because feelings quickly change, and the word never directs us to look at our feelings for evidence, but at what God says. Now what does He say about the present gift of eternal life? He says, in John’s first epistle, “These things have I written unto you that believe on the name of the Son of God, that ye may know that ye have eternal life.” He does not say here that you may feel it, but know it; know it on the authority of God’s Word, which we are told was written that we might know, with divine certainty, that we have eternal life. “God hath given to us eternal life, and this life is in His Son. He that hath the Son hath life, and he that hath not the Son of God hath not life.” Nothing, can be more simple, more decided, more comforting, to such as trust God according to His own word. Nothing less is faith. O, the untold blessedness of knowing, and being able truthfully to say, that God hath given to me eternal life, communicated it to me, and this life is in His Son – Christ my life, so that Christ liveth in me; and so real that it comes out in affections, thoughts, and feelings according to Him. “We know that we have passed from death unto life, because we love the brethren. He that loveth not his brother abideth in death” (1 John 3:14). How very comforting, and yet how solemn!
And more than this; for nothing short of having us in the new and eternal relationship of children could suit the Father’s heart; and this, too, the Scriptures clearly teach, not only for present comfort, but to bring out the duties and affections that necessarily flow from knowing such an endearing relationship. The Scripture not only says, ‘Ye are all the children of God by faith in Christ Jesus, but goes on to tell us that the Holy Spirit dwells in us because We are sons. “Because ye are sons, God hath sent forth the Spirit of His Son into your hearts, crying, Abba. Father”; and it is also clear that those who have the Spirit in them know that thee are God’s children for “the Spirit itself beareth witness with our spirit the new life or nature], that we are the children of God” (Gal. 4:6; Rom. 8:16). Those, then, who know, or are personally assured, on the authority of God’s Word, that they are children of God, have received the Spirit, and can approach God as their Father. We have received “the Spirit of adoption, whereby we cry, Abba, Father Duties and affections, as we have said, flow from known relationships; for how can any one manifest the love, interest, subjection, and devotion of a child, or walk as a child, unless he knows he is a child? O the unfathomable blessedness and treasure of the Scriptures!
And yet more still. We learn also from the page of inspiration, that the believer is now set by God in a totally new standing, “in heavenly places in Christ Jesus’’: so that he is recognized and addressed in the epistles as not in his sins, not in the flesh, not under law, not of the world, not now in Adam, but “in the Spirit,’’ and “in Christ Jesus.” Hence there is a new creation in Christ Jesus, and such are always before God in all the nearness and acceptance of “the Beloved. What a position of favor and blessing to be always thus before God in all the nearness and relationship of children! Known, too, on the unfailing authority of His Word, with joy and peace in believing, and for such present blessing that we may rejoice in the Lord always, obey His Word, and wait for His return from heaven. The question now for every true believer is not, Am I in Christ? or am I a child of God? for God has told such in His Word, as we have seen, over and over again, that these questions are settled forever; but the important question for us is, “Am I living in the enjoyment of communion with the Father, and with His Son Jesus Christ?” How can this be, if His Word be not loved, received, and meditated on by us, as the treasury of His thoughts, affections, purposes, and ways? Jesus said, “If a man love Me, he will keep My words;... he that loveth Me not keepeth not My sayings: and the word which ye hear is not Mine but the Father’s which sent Me” (John 14:21-24).
(Continued from page 284).
Concluded.

The Arithmetic of Heaven

One day when Daniel Webster was going away from church, he met a Unitarian, who accosted him thus: “So you have been to church, where they teach that three times one are one!” Mr. Webster replied with his solemn voice, now more intensely solemn than usual, “My friend, you and I do not understand the arithmetic of heaven.”
If any man less than Mr. Webster had made this reply, it might be considered an evasion of the difficulty suggested. Mr. Webster had been attending a church where the doctrine of the Trinity is taught – three Persons in one Godhead – Father, Son, and Holy Spirit; Three in One. No human intellect can comprehend the mode of such existence; and there are those who reject this truth because it does not seem to them reasonable. But those who are wise will take God at His Word, believing, with Mr. Webster, that “the arithmetic of heaven” may not be understood on earth. In Deuteronomy 6:4, we get these words, “The Lord Our God is one Lord.” In the New Testament we get three distinct personalities, Father. Son, and Holy Spirit, repeatedly represented, and the working of these three is in perfect harmony. We own a triune God.

Practical Conversations With Our Young People: God's Will and Our Work

In connection with the Christian’s pathway through the world, it is of all importance that he should rest on the power, the wisdom, and the love of God, the “three pillows,” as a sick saint once called them, “for a weary head to lie upon,” and seek to “fill the little space” which He gives us to do His will in. We may not be able to trace His goings, for “His way is in the sea, and His path in the great waters, and His footsteps are not known.” But He knows the way He takes, whether with His people or in connection with His government of the world. And we are wise if we leave Him to carry out His own will which must be good and right for He is good and right.
This thought has been beautifully expressed by a well-known writer in the following words: “The persecuting emperor little thought what He was giving to us when he banished the Apostle [John, to Patmos]; no more than Augustus, in his political plans as to the census of the empire, knew he was sending a poor carpenter to Bethlehem, with his espoused wife, that Christ might be born there; or the Jews and Pilate’s soldiers, that they were sending the thief to heaven when they broke his legs in heartless respect for their own superstitions or ordinances. God’s ways are behind the scenes; but He moves all the scenes which He is behind. We have to learn this, and let Him work, and not think much of man’s busy movements; they will accomplish God’s. The rest of them all perish and disappear. We have only peacefully to do His will.’
Every Christian has his own appointed service. One in this way and another in that; but each in some way. Many scriptures may occur to the mind of the reader, but a few are appended. They are “seed thoughts” for him as he meditates on these things. “I know that Thou canst do everything, and that no thought of Thine can be hindered” (Job 42:2, margin).
“The floods have lifted up, O Lord, the floods have lifted up their voice; the floods lift up their waves. The Lord on high is mightier than the noise of many waters, yea, than the mighty waves of the sea” (Psa. 93:3-4).
“There are many devices in a man’s heart; nevertheless the counsel of the Lord, that shall stand” (Prov. 19:21).
“Known unto God are all His works from the beginning of the world” (Acts 15:18).
“Who is among you that feareth the Lord, that obeyeth the voice of His servant, that walketh in darkness, and hath no light? let him trust in the name of the Lord, and stay upon his God” (Isa. 50:10).

Questions for December

Always give Scripture for answers, as well as your own expressions.
1. Give scriptures, proving that Christ’s body was actually raised from the dead.
2. Give scriptures in regard to those who saw Him after the resurrection, and recognized Him as having the same body He had before, even to the wound-prints.
3. Give five Scripture proofs that the Lord is risen:
(a) The empty tomb.
(b) The Lord’s day.
(c) The church.
(d) The New Testament.
(e) Five hundred and fourteen witnesses are recorded as having seen the Lord.
4. When and where must we give an account of our lives?
5. Give five scriptures which tell us that the Lord will come again for His own.

Truths for Young Christians: Christian Devotedness, Part 3

And in point of fact, if the flesh be practically allowed, it is a continual hindrance, and reproach and opposition are then a burden, not a glory. We have with Paul to “bear about in our body the dying of the Lord Jesus, that the life also of Jesus may be manifested in our mortal bodies,” and so to have the sentence of death made good in ourselves. Here the Lord’s help, through trials and difficulties, comes in. But we are “more than conquerors through Him that loved us.” Nothing separates us from that love. But if we come to the management of our own heart, we shall find that this “always bearing about in our body the dying of the Lord Jesus” is the great difficulty and tests the inward state of the soul. Yet there is no liberty of service nor power but in the measure of it; only, remark, we have this power in the sense of grace. It is the power of the sense we have of His dying and giving Himself for us, which by grace makes us hold ourselves as dead to all but Him. Outwardly it may be comparatively easy, and so is outward labor when self and Satan’s power are not felt in opposition. But to have Christ’s dying always made good against self, detected by the cross, supposes Christ to be all in the affections. The true power and quality of work is measured by it the operation of God’s Spirit by us. This is the one way of devotedness in God’s sight, and God’s power and the having the mind of Christ in the service we do render. This only is life.
And the rest of our life, not to speak of loss or judgment, perishes when our breath goes forth, It belongs to the first Adam and to the scene he moves in, not to the second. It is only the life which we live by Christ which remains as life.
Its motives and character are twofold: the cross and Christ in glory. The love of Christ constrains us in the cross to give ourselves wholly up to Him who has so loved us, given Himself wholly up for us. The winning Christ and being like Him in glory gives energy, and the spring and power of hope to our path. But how constraining and mighty is the first motive, if we have really felt it! Yet how lowly! It makes us of little esteem to ourselves in the presence of such love. We see we are not our own, but bought with a price. Nor is that all. The sense of the love of Christ takes possession of the heart and constrains us. We desire to live, too, to Him who gave Himself for us. The perfection of the offering and the absoluteness and perfectness with which it was offered, alike His love to us in it, has power over our souls. “Through the eternal Spirit He offered Himself without spot to God.” The sense that we are not our own deepens the claim in our hearts, yet takes away all merit in the devotedness. So wise and sanctifying are God’s ways! How does the thought too, of winning Him make all around us but dross and dung for the excellency of the knowledge of Him! What is all compared with pleasing Him, possessing Him, being with Him and like Him forever! It puts the value of Christ, as the motive, on everything we do. It leads to true largeness of heart, for all dear to Him becomes precious to us, yet keeps from all looseness of nature, feelings, for we are shut up to Christ. What is not His glory is impossible. It puts sin practically out of the heart by the power of divine affections, by having the heart filled with Him. Practically the new nature only lives with Christ for its object.
It applies, too, remark, to everything, because we have to please Christ in everything. Dress, worldly manners, worldliness in every shape disappears. They cannot be alike or agreeable to Him whom the world rejected, because He testified to it that its works were evil. The tone of the mind is unworldly, does not refer to it, save to do good to it when it can. The place of the Christian is to be the epistle of Christ. Christ thus possessing the heart has a circumscribing power. The motives, thoughts, relationships of the world do not enter into the heart. But, Christ moving all within, and all being referred in the heart to Him, it carries out its own character in Him out into the world. Kept from the evil, it is the active exercise of good that is in Him, the love of God; the heart shut up to God, bin all the blessedness of God going out in the measure in which the vessel contains it.
The love is thus active. Christ has “purified to Himself a peculiar people, zealous of good works... Christ’s love was active, but it is guided by the mind of Christ. It loves the brethren as Christ did; that is, has its spring in itself, not in the object: but feeds all their sorrows and infirmities, yet is above them all so as to bear and forbear, and find in them the occasion of its holy exercises. It is alike tender in spirit and firm in consistency with the divine path, for such was Christ’s love.
It has another character: whatever its devotedness and activity, it is obedience. There cannot be a righteous will in a creature, for righteousness in a creature is obedience. Adam fell, having a will independent of God. Christ came to do the will of Him that sent Him, and in His highest devotedness His path was that of obedience. “The prince of this world cometh and hath nothing in Me, but that the world may know that I love the Father, and as the Father hath given Me commandment, so I do.” This both guides in devotedness and keeps us quiet and humble.
Our conclusion, then, is simple undivided devotedness to Christ; Christ the only object, whatever duties that motive may lead to faithfulness in; nonconformity to the world which rejected Him; a bright heavenly hope connecting itself with Christ in glory, who will come and receive us to Himself and make us like Him, so that we should be as men that wait for their Lord; His love constraining us, in all things caring for what He cares for; Christ crucified, and Christ before us as our hope, the center round which our whole life turns.
There is another point one may do well to notice, which makes the plain difference between devotedness and natural kindness. “Let your light so shine before men that they may see your good works, and glorify your Father which is in heaven.” The Lord does not tell them to let their good works shine before men; elsewhere He says the contrary. But their profession of Christ is to be so distinct that men may know to what to attribute their good works, and glorify their Father which is in heaven. What is wanted among Christians is, that through grace they should be Christians devoted, plainly devoted, in all their ways, devoted in heart and soul to Him who loved them and gave Himself for them.
(Continued from page 297)
(Concluded).

Scripture Study: Matthew 26:31-75

Matthew 26:31-35. “Then saith Jesus unto them, All ye shall be offended because of Me this night; for it is written, I will smite the Shepherd, and the sheep of the flock shall be scattered abroad. But after I am risen again, I will go before you into Galilee.” The smiting and the scattering must be, but He would again be with them when raised from the dead. (John 18:8-9). Peter knew not his own weakness; means to be faithful, and says, “Though all men shall be offended because of Thee, yet will I never be offended.” This natural energy that carried him on made a fall further than the rest, but all forsook Him and fled. All had asserted their faithfulness. “Jesus said unto him, Verily I say unto thee, that this night, before the cock crow, thou shalt deny Me thrice. Peter said unto Him, Though I should die with Thee, yet will I not deny Thee. Likewise also said all the disciples.”
Matthew 26:36-45 present the solemn scene of His interview with His Father about His atoning sufferings on the cross. It was on the cross alone that He was made sin for us. Here He isolates Himself to present to the Father in supplication the sufferings He was about to endure. Peter, James and John go on with Him. He began to be sorrowful and very heavy. Then saith He unto them, Tarry ye here, and watch with Me. And He went a little farther, and fell on His face, and prayed, saying, O My Father, if it be possible, let this cup pass from Me; nevertheless, not as I but as Thou wilt.” Luke 22:44 and Hebrews 5:7, show us how great was His agony.
He comes to His disciples and finds them asleep, and says unto Peter, “What, could ye not watch with Me one hour? Watch and pray, that ye enter not into temptation; the spirit indeed is willing, but the flesh is weak.” The flesh profiteth nothing. Satan’s power is trying to overwhelm the Lord by this trial that is to be endured, and His holy soul desires to be relieved, but it must be the Father’s will. Three times He presents His petition; and then comes to His disciples, saving, `Sleep on now, and take your rest; behold, the hour at hand, and the Son of Man is betrayed into the hand of sinners.” The Lord’s submission is perfect. Ile was distressed and full of anguish, but His communion with His Father was not interrupted. It was love to His Father and obedience to His will that brought Him there. He takes it from the Father; the cross must be endured, death must be entered into to put away our sins, and to annul him who had the power of death lie knew what this meant in its sting, as the wages of sin, and the curse of God’s judgment. If God was to be glorified in Him who had come to undertake this work, and sinners were to be saved, He must drink this terrible cup. No creature could do it, only He who was Jehovah’s fellow, God manifest in the flesh.
“Sleep on now, and take your rest.” Now, it is all settled; in the purpose of His heart it is done; He will not draw back, but will yield Himself up to His enemies. Man and Satan are the instruments to nail Him to the cross, but it is the will of God, and He will not look at second causes. What a lesson for us is here!
Matthew 26:46. “Rise, let us be going; behold, he is at hand that doth betray Me.” He is ready to submit Himself to the suffering that awaits Him.
Matthew 26:47-50. Judas comes with the multitude with swords and staves to take Him, the unresisting Lamb, and is betrayed with kisses, the pretense of love. Judas salutes Him, “Hail, Master,” and kissed Him caressingly. This son of perdition, slave of Satan, will not be warned. Jesus replies, “Friend, wherefore art thou come?” A polished hypocrite, coming as a friend. The only one to own Jesus now was His betrayer.
Matthew 26:51-68. When the Lord was praying in the garden, Peter was sleeping; he could not watch one hour. Now the Lord is yielding Himself up; Peter is fighting. The Lord rebukes him, “Put up again thy sword into his place; for all they that take the sword shall perish with the sword.” It is not the path of those who belong to a suffering and rejected Christ to fight with carnal weapons, and if they do they will reap what they sow (Gal. 6:7-8).
The Lord could, have twelve legions of angels to succor Him, but then how could the Scriptures be fulfilled that thus it must be. And to the multitude He could say, “Are ye come out as against a thief with swords and staves for to take Me? I sat daily with you teaching in the temple, and ye laid no hold on Me.” It was the hour now for Him to be taken, that the Scriptures might be fulfilled. Then all the disciples forsook Him and fled.
They led Him away to Caiaphas, the high priest, who with the scribes and elders were assembled, waiting for the victim.
But Peter followed Him afar off, and went in and sat with the servants to see the end.
They have Him now without any witness of evil against Him. They seek false witness, and many are willing to witness against Him, but they have nothing to say. At last two false witnesses think they have something against Him. It was but the witness of His power in resurrection (John 2:18-22), and declared Him to be the Son of God with power (Rom. 1:4). There is nothing to condemn Him by. The Lord is silent before them as a sheep before her shearers is dumb, till the high priest adjures Him by the living God to tell whether He is the Christ, the Son of God. Then He confesses the glory of His person, and declares that henceforth they shall see Him as Son of Man sitting on the right hand of power, and coming in the clouds of heaven. He will be Judge then. In pretended sanctity, the high priest declares He has spoken blasphemy, and all agree to put Him to death. Their malice breaks forth in insults, and they spit in His face and buffet Him. Others smite Him with the palms of their hands, saying, “Prophesy unto us, thou Christ, Who is he that smote Thee?” There is no justice for Him; they condemn and insult the one who came in grace to be their Savior, their own Messiah. He bore it all in meekness, and without a murmur. What a solemn moment for them!
Matthew 26:69-75. Peter is now to learn his weakness. His failure is seen; the flesh could not stand; he lies and curses and swears that he did not know Jesus. O, how helpless we are, unless kept in dependence on the Lord for strength. The Lord’s word is fulfilled, “Before the cock crow, thou shalt deny Me thrice.”
And Peter remembered the word of Jesus. It began to work in his conscience; the look of Jesus brought it home to him, and he went out and wept bitterly. (Luke 22:61-62).
(Continued from page 302).

Worship

Are you rejoicing in Christ as the Burnt Offering? Do you view with wonder, love and praise, the work of Christ as the delight of the heart of God? The Lord Jesus Christ was at the cross in perfect love and obedience to His Father for the vindication of God’s glory in every respect. O! what delight God has found in having His glory sustained and established in imperishable splendor, in connection with the very thing – sin that, so to speak, threatened to overturn it! The first man was disobedient unto death; here was one who was obedient unto death; whose very death was a motive to call forth His Father’s love; who could say: “Therefore doth My Father love Me because I lay down My life.” God never was, and never will be so glorified as He was at the cross. Christ has given Himself for you an offering and a sacrifice to God for a sweet-smelling savor. He has delighted the heart of God in an infinite way. God has accepted Him and His work for us, and you are now “accepted in the Beloved.” In the Sin Offering God’s claims are satisfied; in the Burnt Offering His heart is gratified. What is the result? Why, there is not a splendor in glory, or a blessing in grace that is not free to flow out now from His gratified heart upon you as His beloved child. Are you enjoying this favor?
Are you rejoicing in the personal perfections of Christ.? Think of that precious incense (Ex. 30:34), of unlimited quantity, but in which every fragrant spice was of “like weight. Those precious spices blended together in equal proportions represented the manifold perfections of Christ all equally balanced. The best saints have been unequal: perhaps full of love and lacking in righteousness; advanced in truth and lacking in grace; great energy and little spiritual discernment; or much intelligence and very little zeal. There was no inequality like that in Christ. All those graces and perfections were found in Him in unmeasured fullness, and each of them was perfectly balanced with all the others. So that as it has been beautifully said, the Lord Jesus had no character. Character is some particular thing in a man which is more prominent in him than anything else. In the Lord Jesus Christ everything was evenly balanced; no one grace shone out more than another. He was infinitely and divinely perfect.
How precious to know that God’s thoughts of us are not according to our poor thoughts of Christ, but according to His own appreciation of that peerless person and His infinite work; and the knowledge of this in the power of the Holy Spirit sets our hearts aglow with desire to know Him better, and to enjoy Him more fully. And as He becomes precious to us, as thoughts of Himself yield “unchanging, fresh delight” to our hearts, we shall find increasing joy in the presentation of Him to the Father’s heart in true spiritual worship.
If the question is asked, “How can we grow in the knowledge and appreciation of Christ?” answer, The Comforter has come to testify of Him, to glorify Him, and to receive of His and show it unto us. We have the full revelation of Christ in the Word, and the Holy Spirit is the power by which we enjoy Him. “Grieve not the Holy Spirit of God,” lest you hinder Him in His precious ministry of Christ, but rather “be filled with the Spirit.” The proof of a man being filled with the Spirit is that he is taken up with Christ.
Christ Himself occupies his heart, and Christ’s interests on earth are what he lives for. Conscious of weakness, and humble through knowledge of self, he can yet say, “To me to live is Christ.”
You may think that all this is more about communion than worship; but communion is the very basis of worship, for we can only really bring Christ to God in the measure of our own enjoyment of Him. Thank God, however small our appreciation of Christ may be, He is acceptable to God. If you have a turtle dove or pigeon (Lev. 1:14), do not hold back because it is not a sheep or a bullock; bring it; it is a “sweet savor unto the Lord.” But, dear brother, may we go on to have enlarged thoughts of Christ, and increasingly to “rejoice in Him.” I am sure that if we are individually in this state, the tone of worship in our assemblies will be raised exceedingly, and we shall not have to mourn over “flat” meeting conclusion, remember and meditate upon “the special interest of the Father in this hour.” “The hour cometh and now is, when the true worshipers shall worship the Father in spirit and in truth: for the Father seeketh such to worship Him.”
(Continued from page 305).

They That Are Christ's at His Coming: Part 2

The saints will stand before the tribunal of Christ, to receive the rewards of the kingdom; but they will appear there as glorified saints, no stain of sin shall be there, the last trace of the curse shall have been removed, the reproach of Egypt clean and forever rolled away; the death of the slain Lamb will be learned in the light of glory, and in the presence of God.
Earth may move on still in its course and projects, as it did when its light was set in the darkness of the cross; its religion may go on, too, quite compatible with its godless pursuits, until judgment break the spell of its delusion, and dissolve the dream – awaking men to the dread reality of falling “into the hands of the living God.” The light, God’s light, shall have been removed to its own proper sphere, there to reflect each its peculiar brightness, “shining as the brightness of the firmament” – “as the sun in the kingdom of their Father” – being with Him who is the sun and center of that heavenly system, undimmed, unobscured, by the clouds of unbelief or doubt.
They are with Him as He moves on in the course of the counsels of God, whether relating to the heavens above, or the earth beneath. In the presence of His glory, they shall be presented faultless, with exceeding joy.” Will He “take His great power, and reign,” swaying the scepter of righteous supremacy over a judged and renovated earth? They will be with Him there After the course of the kingdom shall be complete, and He shall have delivered up the kingdom to the Father, will He be tabernacled in the home, the dwelling-place of righteousness, in the new heavens and new earth? They that are His, will still be with Him. They are Christ’s present and eternal portion, and their place is to be “forever with the Lord.” Whether in the kingdom, or in the new heavens, and new earth, they will enjoy the rest of God in its perfection, and bear witness to His glory in the exalted sphere in which grace has set them, and for which grace has adapted them.
The hope for which we wait is not judgment, not the kingdom set up, not Israel’s restoration, or the deliverance of creation from its present bondage – all true in its place – but God’s on from heaven! He is coming, not to fulfill prophecy, but to fulfill promise: “I will come again, and receive you unto Myself, that where I am, there ye may be also.” Judgment waits for this! – the restoration of Israel, creation’s deliverance – all waits suspended until the rapture of the saints – “they that are Christ’s at His coming.”
After the Lord Jesus has gathered His own to Himself in the heavens, He will make good the prophetic Word in its bearing towards the earth, and deliver creation, bringing it into the liberty of redemption.
Well may the affections of the heart be moved at the prospect! Well may the sound of that well-known scripture reverberate in the inner man, “Behold, I come quickly!” Yes, He is corning, to appropriate to Himself that which He has purchased at His own personal cost; to whom He can say, “I have redeemed thee; thou art Mine!” to surround Himself with the trophies of redeeming love. The Father’s Will will be fully accomplished in the resurrection and glorification of those who were the objects of it; for this, they were saved. Our necessities were not the first cause; God is glorified in the redemption He has wrought, and the objects of His love are prepared for the glory that awaits them. They shall stand in the clear, unclouded light of divine righteousness, and be at home there. The robe in which they are arrayed, is divinely righteous, and meet for the occasion.
God, resting in the complacency of omnipotent love, will welcome them to Himself – His own immediate presence will be their rest – His unclouded glory, the sphere of their worship – God and the Lamb, their light and their temple: He will dwell in their midst – they, His people – He, their God.
What a prospect! Even the anticipation of such a hope lifts our spirits above the clouds and mists of earth; but we need purified hearts, to be prepared to allow the rays of that glory to reach within, and shed its light abroad there; there should be nothing allowed discordant with that holy scene; it will darken the vision, and confuse the affections: the Holy Spirit will be leading us within, to look after the house, and rid it of its corruptions and intruders, instead of opening the windows of the heart to allow the light of a new heaven to fill and irradiate it with its illuminating glory.
O that our constant position may be as those who are “turned to God from idols, to serve the living and true God and to wait for His Son from heaven,” with the heart purified, and the eve single; with staff and girdle; ready to welcome the shout in the air, whenever it may be uttered; ready! with nothing to leave behind that would retard our upward flight, nothing that may clash with that oft-expressed desire.
“Amen! Even so, come, Lord Jesus!”
Lord Jesus, come!
Nor let us longer roam
Afar from Thee, and that bright place
Where we shall see Thee face to face,
Lord Jesus, come!
Lord Jesus, come!
Thine absence here we mourn;
No joy we know apart from Thee,
No sorrow in Thy presence see.
Come, Jesus, come!
Lord Jesus, come!
And claim us as Thine own;
Our weary feet would wander o’er
This dark and sinful world no more,
Come, Savior, come!
(Continued from page 307).

Correspondence: New Birth and Possession of the Holy Spirit

Question 112:
(A) Please make clear what it means: “Ye must be born again.” What is the new birth?
(B) Does the Holy Spirit come when one has just received the new birth, or does He come later?
(C) How can one be sure he has the Holy Spirit?
(D) Will the Holy Spirit keep evil thoughts away? G. R.
Answer: (A) John 1:12, 13; 3:3-7; James 1:18; 1 Peter 1:23. These scriptures teach us that “new birth” is a new life and nature, communicated by the Holy Spirit working in the soul by the Word of God, whereby the soul is able to see and enter into the things of God. Without this operation of the Spirit we could not participate in the things of God (1 Cor. 2:14). Without it we are spiritually dead (John 5:25). One that is born again begins to feel his need of a better life, has desires after better things, and is ready to own that he is a sinner against God. Here is where we see the new life begun in the soul.
But the “new birth” does not give peace with God. We do not look within to get this; we must look at the finished work of Christ and believe that He died for our sins. John 3:14-16 shows that, as the bitten Israelites looked to the brazen serpent on the pole and lived, so the sinner must look to the Son of Man lifted up on the cross, a sacrifice for sin, and in that work of atonement see the love of God, giving eternal life to whosoever believeth on Christ. God raised Christ from the dead to prove His acceptance of that work, and by believing it “we have peace with God through our Lord Jesus Christ” (Rom. 4:23; 5:1).
If you are anxious to be saved, do not trouble yourself, trying to understand the new birth, but come to Jesus as you are, in your sins. He will show you how He died for you, and that believing on Him your sins are all forgiven (Acts 10:43; 13:38-39), and that you have eternal life (John 3:16,36; 5:24).
The Holy Spirit is not given to us because we are born again, but because we believe the gospel of our salvation (Eph. 1:13). Cornelius and his house were pious Gentiles, their prayers and alms were accepted of God (Acts 10:1-3); this is proof that they were born again; but it was when they believed the gospel that the Holy Spirit fell upon them (Acts 10:43-44; John 7:39). The Holy Spirit comes to dwell in the believer, witnessing that he is washed in the blood of Christ (Heb. 10:14-15). The reason, therefore, that some who are born of God are not sealed by the Spirit, must be, that they have not believed the gospel of their salvation (Eph. 1:13).
If we believe God’s Word, we must believe that we have received the Holy Spirit. We know that our sins are forgiven and that we are children of God, and that we have eternal life, because the Word of God says so and in the same way we know the Holy Spirit dwells in us. His love is shed abroad in our hearts by the Holy Spirit which is given unto us (Rom. 5:6). The Spirit itself bears witness with our spirit, that we are the children of God, and we say “Abba, Father,” by the Spirit (Rom. 8:15-17).
The Holy Spirit is the power of the new life (Rom. 8:2), our power for living for Christ. But He does not direct us to look at ourselves. We are to look off to Jesus. We know the flesh is in us and always will be while we are in this world (John 3:6). So we are taught that the old man is crucified with Christ, and we are to reckon ourselves dead indeed unto sin, but alive unto God, through Jesus Christ our Lord. (Rom. 6:6,11). The Holy Spirit dwelling in us desires to occupy us with Christ, and in this way we are to overcome evil in our thoughts and ways. The moment we get the Lord Jesus before our minds, our evil thoughts and Satan also flee away. Walking in the Spirit is occupation with Christ, and the fruit of the Spirit grows from this. (Gal. 5:16,22-23,25).
Courtesy of BibleTruthPublishers.com. Most likely this text has not been proofread. Any suggestions for spelling or punctuation corrections would be warmly received. Please email them to: BTPmail@bibletruthpublishers.com.